Re: [obm-l] f(f(x))=e^(-x)

2023-09-24 Por tôpico Carlos Gustavo Tamm de Araujo Moreira
Por outro lado existem funções (necessariamente descontínuas) de R em R que
satisfazem essa equação funcional. Vou tentar
descrever uma delas.
Seja a=LambertW(1)~0,5671432904... a solução real de e^(-x)=x, como o Ralph
mencionou. Vou escrever g(x)=e^(-x).
Queremos f(f(x))=g(x). Vamos definir recursivamente g^n(x) por g^0(x)=x,
g^(n+1)(x)=g(g^n(x)). Vou usar o seguinte fato,
que deixo como exercício: para todo y real diferente de a existe um único x
em (-infinito,0] e um único n natural tais que y=g^n(x).
Definimos f(x) para x em (-infinito,-1] como f(x)=-(x+1)/x (assim f leva
(-infinito,-1] em (-1,0]), e definimos f em (-1,0] para
termos f(f(x))=g(x) se x está em (-infinito,-1], ou seja, f(y)=e^(1/(y+1))
para y em (-1,0]. A partir daí, se y=g^n(x) com n
natural e x em (-infinito,0], definimos f(y)=g^n(f(x)).Finalmente definimos
f(a)=a.
Abraços,
Gugu

On Sat, Sep 23, 2023 at 9:32 PM Ralph Costa Teixeira 
wrote:

> P.S.: Existe um argumento simples para mostrar que NÃO existe *f:R->R*
> *contínua* com f(f(x))=g(x) que serve para qualquer g estritamente
> decrescente (como esta g(x)=e^(-x)). Funciona assim:
>
> i) f teria que ser bijetiva. Afinal, f(a)=f(b) implica f(f(a))=f(f(b)) e,
> daqui (g bijetiva) vem a=b.
> ii) Mas f bijetiva continua em R implica f (estritamente) monótona!
> iiia) se f (estritamente) crescente, absurdo, pois f(f(x))=g(x) seria
> crescente;
> iiib) se f (estritamente) decrescente, absurdo, pois f(f(x))=g(x) seria
> crescente de novo!
>
> Ralph.
>
> On Sat, Sep 23, 2023 at 9:03 PM Ralph Costa Teixeira 
> wrote:
>
>> Tecnicamente esta f existe: você pode tomar f:{a}->{a} dada por f(a)=a
>> onde a=LambertW(1)~0,56714...  (a raiz de e^(-x)=x).  ;D ;D ;D
>>
>> Ou melhor dizendo: o problema fala algo sobre o domínio dessa f? Ou dela
>> ser contínua, pelo menos?
>>
>>
>> On Sat, Sep 23, 2023 at 8:25 PM Luís Lopes  wrote:
>>
>>> Saudações,
>>>
>>> Existe tal f? Se sim, qual seria?
>>>
>>> Recebi um e-mail com esta pergunta, sem maiores detalhes. Pelo e-mail,
>>> tal f não existe. Problema encontrado pelo remetente no YouTube.
>>>
>>> Luís
>>>
>>>
>>> --
>>> Esta mensagem foi verificada pelo sistema de antivírus e
>>>  acredita-se estar livre de perigo.
>>>
>>>
>>> =
>>> Instruções para entrar na lista, sair da lista e usar a lista em
>>> http://www.mat.puc-rio.br/~obmlistas/obm-l.html
>>> =
>>>
>>
> --
> Esta mensagem foi verificada pelo sistema de antivírus e
> acredita-se estar livre de perigo.

-- 
Esta mensagem foi verificada pelo sistema de antiv�rus e
 acredita-se estar livre de perigo.



[obm-l] Re: [obm-l] Uma recorrência diferente

2023-04-06 Por tôpico Carlos Gustavo Tamm de Araujo Moreira
Caro Vanderlei,
Não parece haver uma fórmula fechada muito simples. Veja
https://oeis.org/A85 para várias referências
sobre essa sequência.
Abraços,
Gugu

On Wed, Apr 5, 2023 at 11:41 PM Professor Vanderlei Nemitz <
vanderma...@gmail.com> wrote:

> Oi, mestres!
>
> Estava resolvendo um problema de combinatória e obtive essa recorrência:
>
> *x(n) = x(n - 1) + (n - 1).x(n - 2), com x1 = 1 e x2 = 2*.
>
> Por exemplo, x3 = x2 + 2.x1 e x9 = x8 + 8.x7
>
> Como resolver quando os coeficientes não são todos constantes?
>
> Apenas como curiosidade, o problema que originou a recorrência é:
> (IME - RJ) - Um professor dá um teste surpresa para uma turma de 9 alunos,
> e diz que o teste pode ser feito sozinho ou em grupos de 2 alunos. De
> quantas formas a turma pode ser organizar para fazer o teste? (Por exemplo,
> uma turma de 3 alunos pode ser organizar de 4 formas e uma turma de 4
> alunos pode se organizar de 10 formas)
>
>
>
> 
>  Não
> contém vírus.www.avast.com
> 
> <#m_4118911927943204904_DAB4FAD8-2DD7-40BB-A1B8-4E2AA1F9FDF2>
>
> --
> Esta mensagem foi verificada pelo sistema de antivírus e
> acredita-se estar livre de perigo.

-- 
Esta mensagem foi verificada pelo sistema de antiv�rus e
 acredita-se estar livre de perigo.



Re: [obm-l] Arimetica Diofanto

2022-02-01 Por tôpico Carlos Gustavo Tamm de Araujo Moreira
Vou enviar uma solução resumida:
Se 3^x-5^y=2, vamos testar os menores valores de y: se y=0 então 3^x=3 e
x=1. Se y=1 então 3^x=7, o que não dá solução inteira.
Se y=2 então 3^x=27 e x=3. Vamos então supor y=2+b>2, o que dá 3^x>27, logo
x=3+a>3, e podemos escrever a equação como
3^3(3^a-1)=5^2(5^b-1). Como a ordem de 3 módulo 5^2 é 20, segue que 20|a,
logo 3^a-1 é múltiplo de 3^20-1, e portanto é múltiplo de 61.
Como a ordem de 5 módulo 61 é 30, segue que 30|b, logo 5^b-1 é múltiplo de
5^30-1, e portanto é múltiplo de 31. Como a ordem de 3
módulo 31 é 30, segue que 30|a, logo 3^a-1 é múltiplo de 3^30-1, e portanto
é múltiplo de 271. Como a ordem de 5 módulo 271 é 27,
segue que 27|b, e como 30|b segue que 54|b, logo 5^b-1 é múltiplo de
5^54-1, e portanto é múltiplo de 81, absurdo, pois, como 3^a-1 não
é múltiplo de 3,  3^3(3^a-1) não é múltiplo de 81.
Abraços,
 Gugu
é múltiplo de 81

On Fri, Jan 28, 2022 at 5:28 PM Carlos Gustavo Tamm de Araujo Moreira <
g...@impa.br> wrote:

> Mas acho que lá uma solução está incompleta e as outras duas erradas...
>
> On Fri, Jan 28, 2022 at 5:11 PM Gabriel Torkomian 
> wrote:
>
>> https://artofproblemsolving.com/community/q1h2640462p22841017
>> Tem no aops
>>
>> Em sex., 28 de jan. de 2022 10:32, Israel Meireles Chrisostomo <
>> israelmchrisost...@gmail.com> escreveu:
>>
>>> 3^x-5^y=2
>>>
>>> Em sex., 28 de jan. de 2022 09:53, Esaú Gomes 
>>> escreveu:
>>>
>>>> E qual a equação?
>>>>
>>>> On Wed, Jan 26, 2022 at 3:33 PM Israel Meireles Chrisostomo <
>>>> israelmchrisost...@gmail.com> wrote:
>>>>
>>>>> Olá pessoal, recentimente estava estudando e me deparei com uma
>>>>> equação diofantina.eu tentei resolve-la mas ñ sei se está correta a
>>>>> solução ou incompleta, vcs poderiam por favor me ajudar a fechar o
>>>>> argumento?ñ quero outra solução só quero fazer da minha solução uma 
>>>>> solução
>>>>> top.Tenho a impressão que falta alguma coisa.
>>>>>
>>>>> --
>>>>> Esta mensagem foi verificada pelo sistema de antivírus e
>>>>> acredita-se estar livre de perigo.
>>>>
>>>>
>>>> --
>>>> Esta mensagem foi verificada pelo sistema de antivírus e
>>>> acredita-se estar livre de perigo.
>>>
>>>
>>> --
>>> Esta mensagem foi verificada pelo sistema de antivírus e
>>> acredita-se estar livre de perigo.
>>
>>
>> --
>> Esta mensagem foi verificada pelo sistema de antivírus e
>> acredita-se estar livre de perigo.
>
>

-- 
Esta mensagem foi verificada pelo sistema de antiv�rus e
 acredita-se estar livre de perigo.



Re: [obm-l] Arimetica Diofanto

2022-01-28 Por tôpico Carlos Gustavo Tamm de Araujo Moreira
Mas acho que lá uma solução está incompleta e as outras duas erradas...

On Fri, Jan 28, 2022 at 5:11 PM Gabriel Torkomian  wrote:

> https://artofproblemsolving.com/community/q1h2640462p22841017
> Tem no aops
>
> Em sex., 28 de jan. de 2022 10:32, Israel Meireles Chrisostomo <
> israelmchrisost...@gmail.com> escreveu:
>
>> 3^x-5^y=2
>>
>> Em sex., 28 de jan. de 2022 09:53, Esaú Gomes 
>> escreveu:
>>
>>> E qual a equação?
>>>
>>> On Wed, Jan 26, 2022 at 3:33 PM Israel Meireles Chrisostomo <
>>> israelmchrisost...@gmail.com> wrote:
>>>
 Olá pessoal, recentimente estava estudando e me deparei com uma equação
 diofantina.eu tentei resolve-la mas ñ sei se está correta a solução ou
 incompleta, vcs poderiam por favor me ajudar a fechar o argumento?ñ quero
 outra solução só quero fazer da minha solução uma solução top.Tenho a
 impressão que falta alguma coisa.

 --
 Esta mensagem foi verificada pelo sistema de antivírus e
 acredita-se estar livre de perigo.
>>>
>>>
>>> --
>>> Esta mensagem foi verificada pelo sistema de antivírus e
>>> acredita-se estar livre de perigo.
>>
>>
>> --
>> Esta mensagem foi verificada pelo sistema de antivírus e
>> acredita-se estar livre de perigo.
>
>
> --
> Esta mensagem foi verificada pelo sistema de antivírus e
> acredita-se estar livre de perigo.

-- 
Esta mensagem foi verificada pelo sistema de antiv�rus e
 acredita-se estar livre de perigo.



Re: [obm-l] Re: transcendencia

2021-04-02 Por tôpico Carlos Gustavo Tamm de Araujo Moreira
Por outro lado, se v é algébrico e u é algébrico sobre o corpo Q(v) então u
é algébrico.
O meu exemplo é um pouco "roubado": parece que b satisfaz a equação
(a^2-2)b+a(a^2-2)=0, mas, como
a^2-2=0, essa equação é identicamente nula...
Abraços,
 Gugu

On Fri, Apr 2, 2021 at 4:57 PM Israel Meireles Chrisostomo <
israelmchrisost...@gmail.com> wrote:

> Muito obrigado professor gugu
>
> Em sex, 2 de abr de 2021 16:00, Carlos Gustavo Tamm de Araujo Moreira <
> g...@impa.br> escreveu:
>
>> Não. Se a=sqrt(2) e b=pi então a^3+b.a^2-2a-2b=0, por exemplo.
>>
>> Em sex, 2 de abr de 2021 15:31, Israel Meireles Chrisostomo <
>> israelmchrisost...@gmail.com> escreveu:
>>
>>>   Se u é um número transcendente e v é um número, se u,v são
>>> algebricamente dependentes então v é transcendente?
>>>
>>>
>>> Em sex., 2 de abr. de 2021 às 14:58, Israel Meireles Chrisostomo <
>>> israelmchrisost...@gmail.com> escreveu:
>>>
>>>> Se a é um número transcendente e v é um número, se u,v são
>>>> algebricamente dependentes então v é transcendente?
>>>>
>>>> --
>>>> Israel Meireles Chrisostomo
>>>>
>>>
>>>
>>> --
>>> Israel Meireles Chrisostomo
>>>
>>> --
>>> Esta mensagem foi verificada pelo sistema de antivírus e
>>> acredita-se estar livre de perigo.
>>
>>
>> --
>> Esta mensagem foi verificada pelo sistema de antivírus e
>> acredita-se estar livre de perigo.
>
>
> --
> Esta mensagem foi verificada pelo sistema de antivírus e
> acredita-se estar livre de perigo.

-- 
Esta mensagem foi verificada pelo sistema de antiv�rus e
 acredita-se estar livre de perigo.



Re: [obm-l] Re: transcendencia

2021-04-02 Por tôpico Carlos Gustavo Tamm de Araujo Moreira
Não. Se a=sqrt(2) e b=pi então a^3+b.a^2-2a-2b=0, por exemplo.

Em sex, 2 de abr de 2021 15:31, Israel Meireles Chrisostomo <
israelmchrisost...@gmail.com> escreveu:

>   Se u é um número transcendente e v é um número, se u,v são
> algebricamente dependentes então v é transcendente?
>
>
> Em sex., 2 de abr. de 2021 às 14:58, Israel Meireles Chrisostomo <
> israelmchrisost...@gmail.com> escreveu:
>
>> Se a é um número transcendente e v é um número, se u,v são algebricamente
>> dependentes então v é transcendente?
>>
>> --
>> Israel Meireles Chrisostomo
>>
>
>
> --
> Israel Meireles Chrisostomo
>
> --
> Esta mensagem foi verificada pelo sistema de antivírus e
> acredita-se estar livre de perigo.

-- 
Esta mensagem foi verificada pelo sistema de antiv�rus e
 acredita-se estar livre de perigo.



[obm-l] Re: [obm-l] transcendência

2020-07-08 Por tôpico Carlos Gustavo Tamm de Araujo Moreira
Caro Israel,
Sim. Suponha que x e y são algebricamente dependentes sobre um corpo de
base K. Se y é algébrico, K(y)|K é uma extensão algébrica. Como x é raiz de
uma equação polinomial com coeficientes em K(y) (pois  x e y são
algebricamente dependentes), a extensão K(x,y)=K(y)(x)|K(y) é algébrica.
Portanto, como K(x,y)|K(y) e K(y)|K são extensões algébricas, K(x,y)|K
também é algébrica, e, em particular, x também é algébrico. Portanto, se x
é transcendente então y também é transcendente.
Abraços,
 Gugu


On Wed, Jul 8, 2020 at 2:04 PM Israel Meireles Chrisostomo <
israelmchrisost...@gmail.com> wrote:

> se dois números são algebricamente dependentes e se um deles é
> transcendente então isso implica o outro seja transcendente?isso me
> parece meio óbvio mas nao sei como provar
>
> --
> Israel Meireles Chrisostomo
>
> --
> Esta mensagem foi verificada pelo sistema de antivírus e
> acredita-se estar livre de perigo.

-- 
Esta mensagem foi verificada pelo sistema de antiv�rus e
 acredita-se estar livre de perigo.



[obm-l] Re: [obm-l] Re: [obm-l] Re: [obm-l] Re: [obm-l] Teoria dos números

2020-03-17 Por tôpico Carlos Gustavo Tamm de Araujo Moreira
Acho que a pergunta deve ser qual é o maior inteiro positivo que divide
essa expressão para todo valor de n ao mesmo tempo.

On Tue, Mar 17, 2020 at 6:58 AM Pedro José  wrote:

> Bom dia!
> Se você considerar a expressão n(427-90n-70n^2+45n^3+18n^4)
> D=|n(427-90n-70n^2+45n^3+18n^4)|
> Por exemplo, n=1
> D=330.
> Agora se liberar n para variar D tende a oo.
>
> Se n for raiz da expressão, também tende a oi, pois qualquer inteiro
> divide 0.
>
>
> Em seg, 16 de mar de 2020 22:16, Israel Meireles Chrisostomo <
> israelmchrisost...@gmail.com> escreveu:
>
>> não entendi
>>
>> Em seg., 16 de mar. de 2020 às 22:01, Pedro José 
>> escreveu:
>>
>>> Para um dado n é o módulo do valor da expressão.
>>>
>>> Em seg, 16 de mar de 2020 21:49, Pedro José 
>>> escreveu:
>>>
 Boa noite!
 O módulo dessa expressão tende a oo. Não existe máximo.
 Saudações,
 PJMS

 Em seg, 16 de mar de 2020 20:36, Israel Meireles Chrisostomo <
 israelmchrisost...@gmail.com> escreveu:

> Qual é o maior inteiro que divide  n (427 - 90n - 70n^2 + 45n^3 +
> 18n^4)?
>
> --
> Israel Meireles Chrisostomo
>
> --
> Esta mensagem foi verificada pelo sistema de antivírus e
> acredita-se estar livre de perigo.


>>> --
>>> Esta mensagem foi verificada pelo sistema de antivírus e
>>> acredita-se estar livre de perigo.
>>
>>
>>
>> --
>> Israel Meireles Chrisostomo
>>
>> --
>> Esta mensagem foi verificada pelo sistema de antivírus e
>> acredita-se estar livre de perigo.
>
>
> --
> Esta mensagem foi verificada pelo sistema de antivírus e
> acredita-se estar livre de perigo.

-- 
Esta mensagem foi verificada pelo sistema de antiv�rus e
 acredita-se estar livre de perigo.



Re: [obm-l]

2019-11-16 Por tôpico Carlos Gustavo Tamm de Araujo Moreira
Oi pessoal,
Eu achava que sairia mais fácil olhando em Z[i.sqrt(2)], mas mesmo assim dá
trabalho. Há uma discussão bem mais completa sobre esse problema (que caiu
em uma olimpíada polonesa) em
https://mathoverflow.net/questions/250312/diophantine-equation-3n-1-2x2
Em particular há uma solução que envolve olhar uma recorrência (ligada à
equação de Pell) módulo 27 e módulo 17.
Abraços,
   Gugu

On Fri, Nov 15, 2019 at 5:17 PM Pedro José  wrote:

> Boa tarde!
> Esdras,
> Boa sacada!
> (b^2+1)^2=b^4+2b^2+1=b^4+(3^k)^2.
> Depois ternos pitagóricos sem restrição de primitivo.
> Aí subtraindo a primeira da segunda ou somando dão quadrados perfeitos em
> p e q. Basta igualar a1 ou então tira a raiz e iguala u^2 - v^2. Sai que
> p-q=1.
> Aí fica fácil.
> Parabéns!
> Falta achar uma lei de geração para outras soluções ou uma restrição
> (acredito mais nessa) para a e b ímpares.
> Saudações,
> PJMS
>
> Em sex, 15 de nov de 2019 13:05, Pedro José 
> escreveu:
>
>> Bom dia!
>> Esdras,
>> grato, vou tentar seguir a linha.
>>
>> Douglas,
>> Tentei combinar mod 8 com mod9 e não saiu uma restrição.
>>
>> Carlos Gustavo,
>> teria como propor material sobre o tema que você levantou. Compreendi a
>> fatoração, mas não como seriam os primos nesse universo.
>> Ainda sem tempo para tentar uma restrição.
>>
>> Saudações,
>> PJMS
>>
>>
>> Em ter., 12 de nov. de 2019 às 23:21, Esdras Muniz <
>> esdrasmunizm...@gmail.com> escreveu:
>>
>>> O caso "a" par eu fiz assim: a=2k, daí, (3^k)^2+ b^4=(d^2+1)^2, então vc
>>> usa que para algum par p, q, com 0>> p^2-q^2. Daí vc mostra que p=q+1 e em seguida que q=1.
>>>
>>> Em ter, 12 de nov de 2019 22:29, Prof. Douglas Oliveira <
>>> profdouglaso.del...@gmail.com> escreveu:
>>>
 Será que não sai usando somente congruência módulo 8?

 Em ter., 12 de nov. de 2019 às 20:07, Pedro José 
 escreveu:

> Boa noite!
> Esdras,
> tem como você postar, mesmo para o caso apenas de n par?
>
> Grato!
>
> Saudações,
> PJMS.
>
> Em ter., 12 de nov. de 2019 às 19:52, Pedro José 
> escreveu:
>
>> Boa noite!
>> Carlos Gustavo,
>> grato pela luz, estava tão obsecado e só rodando em círculos, tal
>> qual patrulha perdida.
>>
>> Saudações,
>> PJMS
>>
>> Em ter., 12 de nov. de 2019 às 19:19, Esdras Muniz <
>> esdrasmunizm...@gmail.com> escreveu:
>>
>>> Dá para mostrar que a única solução com a e b pares é (2, 2). Agora
>>> com a e b ímpares, não consegui.
>>>
>>> Em ter, 12 de nov de 2019 18:19, Pedro José 
>>> escreveu:
>>>
 Boa noite!
 Agora captei vosso pensamento.
 Só que ao transformar a equação em uma equação de Pell, nós
 maculamos a função 3^n.
 Em verdade a solução para a par a= 2n, seria (2,2); pois, como
 mencionara anteriormente se a é par, b também o é.
 Só que quando procuramos as outras soluções, baseando-se na
 propriedade de que a norma em Q [RAiz(A)] conserva a multiplicação. Só 
 que
 quando eu pego a solução
 3 + 2 Raiz(2) e elevo ao quadrado 17 + 12 Raiz(2). Se eu pegar
 17^2-2*12^2=1 eu atendo x^2 - 2Y^2=1. E assim sucessivamente. Mas não
 existe n inteiro tal que 3^n=17, então não é uma solução da equação
 original.
 Creio que seja um pouco mais complicada a solução. Pois o difícil é
 saber quando atende também a 3^n.
 Acredito que deva haver uma forma de restringir a essas soluções,
 pois, definir em que condições a solução terá x como uma potência de 3 
 seja
 bem difícil.
 Estou apanhando mais do que mala velha em véspera de viagem.
 Se alguém postar uma solução, me ajudaria bastante.

 Saudações,
 PJMS


 Saudações,
 PJMS.



 Em ter., 12 de nov. de 2019 às 17:25, Pedro José <
 petroc...@gmail.com> escreveu:

> Boa tarde!
> Douglas,
> perdoe-me pela minha miopia, mas você poderia detalhar melhor onde
> entra a equação de Pell?
> A equação de Pell não é x^2-Dy^2 = N?
> Se a é par b é par e se a ímpar b é ímpar para atender mod8,
> Não consegui captar a sugestão.
>
> Saudações,
> PJMS
>
> Em ter., 12 de nov. de 2019 às 16:50, Prof. Douglas Oliveira <
> profdouglaso.del...@gmail.com> escreveu:
>
>> Hum, então, vamos analisar o caso de a ser par do tipo 2n.
>>
>> Assim podemos escrever que (3^n+b(sqrt2))(3^n-b(sqrt2))=1
>> Dai através da solução mínima que o Pedro fez, como (1,1) por
>> exemplo, da pra ver que são infinitas soluções usando a equação de 
>> Pell.
>>
>> Abraco
>> Douglas Oliveira.
>>
>>
>>
>> Em dom, 10 de nov de 2019 19:33, gilberto azevedo <

Re: [obm-l]

2019-11-12 Por tôpico Carlos Gustavo Tamm de Araujo Moreira
Há uma menção a esse problema em
https://math.stackexchange.com/questions/2826307/integer-solutions-of-3n-1-2m2
Uma sugestão é usar o fato de que Z[i.sqrt(2)] é um domínio de fatoração
única, e escrever 1+2b^2 como (1+b.i.sqrt(2))(1-b.i.sqrt(2)).
Notem que 3 se fatora aí como (1+i.sqrt(2))(1- i.sqrt(2)).
Abraços,
 Gugu

On Tue, Nov 12, 2019 at 7:21 PM Pedro José  wrote:

> Boa noite!
> Agora captei vosso pensamento.
> Só que ao transformar a equação em uma equação de Pell, nós maculamos a
> função 3^n.
> Em verdade a solução para a par a= 2n, seria (2,2); pois, como mencionara
> anteriormente se a é par, b também o é.
> Só que quando procuramos as outras soluções, baseando-se na propriedade de
> que a norma em Q [RAiz(A)] conserva a multiplicação. Só que quando eu pego
> a solução
> 3 + 2 Raiz(2) e elevo ao quadrado 17 + 12 Raiz(2). Se eu pegar
> 17^2-2*12^2=1 eu atendo x^2 - 2Y^2=1. E assim sucessivamente. Mas não
> existe n inteiro tal que 3^n=17, então não é uma solução da equação
> original.
> Creio que seja um pouco mais complicada a solução. Pois o difícil é saber
> quando atende também a 3^n.
> Acredito que deva haver uma forma de restringir a essas soluções, pois,
> definir em que condições a solução terá x como uma potência de 3 seja bem
> difícil.
> Estou apanhando mais do que mala velha em véspera de viagem.
> Se alguém postar uma solução, me ajudaria bastante.
>
> Saudações,
> PJMS
>
>
> Saudações,
> PJMS.
>
>
>
> Em ter., 12 de nov. de 2019 às 17:25, Pedro José 
> escreveu:
>
>> Boa tarde!
>> Douglas,
>> perdoe-me pela minha miopia, mas você poderia detalhar melhor onde entra
>> a equação de Pell?
>> A equação de Pell não é x^2-Dy^2 = N?
>> Se a é par b é par e se a ímpar b é ímpar para atender mod8,
>> Não consegui captar a sugestão.
>>
>> Saudações,
>> PJMS
>>
>> Em ter., 12 de nov. de 2019 às 16:50, Prof. Douglas Oliveira <
>> profdouglaso.del...@gmail.com> escreveu:
>>
>>> Hum, então, vamos analisar o caso de a ser par do tipo 2n.
>>>
>>> Assim podemos escrever que (3^n+b(sqrt2))(3^n-b(sqrt2))=1
>>> Dai através da solução mínima que o Pedro fez, como (1,1) por exemplo,
>>> da pra ver que são infinitas soluções usando a equação de Pell.
>>>
>>> Abraco
>>> Douglas Oliveira.
>>>
>>>
>>>
>>> Em dom, 10 de nov de 2019 19:33, gilberto azevedo 
>>> escreveu:
>>>
 [HELP]

 Achas todos os pares (a,b) inteiros positivos tais que :
 3^a = 2b² + 1.


 --
 Esta mensagem foi verificada pelo sistema de antivírus e
 acredita-se estar livre de perigo.
>>>
>>>
>>> --
>>> Esta mensagem foi verificada pelo sistema de antivírus e
>>> acredita-se estar livre de perigo.
>>
>>
> --
> Esta mensagem foi verificada pelo sistema de antivírus e
> acredita-se estar livre de perigo.

-- 
Esta mensagem foi verificada pelo sistema de antiv�rus e
 acredita-se estar livre de perigo.



[obm-l] Re: [obm-l] teoria da Medida - provar que f é contínua

2019-07-02 Por tôpico Carlos Gustavo Tamm de Araujo Moreira

Caro Artur,

Seja d>0 pequeno. Existem K compacto e U aberto com K C A C U e 
m(A)-d

(A interseção (x+A)) C (K interseção (x+K)) U (A\K) U (x+(A\K)), temos 
f(x)=m(A interseção (x+A))

m(K interseção (x+K))>f(x)-2d, para todo x em R^n.

Seja agora V aberto contendo (K interseção (x+K)) com m(V)Existe r>0 tq. |x-y|interseção (y+K)) C V,


donde f(y)=m((K interseção V) 
união ((y+K) interseção V))=m(K interseção V)+m((y+K) interseção V)-m(K 
interseção (y+K)),


logo, como m(K interseção V)>=m(K interseção (x+K))>f(x)-2d e m((y+K) 
interseção V)>=m((K interseção (x+K))+y-x)=m(K interseção 
(x+K))>f(x)-2d, temos


f(y)>=m(K interseção (y+K))>=m(K interseção V)+m((y+K) interseção 
V)-m(V)>=2f(x)-4d-m(V)>f(x)-5d. Isso dá a continuidade.


Abraços,

  Gugu

Em 02/07/2019 09:54, Artur Costa Steiner escreveu:
Será que alguém aqui pode me ajudar com isso, ou sabe de algum link 
correlato:?


Sejam  m a medida de Lebesgue, A um subconjunto de R^n com 0 < m(A) < 
oo e f: R^n ---> [0, oo) dada por f(x) =m(A intersecção (x + A)), onde 
x + A = {x + a | a está em A} a translação de A pelo vetor x de R^n.


Mostre que f é contínua.

Este teorema provê uma prova bem simples de que, se m(A) > 0, então A 
- A = {a1 - a2  | a1 e a2 estão em A}  contém uma bola com centro na 
origem. (conheço uma outra prova, que é mais trabalhosa), E esta 
conclusão sobre a bola é utilizada numa linda prova de que o conjunto 
de Vitali (que é bem patológico) não é Lebesgue mensurável.



Obrigado

Artur


--
Esta mensagem foi verificada pelo sistema de antiv�rus e
acredita-se estar livre de perigo. 


--
Esta mensagem foi verificada pelo sistema de antiv�rus e
acredita-se estar livre de perigo.



Re: [obm-l] PELO SIM, PELO

2005-09-14 Por tôpico Carlos Gustavo Tamm de Araujo Moreira
   Oi Nicolau,
   Mas se eu perguntar a ele (e isso, nessa interpretação,  é uma pergunta só): 
diga sobre cada um de vocês se é honesto ou não (na verdade a pergunta da
minha solução não é bem essa, mas algo como se eu perguntasse a você sobre
se cada um de vocês é honesto ou não, o que você responderia em cada caso ?) 
- ele decidirá como quiser no início se vai mentir sobre todos ou se vai dizer 
a verdade sobre todos. A questão é se valem perguntas compostas ou não... 
   Abraços,
Gugu

P.S.: Se não me engano o Elon escreveu sobre isso (talvez com pseudônimo) numa
RPM antiga e no livro Meu professor de matemática e outras histórias.


On Wed, Sep 14, 2005 at 04:35:28PM -0300, [EMAIL PROTECTED] wrote:
 Por
 outro lado, eu não entendi bem a sua última objeção: se os desonestos
 decidirem, em função da minha pergunta,
 entre dizer a verdade em toda a sua resposta ou só fazer afirmações falsas em
 sua resposta então a minha solução funciona.

Acho que não. Suponha que um desonesto pode mentir ou dizer a verdade,
a única restrição sendo que ele deve alternar entre mentiras e verdades.
Ele decide que vai mentir ou dizer a verdade na primeira pergunta em função
da pergunta de uma forma complicada qualquer que inclui os seguintes casos
particulares:

Se a pergunta for
Você é honesto?,
o desonesto vai MENTIR.

Se a pergunta for
Se eu perguntasse a você 'Você é honesto?', o que você responderia?,
o desonesto vai DIZER A VERDADE.

Por exemplo, o critério dele pode ser o seguinte: se a última letra
da pergunta for abcdefghijklm, ele dirá a verdade. Se for nopqrstuvwxyz,
ele mentirá.

Gugu encontra este desonesto e faz a pergunta:
Se eu perguntasse a você 'Você é honesto?', o que você responderia?.
O desonesto (que não é burro) pensa: se ele me fizesse a pergunta curta
eu mentiria e diria SIM. Como eu vou dizer a verdade para a pergunta
longa (a que Gugu realmente fez) vou responder... SIM!.

Gugu não pode com isso deduzir nada sobre a honestidade do cara com
esta primeira pergunta e resposta. Claro que se Gugu repetir a mesma
pergunta a situação já é outra, pois agora o desonesto está obrigado a mentir.

[]s, N.

PS: Onde foi, exatamente, que o Elon discutiu este tipo de coisa?


=
Instruções para entrar na lista, sair da lista e usar a lista em
http://www.mat.puc-rio.br/~nicolau/olimp/obm-l.html
=


=
Instruções para entrar na lista, sair da lista e usar a lista em
http://www.mat.puc-rio.br/~nicolau/olimp/obm-l.html
=


Re: [obm-l] Desigualdade com complexos

2005-07-25 Por tôpico Carlos Gustavo Tamm de Araujo Moreira
   Caro Danilo,
   Fazendo z=a+bi, queremos provar que 
(e^a.cosb-1)^2+(e^a.senb)^2=(e^((a^2+b^2)^(1/2)-1)^2, o que equivale a
e^(2a)-2e^a.cosb=e^(2(a^2+b^2)^(1/2))-2e^((a^2+b^2)^(1/2)).
Vamos mostrar que 0=x=y implica e^(2y)-2e^y-(e^(2x)-2e^x=e^x(y^2-x^2).
Escrevendo y=x+h, isso equivale a e^(x+2h)-2e^h-e^x+2=h^2+2hx (após dividir
por e^x). Isso pode ser escrito como e^x(e^h-1)(e^h+1)-2(e^h-1)=h(h+2x), ou
seja, (e^h-1)(e^(x+h)+e^x-2)=h(h+2x), mas e^h-1=h,e^x-1=x e 
e^(x+h)-1=x+h, donde e^(x+h)+e^x-2=2x+h e (e^h-1)(e^(x+h)+e^x-2)=h(h+2x).
Agora, usamos isso para y=(a^2+b^2)^(1/2) e x=a, obtendo
e^(2(a^2+b^2)^(1/2))-2e^((a^2+b^2)^(1/2))-(e^(2a)-2e^a)=e^a.b^2. Queremos
provar que o lado esquerdo e' =2e^a.(1-cosb), e logo (dividindo por e^a)
basta mostrar que b^2=2(1-cosb), mas 1-cosb=2(sen(b/2))^2=2.(b/2)^2=b^2/2,
donde b^2=2(1-cosb), cqd.
   Abraços,
 Gugu  


 
Pessoal , alguem sabe fazer essa ?
prove que para todo numero complexo z , vale
 |e^z-1| menor ou igual a  e^|z|-1

   Abs.

=
Instruções para entrar na lista, sair da lista e usar a lista em
http://www.mat.puc-rio.br/~nicolau/olimp/obm-l.html
=


Re: RES: [obm-l] Desigualdade com complexos

2005-07-25 Por tôpico Carlos Gustavo Tamm de Araujo Moreira
   Caro Pedro,
   Muito bacana esta solução (embora ligeiramente menos elementar que a
minha) - eu devia ter visto isso...
   Abraços,
 Gugu

P.S.: Claro que dá para tirar os -1, mas aí fica bem mais trivial:
Isso segue, por exemplo, de |e^(a+bi)|=e^a=e^((a^2+b^2)^(1/2)).


|e^z - 1| = |z + z^2/2 + z^3/3! + |
e^|z| - 1 = |z| + |z|^2/2 + |z|^3/3! + ...

Truncando-se as somas, usando desigualdade triangular e tomando o limite,
obtem-se o resultado. Poderia omitir o -1 nesse caso?

Um abraço. Pedro.

-Mensagem original-
De: [EMAIL PROTECTED] [mailto:[EMAIL PROTECTED] Em nome
de Carlos Gustavo Tamm de Araujo Moreira
Enviada em: Monday, July 25, 2005 12:47 PM
Para: obm-l@mat.puc-rio.br
Assunto: Re: [obm-l] Desigualdade com complexos

   Caro Danilo,
   Fazendo z=a+bi, queremos provar que 
(e^a.cosb-1)^2+(e^a.senb)^2=(e^((a^2+b^2)^(1/2)-1)^2, o que equivale a
e^(2a)-2e^a.cosb=e^(2(a^2+b^2)^(1/2))-2e^((a^2+b^2)^(1/2)).
Vamos mostrar que 0=x=y implica e^(2y)-2e^y-(e^(2x)-2e^x=e^x(y^2-x^2).
Escrevendo y=x+h, isso equivale a e^(x+2h)-2e^h-e^x+2=h^2+2hx (apss dividir
por e^x). Isso pode ser escrito como e^x(e^h-1)(e^h+1)-2(e^h-1)=h(h+2x), ou
seja, (e^h-1)(e^(x+h)+e^x-2)=h(h+2x), mas e^h-1=h,e^x-1=x e 
e^(x+h)-1=x+h, donde e^(x+h)+e^x-2=2x+h e (e^h-1)(e^(x+h)+e^x-2)=h(h+2x).
Agora, usamos isso para y=(a^2+b^2)^(1/2) e x=a, obtendo
e^(2(a^2+b^2)^(1/2))-2e^((a^2+b^2)^(1/2))-(e^(2a)-2e^a)=e^a.b^2. Queremos
provar que o lado esquerdo e' =2e^a.(1-cosb), e logo (dividindo por e^a)
basta mostrar que b^2=2(1-cosb), mas 1-cosb=2(sen(b/2))^2=2.(b/2)^2=b^2/2,
donde b^2=2(1-cosb), cqd.
   Abragos,
 Gugu  


 
Pessoal , alguem sabe fazer essa ?
prove que para todo numero complexo z , vale
 |e^z-1| menor ou igual a  e^|z|-1

   Abs.

=
Instrugues para entrar na lista, sair da lista e usar a lista em
http://www.mat.puc-rio.br/~nicolau/olimp/obm-l.html
=


=
Instruções para entrar na lista, sair da lista e usar a lista em
http://www.mat.puc-rio.br/~nicolau/olimp/obm-l.html
=


=
Instruções para entrar na lista, sair da lista e usar a lista em
http://www.mat.puc-rio.br/~nicolau/olimp/obm-l.html
=


Re: RES: [obm-l] Segunda prova da IMO - Solucoes

2005-07-25 Por tôpico Carlos Gustavo Tamm de Araujo Moreira
   Caro Pedro,
   Está certa a sua solução (obviamente supondo que p é um primo ímpar
diferente de 3; os casos p=2 e p=3 são fáceis mas devem ser considerados a
parte), e é bem bonita. Concordo que esse problema não era difícil, mas num
certo sentido todo problema fica trivial depois de resolvido... 
   Abraços,
 Gugu


Gugu,

Acho que tenho uma solução mais simples para o 4.
Seja p um primo impar. Fazendo tudo modulo p temos,
2^(p-2)=a e 3^(p-2)=b onde 2a=1 e 3b=1.
Então 6(2^(p-2)+3^(p-2)+6^(p-2)-1)=3+2+1-6=0.
Isso mostra que para obtermos um múltiplo de p na seqüência, basta fazer
n=p-2. Estranha essa solução, pois aí o problema fica trivial.

Um abraço. Pedro.



-Mensagem original-
De: [EMAIL PROTECTED] [mailto:[EMAIL PROTECTED] Em nome
de Carlos Gustavo Tamm de Araujo Moreira
Enviada em: Sunday, July 24, 2005 12:13 AM
Para: obm-l@mat.puc-rio.br
Assunto: Re: [obm-l] Segunda prova da IMO - Solucoes

   Oi pessoal,
   Am vco minhas solugues do segundo dia, como mencionado na mensagem
anterior.
   Abragos,
Gugu


Oi gente, la vai o segundo dia da IMO.

Como a primeira prova de ontem, eu mesmo traduzi
agora.

Ainda nco pensei nos problemas de hoje, mas eles
parecem ser bem legais!

Os de ontem foram bem legais tambim. No comego, achei
que os problemas eram difmceis porque nem tinha muita
idiia de como resolver, mas depois que parei para
pensar com mais calma consegui resolver dois problemas
(1 e 2).

4. Determine todos os inteiros positivos relativamente
primos com todos os termos da seq|jncia infinita a_n =
2^n + 3^n + 6^n - 1, n = 1.

5. Seja ABCD um quadrilatero convexo e fixado com BC =
DA e BC nco paralelo a DA. Sejam E e F dois pontos
variaveis sobre BC e DA, respectivamente, tais que BE
= DF. As retas AC e BD cortam-se em P; as retas BD e
EF cortam-se em Q; as retas EF e AC cortam-se em R.

Quando variamos E e F, obtemos diferentes tribngulos
PQR. Prove que os circuncmrculos desses tribngulos tjm
um ponto comum diferente de P.

6. Numa competigco de matematica na qual foram
propostos 6 problemas, quaisquer dois problemas foram
resolvidos por mais de 2/5 dos estudantes. Alim disso,
nenhum estudante resolveu todos os 6 problemas. Mostre
que existem pelo menos 2 estudantes que resolveram 5
problemas cada um.

[]'s
Shine


  

Start your day with Yahoo! - make it your home page 
http://www.yahoo.com/r/hs 
 
=
Instrugues para entrar na lista, sair da lista e usar a lista em
http://www.mat.puc-rio.br/~nicolau/olimp/obm-l.html
=


   Solugues:

4)  Vamos achar os primos que dividem algum a_n, com n=1 (esses nco podem
dividir nenhum elemento desse conjunto de inteiros positivos em questco).
Temos que 2 divide a_1=2+3+6-1=10, e 3 divide a_2=2^2+3^2+6^2-1=48. Alim
disso, como (x-1)(x-2)(x-3)(x-6)=x^4-12x^3+47x^2-72x+36, a seq|jncia
satisfaz a recorrjncia a_(n+4)=12.a_(n+3)-47.a_(n+2)+72.a_(n+1)-36.a_n (isso
pode ser verificado diretamente, mas vejam tambim o artigo sobre
recorrjncias 
na Eureka 9). Se p i um primo diferente de 2 e 3, 36 e' inversmvel msdulo p,
e a recorrjncia i reversmvel (msdulo p):
a_n=(36)^(-1).(72.a_(n+1)-47.a_(n+2)+12.a_(n+3)-a_(n+4)) (mod p). Assim, a
seq|jncia a_n msdulo p i puramente perisdica (tem que ser perisdica a partir
de um certo ponto pois tjm que existir naturais distintos k e m com
a_(k+j)=a_(m+j) (mod p) para j=0,1,2,3 (pois (Z/pZ)^4 i finito), donde
a_(k+j)=a(m+j) (mod p) para todo j natural e, pela reversibilidade, para
todo j inteiro. Como a_(-1)=1/2+1/3+1/6-1=0, devemos entco ter a_n=0 (mod p)

para infinitos valores naturais de n. Assim, todo primo divide a_n para
algum inteiro positivo n, donde o znico inteiro positivo que satisfaz a
condigco do enunciado i o 1.

5)  Nesse eu fiz B=(0,0), C=(1,0), D=(c,d) e A=(c+cos(a),d+sen(a)). Se
E=(t,0), F=(c+t.cos(a),d+t.sen(a)), e am as equagues das retas AC, BD e EF
sco: AC: y=(d+sen(a))(x-1)/(c+cos(a)-1), BD: y=dx/c, 
EF: y=(d+t.sen(a))(x-t)/(c+t.(cos(a)-1)). Achamos entco P,Q e R: fazendo
w=c.sen(a)-d(cos(a)-1), temos P=(c(d+sen(a)),d(d+sen(a)))/w, Q=P+(1-t)u,
R=P+tv, onde u=-sen(a).(c,d)/w e v=sen(a).(c+cos(a)-1,d+sen(a))/w (nco vou
me preocupar com denominadores que eventualmente se anulem - esses casos
seguem por continuidade). Agora, via uma homotetia, podemos supor que
P=(0,0), Q=(1-t,0) e R=(tm,th). O circuncmrculo de PQR tem equagco C+tL=0,
onde C=x^2-x+y^2-my/h e L=x-(m^2+h^2+m)/h. Assim, todos eles passam pelos
dois pontos de intersegco de (C=0) e (L=0), que sco (0,0)=P e
s.(m^2+m+h^2,h), onde s=(m^2+h^2)/(h^2+(m^2+m+h^2)^2). E acabou, ni ?

6)  Nesse a gente faz assim: seja A_i, 1=i=6 o conjunto dos caras que
fizeram o problema i. Temos |A_i.A_j|2n/5 (vou representar intersegues por
produtos), qq i,j. Peguemos 3 pares, por exemplo A1A2, A3A4 e A5A6. A
intersegco dos 3 i vazia

Re: [obm-l] Uma de probabilidade...

2005-07-24 Por tôpico Carlos Gustavo Tamm de Araujo Moreira
   Caro Marcos,
   Temos (1+x)^2^m=1+x^2^m (mod 2). Assim, se k(1)k(2)...k(r),
(1+x)^(2^k(1)+2^k(2)+...+2^(k(r))=(1+x^2^k(1))(x^2^k(2))...(1+x^2^k(r)) 
(mod 2), e isso tem 2^r coeficientes iguais a 1 e os outros iguais a 0.
Assim, se m tem r bits não nulos, ha' 2^r valores de k com 0=k=m tais que
Binomial(m,k) é ímpar. Se m2^n, m é uma soma de potências de 2 com
expoentes entre 0 e n-1. Há Binomial(n,r) tais somas com r parcelas. Assim,
o número de pares (k,m) com 0=k=m2^n e Binomial(m,k) ímpar é dado por 
soma(r=0 a n)(Binomial(n,r).2^r)=3^n (para m=2^n haverá dois outros pares,
(0,2^n) e (2^n,2^n), mas isto não afetará a probabilidade), e o número total 
dos pares (k,m) com 0=k=m2^n é 1+2+...+2^n=2^(n-1)(2^n+1)(1/2).4^n. Como
lim 3^n/4^n=0, temos lim Pn=0.
   Abraços,
Gugu


   Olá pessoal da lista! Preciso da ajuda de vocês no seguinte
problema. Consegui resolvê-lo e queria conferir minha resposta.
   Dado n natural, escolhemos k em também naturais tais que
0=k=m=2^n. Seja Pn a probabilidade do coeficiente binomial ser par.
Encontre lim Pn.
   Obrigado!

=
Instruções para entrar na lista, sair da lista e usar a lista em
http://www.mat.puc-rio.br/~nicolau/olimp/obm-l.html
=


=
Instruções para entrar na lista, sair da lista e usar a lista em
http://www.mat.puc-rio.br/~nicolau/olimp/obm-l.html
=


Re: [obm-l] Uma de probabilidade...

2005-07-24 Por tôpico Carlos Gustavo Tamm de Araujo Moreira
Oi Marcos,
É isso mesmo! E essa probabilidade é, de fato, igual a
1-(3^n+2)/(2^(n-1).(2^n+1)+2^n+1), como segue das minhas contas.
Abraços,
 Gugu


   Olá Gugu! Muito obrigado pela atenção! Para esse problema achei o
seguinte valor para P(n)=[4^n-2*3^n+2^(n+1)+2^n-2]/(4^n+3*2^n+2), o
que me forneceria
lim Pn=1. Porém, eu calculei a probabilidade do coeficiente binomial
mCk ser par e creio que na sua solução você calculou a probabilidade
do mesmo ser ímpar. Correto? E assim, evidentemente, chegamos ao mesmo
resultado!

=
Instruções para entrar na lista, sair da lista e usar a lista em
http://www.mat.puc-rio.br/~nicolau/olimp/obm-l.html
=


=
Instruções para entrar na lista, sair da lista e usar a lista em
http://www.mat.puc-rio.br/~nicolau/olimp/obm-l.html
=


Re: [obm-l] Segunda prova da IMO - Problema 6

2005-07-23 Por tôpico Carlos Gustavo Tamm de Araujo Moreira
   Oi pessoal,
   Segue uma solução do problema 6, após a mensagem original do Shine.
   Abraços,
 Gugu


Oi gente, lá vai o segundo dia da IMO.

Como a primeira prova de ontem, eu mesmo traduzi
agora.

Ainda não pensei nos problemas de hoje, mas eles
parecem ser bem legais!

Os de ontem foram bem legais também. No começo, achei
que os problemas eram difíceis porque nem tinha muita
idéia de como resolver, mas depois que parei para
pensar com mais calma consegui resolver dois problemas
(1 e 2).

4. Determine todos os inteiros positivos relativamente
primos com todos os termos da seqüência infinita a_n =
2^n + 3^n + 6^n - 1, n = 1.

5. Seja ABCD um quadrilátero convexo e fixado com BC =
DA e BC não paralelo a DA. Sejam E e F dois pontos
variáveis sobre BC e DA, respectivamente, tais que BE
= DF. As retas AC e BD cortam-se em P; as retas BD e
EF cortam-se em Q; as retas EF e AC cortam-se em R.

Quando variamos E e F, obtemos diferentes triângulos
PQR. Prove que os circuncírculos desses triângulos têm
um ponto comum diferente de P.

6. Numa competição de matemática na qual foram
propostos 6 problemas, quaisquer dois problemas foram
resolvidos por mais de 2/5 dos estudantes. Além disso,
nenhum estudante resolveu todos os 6 problemas. Mostre
que existem pelo menos 2 estudantes que resolveram 5
problemas cada um.

[]'s
Shine


   

Start your day with Yahoo! - make it your home page 
http://www.yahoo.com/r/hs 
 
=
Instruções para entrar na lista, sair da lista e usar a lista em
http://www.mat.puc-rio.br/~nicolau/olimp/obm-l.html
=


=
Instruções para entrar na lista, sair da lista e usar a lista em
http://www.mat.puc-rio.br/~nicolau/olimp/obm-l.html
=


Re: [obm-l] Segunda prova da IMO - Problema 6 - agora vai...

2005-07-23 Por tôpico Carlos Gustavo Tamm de Araujo Moreira
   Mais uma vez esqueci de mandar o problema... Agora via o problema 6,
após a mensagem do Shine.
   Abraços,
Gugu


Oi gente, lá vai o segundo dia da IMO.

Como a primeira prova de ontem, eu mesmo traduzi
agora.

Ainda não pensei nos problemas de hoje, mas eles
parecem ser bem legais!

Os de ontem foram bem legais também. No começo, achei
que os problemas eram difíceis porque nem tinha muita
idéia de como resolver, mas depois que parei para
pensar com mais calma consegui resolver dois problemas
(1 e 2).

4. Determine todos os inteiros positivos relativamente
primos com todos os termos da seqüência infinita a_n =
2^n + 3^n + 6^n - 1, n = 1.

5. Seja ABCD um quadrilátero convexo e fixado com BC =
DA e BC não paralelo a DA. Sejam E e F dois pontos
variáveis sobre BC e DA, respectivamente, tais que BE
= DF. As retas AC e BD cortam-se em P; as retas BD e
EF cortam-se em Q; as retas EF e AC cortam-se em R.

Quando variamos E e F, obtemos diferentes triângulos
PQR. Prove que os circuncírculos desses triângulos têm
um ponto comum diferente de P.

6. Numa competição de matemática na qual foram
propostos 6 problemas, quaisquer dois problemas foram
resolvidos por mais de 2/5 dos estudantes. Além disso,
nenhum estudante resolveu todos os 6 problemas. Mostre
que existem pelo menos 2 estudantes que resolveram 5
problemas cada um.

[]'s
Shine


   

Start your day with Yahoo! - make it your home page 
http://www.yahoo.com/r/hs 
 
=
Instruções para entrar na lista, sair da lista e usar a lista em
http://www.mat.puc-rio.br/~nicolau/olimp/obm-l.html
=


   No 6 a gente faz assim: seja A_i, 1=i=6 o conjunto dos caras que
fizeram o problema i. Temos |A_i.A_j|2n/5 (vou representar interseções por
produtos), qq i,j. Peguemos 3 pares, por exemplo A1A2, A3A4 e A5A6. A
interseção dos 3 é vazia, por hipótese. A soma dos seus cardinais e' maior
que 3.(2n/5), donde a soma dos cardinais de A1A2A3A4, A1A2A5A6 e A3A4A5A6 é
maior que 3.(2n/5)-n=n/5. Assim, a média dos cardinais dos A_iA_jA_rA_s é
maior que n/15 (além disso, a média das diferenças dos cardinais dessas
interseções 4 a 4 e n/15 é igual à média das diferenças entre os cardinais
das interseções dos pares e 2n/5). Como há 15 dessas interseções 4 a 4, elas
não podem ser todas disjuntas, e logo tem que haver alguma interseção entre
elas, i.e, alguém resolveu 5 problemas. Se for exatamente um cara, este está
em 5 dessas interseções 4 a 4, donde a soma dos cardinais das interseções 4
a 4 é no máximo n+4. Se n não for 2 módulo 5, |A_iA_j|-2n/5 é pelo menos 2/5
para todo i,j, donde a média das interseções 4 a 4 é pelo menos n/15+2/5,
donde a soma de seus cardinais é pelo menos n+6, absurdo. Vamos então supor
que n=2(mod 5), e que só o cara x fez 5 problemas. A  soma dos cardinais das
interseções 4 a 4 é no mínimo 15(n/15+1/5)=n+3, donde é n+3 ou n+4. Se algum
cara resolveu no máximo 3 problemas, digamos 1,3 e 5, essa soma dá n+3, pois
ele não está na união das interseções 4 a 4, donde |A_iA_j|-2n/5=1/5 para
todo i,j e a união de A1A2, A3A4 e A5A6 dá tudo (senão a média dos cardinais
das interseções 4 a 4 aumenta), mas por outro lado, se tomamos os três pares
de conjuntos como acima, a união de A1A2, A3A4 e A5A6 não contém esse cara,
que fez no máximo os problemas 1,3 e 5, absurdo. Assim, a soma dos cardinais
das interseções 4 a 4 é n+4 e a união das interseções 4 a 4 dá tudo. Suponha
que {x}=A1A2A3A4A5.
Como existem 15 pares de conjuntos e 15 interseções 4 a 4, e a média dos
excessos |AiAj|-2n/5 e |AiAjArAs|-n/15 é a mesma, igual portanto à segunda,
que vale 4/15, segue que existe exatamente um par i0j0 com
|Ai0Aj0|=(2n+6)/5, e todos os outros |AiAj| são (2n+1)/5. Seja {r,s} um par
contendo 6 e que intersecte mas não seja igual a {i0,j0}. Suponhamos que
{1,2,3,4,5,6}={r,s,a,b,c,d}. Temos que ArAs é a união disjunta de ArAsAaAb,
ArAsAcAd e AaAbAcAd. Temos
|AaAbAcAd|+|ArAsAaAb|+|ArAsAcAd|=|AaAb|+|AcAd|+|ArAs|-n=3(2n+1)/5-n=(n+3)/5
|AaAbAcAd|+|ArAsAaAc|+|ArAsAbAd|=|AaAc|+|AbAd|+|ArAs|-n=3(2n+1)/5-n=(n+3)/5
|AaAbAcAd|+|ArAsAaAd|+|ArAsAbAc|=|AaAd|+|AbAc|+|ArAs|-n=3(2n+1)/5-n=(n+3)/5,
donde, como a soma dos |ArAsAiAj| sobre os i,j é |ArAs|=(2n+1)/5, temos
3|AaAbAcAd|+(2n+1)/5=3(n+3)/5, e logo |AaAbAcAd|=(n+8)/15.  Em particular,
n=7(mod 15). Segue da primeira dessas 3 igualdades que |ArAsAaAb| ou
|ArAsAcAd| é no máximo (n-7)/15 (senão os dois seriam no mínimo (n+8)/15,
absurdo). Digamos que |ArAsAaAb|=(n-7)/15. Considerando igualdades como as
três acima em que trocamos {r,s} por {c,d}, segue que os AcAdAiAj têm
=(n+8)/15 elementos em média, donde, como no máximo 3 deles se intersectam,
sua união AcAd tem pelo menos 2(n+8)/5-2=(2n+6)/5, e logo {c,d}={i0,j0},
absurdo pois {i0,j0} intersecta {r,s}, que não intersecta {c,d}.

Re: [obm-l] Segunda prova da IMO - Problema 4

2005-07-23 Por tôpico Carlos Gustavo Tamm de Araujo Moreira
   Oi pessoal,
   Segue solução do problema 4 após a mensagem do Shine. Creio que isso
completa as minhas soluções da IMO. Comentários, dúvidas, críticas, etc 
serão muito bem vindos.
   Abraços,
Gugu


Oi gente, lá vai o segundo dia da IMO.

Como a primeira prova de ontem, eu mesmo traduzi
agora.

Ainda não pensei nos problemas de hoje, mas eles
parecem ser bem legais!

Os de ontem foram bem legais também. No começo, achei
que os problemas eram difíceis porque nem tinha muita
idéia de como resolver, mas depois que parei para
pensar com mais calma consegui resolver dois problemas
(1 e 2).

4. Determine todos os inteiros positivos relativamente
primos com todos os termos da seqüência infinita a_n =
2^n + 3^n + 6^n - 1, n = 1.

5. Seja ABCD um quadrilátero convexo e fixado com BC =
DA e BC não paralelo a DA. Sejam E e F dois pontos
variáveis sobre BC e DA, respectivamente, tais que BE
= DF. As retas AC e BD cortam-se em P; as retas BD e
EF cortam-se em Q; as retas EF e AC cortam-se em R.

Quando variamos E e F, obtemos diferentes triângulos
PQR. Prove que os circuncírculos desses triângulos têm
um ponto comum diferente de P.

6. Numa competição de matemática na qual foram
propostos 6 problemas, quaisquer dois problemas foram
resolvidos por mais de 2/5 dos estudantes. Além disso,
nenhum estudante resolveu todos os 6 problemas. Mostre
que existem pelo menos 2 estudantes que resolveram 5
problemas cada um.

[]'s
Shine


   

Start your day with Yahoo! - make it your home page 
http://www.yahoo.com/r/hs 
 
=
Instruções para entrar na lista, sair da lista e usar a lista em
http://www.mat.puc-rio.br/~nicolau/olimp/obm-l.html
=


   Vamos achar os primos que dividem algum a_n, com n=1 (esses não podem
dividir nenhum elemento desse conjunto de inteiros positivos em questão).
Temos que 2 divide a_1=2+3+6-1=10, e 3 divide a_2=2^2+3^2+6^2-1=48. Além
disso, como (x-1)(x-2)(x-3)(x-6)=x^4-12x^3+47x^2-72x+36, a seqüência
satisfaz a recorrência a_(n+4)=12.a_(n+3)-47.a_(n+2)+72.a_(n+1)-36.a_n (isso
pode ser verificado diretamente, mas vejam também o artigo sobre recorrências 
na Eureka 9). Se p é um primo diferente de 2 e 3, 36 e' inversível módulo p,
e a recorrência é reversível:
a_n=(36)^(-1).(72.a_(n+1)-47.a_(n+2)+12.a_(n+3)-a_(n+4)) (mod p). Assim, a
seqüênciaa_n módulo p é puramente periódica (tem que ser periódica a partir
de um certo ponto pois têm que existir naturais distintos k e m com
a_(k+j)=a_(m+j) (mod p) para j=0,1,2,3 (pois (Z/pZ)^4 é finito), donde
a_(k+j)=a(m+j) (mod p) para todo j natural e, pela reversibilidade, para
todo j inteiro. Como a_(-1)=1/2+1/3+1/6-1=0, devemos então ter a_n=0 (mod p) 
para infinitos valores naturais de n. Assim, todo primo divide a_n para
algum inteiro positivo n, donde o único inteiro positivo que satisfaz a
condição do enunciado é o 1.

=
Instruções para entrar na lista, sair da lista e usar a lista em
http://www.mat.puc-rio.br/~nicolau/olimp/obm-l.html
=


Re: [obm-l] Prova da IMO - Primeiro dia - Solucoes

2005-07-23 Por tôpico Carlos Gustavo Tamm de Araujo Moreira
   Oi pessoal,
   Resolvi compilar as minhas soluções de cada um dos dias para fins de
referência (em particular porque algumas de minhas mensagens anteriores
foram um pouco confusas, ou por não ter a solução junto ou por não dizerem
no subject sobre que problema tratavam). Seguem aqui (como sempre, após a
mensagem original do Shine) as soluções do primeiro dia.
   Abraços,
Gugu

P.S.: Seria ótimo se outras pessoas mandassem também as suas soluções -
vários problemas admitem mais de uma solução interessante...


Oi gente,

Acabei de ver a primeira prova da IMO no site
http://www.mathlinks.ro/

Lá vão os enunciados (eu mesmo traduzi agora).

1. Escolhemos seis pontos sobre os lados do triângulo
equilátero ABC: A_1, A_2 sobre BC; B_1, B_2 sobre AC;
C_1, C_2 sobre AB. Essa escolha é feita de modo que
A_1A_2B_1B_2C_1C_2 é um hexágono convexo com todos os
seus lados iguais.

Prove que A_1B_2, B_1C_2 e C_1A_2 são concorrentes.

2. Seja a_1,a_2,... uma seqüência de inteiros com
infinitos termos positivos e negativos. Suponha que
para todo n inteiro positivo os números
a_1,a_2,...,a_n deixam n restos diferentes na divisão
por n.

Prove que todo inteiro aparece exatamente uma vez na
seqüência a_1,a_2,...

3. Sejam x,y,z reais positivos tais que xyz = 1.
Prove que
(x^5-x^2)/(x^5+y^2+z^2) + 
(y^5-y^2)/(x^2+y^5+z^2) + 
(z^5-z^2)/(x^2+y^2+z^5) = 0.

[]'s
Shine


   

Start your day with Yahoo! - make it your home page
http://www.yahoo.com/r/hs
 
=
Instruções para entrar na lista, sair da lista e usar a lista em
http://www.mat.puc-rio.br/~nicolau/olimp/obm-l.html
=

Soluções:

1) Se o lado do triângulo e' 1 e denotamos um segmento em cada lado por a, b
e c, obtemos umas identidades como segue (pela lei dos cossenos): 
a^2+(1-x-b)^2-a(1-x-b)=x^2
b^2+(1-x-c)^2-b(1-x-c)=x^2
c^2+(1-x-a)^2-c(1-x-a)=x^2
Subtraindo a primeira da segunda temos a^2-c^2+(2c-b-a)(1-x)+b(a-c)=0, e,
subtraindo a terceira da segunda, temos b^2-a^2+(2a-c-b)(1-x)+c(b-a)=0, que
podem ser escritas como (a-c)(a+b+c)=(1-x)(a+b-2c) e
(b-a)(a+b+c)=(1-x)(b+c-2a). Multiplicando a primeira por b+c-2a, a segunda
por a+b-2c e igualando, temos, depois de cortar o a+b+c,
(a-c)(b+c-2a)=(b-a)(a+b-2c), donde a(2b-a-c)=b^2+ab-bc+c^2-2ac, ou seja,
ab+ac+bc=a^2+b^2+c^2, donde a=b=c, e a primeira equação vira algo como
a^2+(1-a-x)^2-a(1-a-x)=x^2, donde a^2+(1-a)^2-a(1-a)-2x(1-a)+ax=0, ou seja,
x(2-3a)=3a^2-3a+1, e 1-a-x=(1-2a)/(2-3a). Basta ver então que a reta que
passa por ((1-2a)/(2-3a),0) e (1/2+a/2,(1-a)sqrt(3)/2) passa por
(1/2,sqrt(3)/6), o centro do triângulo, e acabou.

2) Essa eu fiz assim: se 1=ij então |a_i-a_j|j, senão, fazendo
n=|a_i-a_j|, temos 1=ij=n mas a_i e a_j deixam o mesmo resto na divisão
por n. Assim, para todo n=1, {a_1,a_2,...,a_n} tem que ser um intervalo,
isto é, um conjunto de n inteiros consecutivos (com efeito, pelo fato acima, 
a diferença entre o menor e o maior desses números é menor que n, e eles são
todos distintos). Como uma união crescente de intervalos de inteiros que é 
ilimitada dos dois lados tem que o conjunto de todos os inteiros, acabou.

3) Aqui, chamei x^2+y^2+z^2 de A (que e' pelo menos 3, por MA-MG, pois
xyz=1) e ai' o negócio fica r/(A+r)+s/(A+s)+t/(A+t), onde r, s e t são 
x^5-x^2, y^5-y^2 e z^5-z^2, respectivamente. Aí eu troquei os w^5-w^2 por
w^3-1, para w=x,y,z (se der certo a desigualdade com essa troca então vale a 
original, pois, como w0, w^5-w^2 e' maior que w^3-1). Assim, basta provar
que (x^3-1)/(A+x^3-1)+(y^3-1)/(A+y^3-1)+(z^3-1)/(A+z^3-1)=0, o que equivale
a 3-A(1/(A+x^3-1)+1/(A+y^3-1)+1/(A+z^3-1))=0, e, fazendo c=A-1=2, m=x^3,
n=y^3, p=z^3, isso equivale a mostrar que 1/(m+c)+1/(n+c)+1/(p+c)=3/(1+c),
sabendo que m,n,p0, mnp=1 e c=2. Isso pode ser mostrado assim: podemos
supor mnp=1 (se aumentamos um deles, o lado esquerdo diminui, e o direito
não muda). Supondo m=n=p, como mnp=1, segue que mn=1, e portanto, fazendo
u=m^(1/2) e v=n^(1/2), temos 1/(u^2+c)+1/(v^2+c)-2/(uv+c)=
=(uv+c)(u^2+v^2+2c)-2(u^2+c)(v^2+c)=(uv-c)(u^2+v^2-2uv)=0, i.e, trocando m
e n por (mn)^(1/2), o lado direito não diminui, e logo podemos supor m=n, e 
p=1/m^2. A desigualdade então fica 2/(m+c)+m^2/(1+cm^2)=3/(1+c), o que
equivale a (1+c)(2(1+cm^2)+m^2.(m+c))-3(m+c)(cm^2+1)=0. O lado esquerdo é
(1-2c)m^3+3cm^2-3m+(2-c)=(m-1)^2.((1-2c)m+2-c)=0, pois c=2 e m=0, cqd.  
=
Instruções para entrar na lista, sair da lista e usar a lista em
http://www.mat.puc-rio.br/~nicolau/olimp/obm-l.html
=


Re: [obm-l] Segunda prova da IMO - Solucoes

2005-07-23 Por tôpico Carlos Gustavo Tamm de Araujo Moreira
   Oi pessoal,
   Aí vão minhas soluções do segundo dia, como mencionado na mensagem
anterior.
   Abraços,
Gugu


Oi gente, lá vai o segundo dia da IMO.

Como a primeira prova de ontem, eu mesmo traduzi
agora.

Ainda não pensei nos problemas de hoje, mas eles
parecem ser bem legais!

Os de ontem foram bem legais também. No começo, achei
que os problemas eram difíceis porque nem tinha muita
idéia de como resolver, mas depois que parei para
pensar com mais calma consegui resolver dois problemas
(1 e 2).

4. Determine todos os inteiros positivos relativamente
primos com todos os termos da seqüência infinita a_n =
2^n + 3^n + 6^n - 1, n = 1.

5. Seja ABCD um quadrilátero convexo e fixado com BC =
DA e BC não paralelo a DA. Sejam E e F dois pontos
variáveis sobre BC e DA, respectivamente, tais que BE
= DF. As retas AC e BD cortam-se em P; as retas BD e
EF cortam-se em Q; as retas EF e AC cortam-se em R.

Quando variamos E e F, obtemos diferentes triângulos
PQR. Prove que os circuncírculos desses triângulos têm
um ponto comum diferente de P.

6. Numa competição de matemática na qual foram
propostos 6 problemas, quaisquer dois problemas foram
resolvidos por mais de 2/5 dos estudantes. Além disso,
nenhum estudante resolveu todos os 6 problemas. Mostre
que existem pelo menos 2 estudantes que resolveram 5
problemas cada um.

[]'s
Shine


   

Start your day with Yahoo! - make it your home page 
http://www.yahoo.com/r/hs 
 
=
Instruções para entrar na lista, sair da lista e usar a lista em
http://www.mat.puc-rio.br/~nicolau/olimp/obm-l.html
=


   Soluções:

4)  Vamos achar os primos que dividem algum a_n, com n=1 (esses não podem
dividir nenhum elemento desse conjunto de inteiros positivos em questão).
Temos que 2 divide a_1=2+3+6-1=10, e 3 divide a_2=2^2+3^2+6^2-1=48. Além
disso, como (x-1)(x-2)(x-3)(x-6)=x^4-12x^3+47x^2-72x+36, a seqüência
satisfaz a recorrência a_(n+4)=12.a_(n+3)-47.a_(n+2)+72.a_(n+1)-36.a_n (isso
pode ser verificado diretamente, mas vejam também o artigo sobre recorrências 
na Eureka 9). Se p é um primo diferente de 2 e 3, 36 e' inversível módulo p,
e a recorrência é reversível (módulo p):
a_n=(36)^(-1).(72.a_(n+1)-47.a_(n+2)+12.a_(n+3)-a_(n+4)) (mod p). Assim, a
seqüência a_n módulo p é puramente periódica (tem que ser periódica a partir
de um certo ponto pois têm que existir naturais distintos k e m com
a_(k+j)=a_(m+j) (mod p) para j=0,1,2,3 (pois (Z/pZ)^4 é finito), donde
a_(k+j)=a(m+j) (mod p) para todo j natural e, pela reversibilidade, para
todo j inteiro. Como a_(-1)=1/2+1/3+1/6-1=0, devemos então ter a_n=0 (mod p) 
para infinitos valores naturais de n. Assim, todo primo divide a_n para
algum inteiro positivo n, donde o único inteiro positivo que satisfaz a
condição do enunciado é o 1.

5)  Nesse eu fiz B=(0,0), C=(1,0), D=(c,d) e A=(c+cos(a),d+sen(a)). Se
E=(t,0), F=(c+t.cos(a),d+t.sen(a)), e aí as equações das retas AC, BD e EF
são: AC: y=(d+sen(a))(x-1)/(c+cos(a)-1), BD: y=dx/c, 
EF: y=(d+t.sen(a))(x-t)/(c+t.(cos(a)-1)). Achamos então P,Q e R: fazendo
w=c.sen(a)-d(cos(a)-1), temos P=(c(d+sen(a)),d(d+sen(a)))/w, Q=P+(1-t)u,
R=P+tv, onde u=-sen(a).(c,d)/w e v=sen(a).(c+cos(a)-1,d+sen(a))/w (não vou
me preocupar com denominadores que eventualmente se anulem - esses casos
seguem por continuidade). Agora, via uma homotetia, podemos supor que
P=(0,0), Q=(1-t,0) e R=(tm,th). O circuncírculo de PQR tem equação C+tL=0,
onde C=x^2-x+y^2-my/h e L=x-(m^2+h^2+m)/h. Assim, todos eles passam pelos
dois pontos de interseção de (C=0) e (L=0), que são (0,0)=P e
s.(m^2+m+h^2,h), onde s=(m^2+h^2)/(h^2+(m^2+m+h^2)^2). E acabou, né ?

6)  Nesse a gente faz assim: seja A_i, 1=i=6 o conjunto dos caras que
fizeram o problema i. Temos |A_i.A_j|2n/5 (vou representar interseções por
produtos), qq i,j. Peguemos 3 pares, por exemplo A1A2, A3A4 e A5A6. A
interseção dos 3 é vazia, por hipótese. A soma dos seus cardinais e' maior
que 3.(2n/5), donde a soma dos cardinais de A1A2A3A4, A1A2A5A6 e A3A4A5A6 é
maior que 3.(2n/5)-n=n/5. Assim, a média dos cardinais dos A_iA_jA_rA_s é
maior que n/15 (além disso, a média das diferenças dos cardinais dessas
interseções 4 a 4 e n/15 é igual à média das diferenças entre os cardinais
das interseções dos pares e 2n/5). Como há 15 dessas interseções 4 a 4, elas
não podem ser todas disjuntas, e logo tem que haver alguma interseção entre
elas, i.e, alguém resolveu 5 problemas. Se for exatamente um cara, este está
em 5 dessas interseções 4 a 4, donde a soma dos cardinais das interseções 4
a 4 é no máximo n+4. Se n não for 2 módulo 5, |A_iA_j|-2n/5 é pelo menos 2/5
para todo i,j, donde a média das interseções 4 a 4 é pelo menos n/15+2/5,
donde a soma de seus cardinais é pelo menos n+6, absurdo. Vamos então supor
que n=2(mod 5), e que só o cara x fez 5 problemas. A  soma 

Re: [obm-l] Prova da IMO - Primeiro dia

2005-07-22 Por tôpico Carlos Gustavo Tamm de Araujo Moreira
   Oi pessoal,
   Segue uma solução do Problema 3 (após a mensagem original).
   Abraços,
 Gugu

Oi gente,

Acabei de ver a primeira prova da IMO no site
http://www.mathlinks.ro/

Lá vão os enunciados (eu mesmo traduzi agora).

1. Escolhemos seis pontos sobre os lados do triângulo
equilátero ABC: A_1, A_2 sobre BC; B_1, B_2 sobre AC;
C_1, C_2 sobre AB. Essa escolha é feita de modo que
A_1A_2B_1B_2C_1C_2 é um hexágono convexo com todos os
seus lados iguais.

Prove que A_1B_2, B_1C_2 e C_1A_2 são concorrentes.

2. Seja a_1,a_2,... uma seqüência de inteiros com
infinitos termos positivos e negativos. Suponha que
para todo n inteiro positivo os números
a_1,a_2,...,a_n deixam n restos diferentes na divisão
por n.

Prove que todo inteiro aparece exatamente uma vez na
seqüência a_1,a_2,...

3. Sejam x,y,z reais positivos tais que xyz = 1.
Prove que
(x^5-x^2)/(x^5+y^2+z^2) + 
(y^5-y^2)/(x^2+y^5+z^2) + 
(z^5-z^2)/(x^2+y^2+z^5) = 0.

[]'s
Shine


   

Start your day with Yahoo! - make it your home page
http://www.yahoo.com/r/hs
 
=
Instruções para entrar na lista, sair da lista e usar a lista em
http://www.mat.puc-rio.br/~nicolau/olimp/obm-l.html
=


No 3, chamei x^2+y^2+z^2 de A (que e' pelo menos 3, por MA-MG, pois xyz=1)
e ai' o negócio fica r/(A+r)+s/(A+s)+t/(A+t), onde r, s e t são x^5-x^2,
y^5-y^2 e z^5-z^2, respectivamente. Aí eu troquei os w^5-w^2 por w^3-1, para 
w=x,y,z (se der certo a desigualdae com essa troca então vale a original,
pois, como w0, w^5-w^2 e' maior que w^3-1). Assim, basta provar que
(x^3-1)/(A+x^3-1)+(y^3-1)/(A+y^3-1)+(z^3-1)/(A+z^3-1)=0, o que equivale a
3-A(1/(A+x^3-1)+1/(A+y^3-1)+1/(A+z^3-1))=0, e, fazendo c=A-1=2, m=x^3,
n=y^3, p=z^3, isso equivale a mostrar que 1/(m+c)+1/(n+c)+1/(p+c)=3/(1+c),
sabendo que m,n,p0, mnp=1 e c=2. Isso pode ser mostrado assim: podemos
supor mnp=1 (se aumentamos um deles, o lado esquerdo diminui, e o direito
não muda). Supondo m=n=p, como mnp=1, segue que mn=1, e portanto, fazendo
u=m^(1/2) e v=n^(1/2), temos 1/(u^2+c)+1/(v^2+c)-2/(uv+c)=
=(uv+c)(u^2+v^2+2c)-2(u^2+c)(v^2+c)=(uv-c)(u^2+v^2-2uv)=0, i.e, trocando m
e n por (mn)^(1/2), o lado direito não diminui, e logo podemos supor m=n, e 
p=1/m^2. A desigualdade então fica 2/(m+c)+m^2/(1+cm^2)=3/(1+c), o que
equivale a (1+c)(2(1+cm^2)+m^2.(m+c))-3(m+c)(cm^2+1)=0. O lado esquerdo é
(1-2c)m^3+3cm^2-3m+(2-c)=(m-1)^2.((1-2c)m+2-c)=0, pois c=2 e m=0, cqd. 
=
Instruções para entrar na lista, sair da lista e usar a lista em
http://www.mat.puc-rio.br/~nicolau/olimp/obm-l.html
=


Re: [obm-l] Prova da IMO - Primeiro dia

2005-07-22 Por tôpico Carlos Gustavo Tamm de Araujo Moreira
   Oi Domingos et al,
   Essa eu fiz assim: se 1=ij então |a_i-a_j|j, senão, fazendo
n=|a_i-a_j|, temos 1=ij=n mas a_i e a_j deixam o mesmo resto na divisão
por n. Assim, para todo n=1, {a_1,a_2,...,a_n} tem que ser um intervalo,
isto é, um conjunto de n inteiros consecutivos (com efeito, pelo fato acima, 
a diferença entre o menor e o maior desses números é menor que n, e eles são
todos distintos). Como uma união crescente de intervalos de inteiros que é 
ilimitada dos dois lados tem que o conjunto de todos os inteiros, acabou.



2. Seja a_1,a_2,... uma seqüência de inteiros com
infinitos termos positivos e negativos. Suponha que
para todo n inteiro positivo os números
a_1,a_2,...,a_n deixam n restos diferentes na divisão
por n.

Prove que todo inteiro aparece exatamente uma vez na
seqüência a_1,a_2,...

  

Achei este legal, note que o enunciado é levemente ambíguo, pois devemos 
dizer que há infinitos termos positivos e infinitos termos negativos, 
caso contrário, a sequüência dos números naturais positivos ordenados 
claramente satisfaz a propriedade e não contém nenhum negativo.

--- x ---

Por conveniência, chame de P a propriedade do enunciado
Seja S = {s_1, s_2, ...} uma seqüência que satisfaz P. Sem perda de 
generalidade, podemos supor que 0 está em S. Basta observar que S + a = 
{s_1 + a, s_2 + a, ...} também satisfaz P.

Vamos mostrar que, para todo n = 0, existe N tal que todos os valores 
{-n, 1-n, ..., -1, 0, 1, ..., n} aparecem em {s_1, ..., s_N}.
Por hipótese, o caso n = 0 está ok. Vamos provar que se vale para n-1 
então vale para n.
Existe N' tal que {1-n, ..., 0, ..., n-1} aparecem em {s_1, ..., s_N'}. 
Tome N  N'  2n e considere o único s_i em {s_1, ..., s_N} tal que s_i 
~ n (mod N), se s_i = n, paramos por aqui. Senão
- caso s_i = A + n com N|A e A = N temos, claramente que {s_1, ..., 
s_{A+n}} contém dois valores que são 0 mod A + n, a saber, 0 e s_i = A + 
n, o que não pode ocorrer.
- caso s_i = n - A, com N|A e A  N, então |s_i| = A - n = N e 
novamente {s_1, ..., s_{A-n}} contém dois 0 mod (A-n).
- caso s_i = n - N: neste caso não temos como fazer a mesma 
asserção, mas note que N pode ser qualquer valor  N' e se cairmos 
sempre neste caso para qualquer escolha de N vamos exibir uma 
contradição* em S.
Para s_i ~ -n (mod N) o argumento é análogo ao acima.

* Suponha que para todo N  N' tenhamos n - N pertence a {s_1, ..., 
s_N}, um simples argumento de contagem mostra que há infinitos números 
negativos em S, mas não podemos ter infinitos números *positivos* em S.

Abraços.




=
Instruções para entrar na lista, sair da lista e usar a lista em
http://www.mat.puc-rio.br/~nicolau/olimp/obm-l.html
=


=
Instruções para entrar na lista, sair da lista e usar a lista em
http://www.mat.puc-rio.br/~nicolau/olimp/obm-l.html
=


Re: [obm-l] Segunda prova da IMO - Problema 5

2005-07-21 Por tôpico Carlos Gustavo Tamm de Araujo Moreira
Oi pessoal,
Segue uma solução (por analítica, para manter a tradição) do problema 5
da IMO, após a mensagem original do Shine.
Abraços,
  Gugu


Oi gente, lá vai o segundo dia da IMO.

Como a primeira prova de ontem, eu mesmo traduzi
agora.

Ainda não pensei nos problemas de hoje, mas eles
parecem ser bem legais!

Os de ontem foram bem legais também. No começo, achei
que os problemas eram difíceis porque nem tinha muita
idéia de como resolver, mas depois que parei para
pensar com mais calma consegui resolver dois problemas
(1 e 2).

4. Determine todos os inteiros positivos relativamente
primos com todos os termos da seqüência infinita a_n =
2^n + 3^n + 6^n - 1, n = 1.

5. Seja ABCD um quadrilátero convexo e fixado com BC =
DA e BC não paralelo a DA. Sejam E e F dois pontos
variáveis sobre BC e DA, respectivamente, tais que BE
= DF. As retas AC e BD cortam-se em P; as retas BD e
EF cortam-se em Q; as retas EF e AC cortam-se em R.

Quando variamos E e F, obtemos diferentes triângulos
PQR. Prove que os circuncírculos desses triângulos têm
um ponto comum diferente de P.

6. Numa competição de matemática na qual foram
propostos 6 problemas, quaisquer dois problemas foram
resolvidos por mais de 2/5 dos estudantes. Além disso,
nenhum estudante resolveu todos os 6 problemas. Mostre
que existem pelo menos 2 estudantes que resolveram 5
problemas cada um.

[]'s
Shine


   

Start your day with Yahoo! - make it your home page 
http://www.yahoo.com/r/hs 
 
=
Instruções para entrar na lista, sair da lista e usar a lista em
http://www.mat.puc-rio.br/~nicolau/olimp/obm-l.html
=


=
Instruções para entrar na lista, sair da lista e usar a lista em
http://www.mat.puc-rio.br/~nicolau/olimp/obm-l.html
=


Re: [obm-l] Segunda prova da IMO - Problema 5 (agora vai...)

2005-07-21 Por tôpico Carlos Gustavo Tamm de Araujo Moreira
   Acho que mandei a mensagem anterior sem a solução. Agora la está lá...
   Abraços,
Gugu


Oi gente, lá vai o segundo dia da IMO.

Como a primeira prova de ontem, eu mesmo traduzi
agora.

Ainda não pensei nos problemas de hoje, mas eles
parecem ser bem legais!

Os de ontem foram bem legais também. No começo, achei
que os problemas eram difíceis porque nem tinha muita
idéia de como resolver, mas depois que parei para
pensar com mais calma consegui resolver dois problemas
(1 e 2).

4. Determine todos os inteiros positivos relativamente
primos com todos os termos da seqüência infinita a_n =
2^n + 3^n + 6^n - 1, n = 1.

5. Seja ABCD um quadrilátero convexo e fixado com BC =
DA e BC não paralelo a DA. Sejam E e F dois pontos
variáveis sobre BC e DA, respectivamente, tais que BE
= DF. As retas AC e BD cortam-se em P; as retas BD e
EF cortam-se em Q; as retas EF e AC cortam-se em R.

Quando variamos E e F, obtemos diferentes triângulos
PQR. Prove que os circuncírculos desses triângulos têm
um ponto comum diferente de P.

6. Numa competição de matemática na qual foram
propostos 6 problemas, quaisquer dois problemas foram
resolvidos por mais de 2/5 dos estudantes. Além disso,
nenhum estudante resolveu todos os 6 problemas. Mostre
que existem pelo menos 2 estudantes que resolveram 5
problemas cada um.

[]'s
Shine


   

Start your day with Yahoo! - make it your home page 
http://www.yahoo.com/r/hs 
 
=
Instruções para entrar na lista, sair da lista e usar a lista em
http://www.mat.puc-rio.br/~nicolau/olimp/obm-l.html
=


   No 5 eu fiz B=(0,0), C=(1,0), D=(c,d) e A=(c+cos(a),d+sen(a)). Se
E=(t,0), F=(c+t.cos(a),d+t.sen(a)), e aí as equações das retas AC, BD e EF
são: AC: y=(d+sen(a))(x-1)/(c+cos(a)-1), BD: y=dx/c, 
EF: y=(d+t.sen(a))(x-t)/(c+t.(cos(a)-1)). Achamos então P,Q e R: fazendo
w=c.sen(a)-d(cos(a)-1), temos P=(c(d+sen(a)),d(d+sen(a)))/w, Q=P+(1-t)u,
R=P+tv, onde u=-sen(a).(c,d)/w e v=sen(a).(c+cos(a)-1,d+sen(a))/w (não vou
me preocupar com denominadores que eventualmente se anulem - esses casos
seguem por continuidade). Agora, via uma homotetia, podemos supor que
P=(0,0), Q=(1-t,0) e R=(tm,th). O circuncírculo de PQR tem equação C+tL=0,
onde C=x^2-x+y^2-my/h e L=x-(m^2+h^2+m)/h. Assim, todos eles passam pelos
dois pontos de interseção de (C=0) e (L=0), que são (0,0)=P e
s.(m^2+m+h^2,h), onde s=(m^2+h^2)/(h^2+(m^2+m+h^2)^2). E acabou, né ?
=
Instruções para entrar na lista, sair da lista e usar a lista em
http://www.mat.puc-rio.br/~nicolau/olimp/obm-l.html
=


Re: [obm-l] Prova da IMO - Primeiro dia

2005-07-20 Por tôpico Carlos Gustavo Tamm de Araujo Moreira
   Oi pessoal,
   Vou mandar a minha solução (com contas, naturalmente) do problema 1,
após a cópia da mensagem original do Shine, para ninguém que queira pensar
antes no problema ler involuntariamente a solução. Depois, se não houver
objeções, eu mando as minhas soluções dos problemas que faltam.
   Abraços,
 Gugu


Oi gente,

Acabei de ver a primeira prova da IMO no site
http://www.mathlinks.ro/

Lá vão os enunciados (eu mesmo traduzi agora).

1. Escolhemos seis pontos sobre os lados do triângulo
equilátero ABC: A_1, A_2 sobre BC; B_1, B_2 sobre AC;
C_1, C_2 sobre AB. Essa escolha é feita de modo que
A_1A_2B_1B_2C_1C_2 é um hexágono convexo com todos os
seus lados iguais.

Prove que A_1B_2, B_1C_2 e C_1A_2 são concorrentes.

2. Seja a_1,a_2,... uma seqüência de inteiros com
infinitos termos positivos e negativos. Suponha que
para todo n inteiro positivo os números
a_1,a_2,...,a_n deixam n restos diferentes na divisão
por n.

Prove que todo inteiro aparece exatamente uma vez na
seqüência a_1,a_2,...

3. Sejam x,y,z reais positivos tais que xyz = 1.
Prove que
(x^5-x^2)/(x^5+y^2+z^2) + 
(y^5-y^2)/(x^2+y^5+z^2) + 
(z^5-z^2)/(x^2+y^2+z^5) = 0.

[]'s
Shine


   

Start your day with Yahoo! - make it your home page
http://www.yahoo.com/r/hs
 
=
Instruções para entrar na lista, sair da lista e usar a lista em
http://www.mat.puc-rio.br/~nicolau/olimp/obm-l.html
=


Se o lado do triângulo e' 1 e denotamos um segmento em cada lado por a, b e
c, obtemos umas identidades como segue (pela lei dos cossenos): 
a^2+(1-x-b)^2-a(1-x-b)=x^2
b^2+(1-x-c)^2-b(1-x-c)=x^2
c^2+(1-x-a)^2-c(1-x-a)=x^2
Subtraindo a primeira da segunda temos a^2-c^2+(2c-b-a)(1-x)+b(a-c)=0, e,
subtraindo a terceira da segunda, temos b^2-a^2+(2a-c-b)(1-x)+c(b-a)=0, que
podem ser escritas como (a-c)(a+b+c)=(1-x)(a+b-2c) e
(b-a)(a+b+c)=(1-x)(b+c-2a). Multiplicando a primeira por b+c-2a, a segunda
por a+b-2c e igualando, temos, depois de cortar o a+b+c,
(a-c)(b+c-2a)=(b-a)(a+b-2c), donde a(2b-a-c)=b^2+ab-bc+c^2-2ac, ou seja,
ab+ac+bc=a^2+b^2+c^2, donde a=b=c, e a primeira equação vira algo como
a^2+(1-a-x)^2-a(1-a-x)=x^2, donde a^2+(1-a)^2-a(1-a)-2x(1-a)+ax=0, ou seja,
x(2-3a)=3a^2-3a+1, e 1-a-x=(1-2a)/(2-3a). Basta ver então que a reta que
passa por ((1-2a)/(2-3a),0) e (1/2+a/2,(1-a)sqrt(3)/2) passa por
(1/2,sqrt(3)/6), o centro do triângulo, e acabou. 
=
Instruções para entrar na lista, sair da lista e usar a lista em
http://www.mat.puc-rio.br/~nicolau/olimp/obm-l.html
=


Re: [obm-l] Matriz - IMC

2005-06-15 Por tôpico Carlos Gustavo Tamm de Araujo Moreira
   Mais precisamente, a solução deste problema esta' em
http://www.ucl.ac.uk/~ucahjej/imc/imc1997/prob_sol1.pdf
Ele é o problema 3 da IMC de 1997. A solução é curta, mas depende de uma
idéia que eu não tive quando pensei nele, recentemente, e acabei não
conseguindo uma solução completa...
   Abraços,
Gugu



De:[EMAIL PROTECTED]

Para:obm-l@mat.puc-rio.br

C=F3pia:

Data:Tue, 14 Jun 2005 12:44:01 +

Assunto:[obm-l] Matriz - IMC

 Como fa=E7o essa ... ?
 
 A e B s=E3o matrizes reais NxN tal que A^2 + B^2 =3D AB e BA - AB =E9 i=
nvert=EDvel . 
 Prove q N =E9 m=FAltiplo de 3.
 
 []`s
 Daniel Regufe
 

O site da IMC tem todas as solu=E7=F5es.
http://www.imc-math.org/

[]s,
Claudio.

=
Instruções para entrar na lista, sair da lista e usar a lista em
http://www.mat.puc-rio.br/~nicolau/olimp/obm-l.html
=


Re: [obm-l] numeros binomiais, conjectura

2005-06-15 Por tôpico Carlos Gustavo Tamm de Araujo Moreira
   Este belo argumento combinatório do Shine generaliza a idéia do Dirichlet
e da' uma prova do resultado quando os A_j e os n_j são naturais. Aproveito 
para dar outra prova deste resultado, agora por indução, usando a relação de
Stiffel C(n+1,k+1)=C(n,k+1)+C(n,k). Lembremos que queremos provar que, para
A = A_1+A_2+...+A_t, soma(produto(C(A_j,n_j),j=1..t),n_1+...+n_t=n)=C(A,n) (os 
A_i são dados e os n_i variam sobre as t-uplas de naturais cuja soma e' n). 
Da' para provar isso por indução em A+n+t: Se A+n+t=1, t=1, A=n=0 e os dois 
lados valem 1. Em geral, supondo A_10, temos, por hipótese de indução,
C(A-1,n)=soma(C(A_1-1,n_1).produto(C(A_j,n_j),j=2..t),n_1+...+n_t=n) e 
C(A-1,n-1)=soma(C(A_1-1,n_1-1).produto(C(A_j,n_j),j=2..t),n_10,n_1+...+n_t=n-1).
Note que, se n_1=0, C(A_1,n_1)=1=C(A_1-1,n_1). Assim, como, para n_10,
C(A_1-1,n_1)+C(A_1-1,n_1-1)=C(A_1,n_1), somando as duas igualdades, obtemos 
C(A,n)=C(A-1,n)+C(A-1,n-1)=soma(C(A_1,n_1).produto(C(A_j,n_j),j=2..t),n_1+...+n_t=n)=
=soma(produto(C(A_j,n_j),j=1..t),n_1+...+n_t=n). O caso A_1=0 segue de
trocar t por t-1, o que termina a prova por indução. 
   Entretanto o Eric havia dito que queria provar o resultado supondo os A_j
complexos. E' possível deduzir o caso complexo do caso natural (exercício!). 
Por outro lado, vale a pena observar que esse meu argumento com funções
geratrizes funciona no caso complexo, definindo, para |x|1 e A complexo,
(1+x)^A=e^(A.log(1+x)), onde log(1+x)=x-x^2/2+x^3/3-x^4/4+... (e, para todo
z complexo, e^z=1+z+z^2/2+z^3/6+...=soma(k=0 a infinito)(z^k/k!)). Além
disso, ainda vale, para |x|1, (1+x)^A=1+Ax+(A(A-1)/2).x^2+...=
=soma(k=0 a infinito)(C(A,k).x^k).
   Abraços,
 Gugu


Uma outra maneira de provar é usando combinatória:

Considere t caixinhas C_1, C_2, ..., C_t, sendo que a
caixinha C_i tem A_i objetos, ou seja, temos A = A_1 +
A_2 +...+ A_t objetos.

Suponha que queiramos retirar um total de n objetos
dessas caixas, sem nenhuma restrição.

Podemos fazer isso retirando N_i objetos da caixa C_i,
de modo que N_1 + N_2 + ... + N_t = n. O número de
maneiras de retirar os objetos dessa maneira é igual a
produto(C(A_j,n_j),j=1..t.

O total de maneiras de retirar n objetos é obtido
considerando todas as possibilidades para os números
naturais N_1, N_2, ..., N_t tais que N_1 + N_2 + ... +
N_t = n; então, esse total é
  soma(produto(C(A_j,n_j),j=1..t),n_1+...+n_t=n).

Mas podemos escolher os n objetos da seguinte maneira:
retire todos os A objetos de suas caixas e escolha,
dentre eles, os seus n objetos. Há C(A,n) maneiras de
se fazer isso. Assim, o total acima é também igual a  
C(A,n), ou seja,

soma(produto(C(A_j,n_j),j=1..t),n_1+...+n_t=n)=C(A,n).

A maneira que o Gugu propôs, na verdade, é muito
parecida com a que escrevi; ele faz a mesma contagem
dupla acima utilizando séries formais (as famosas
funções geratrizes). Tem um artigo na Eureka! 12 (não
sei ao certo se o número é esse mesmo) falando
exatamente sobre esse assunto. Vale a pena ler.

[]'s
Shine

--- Carlos Gustavo Tamm de Araujo Moreira
[EMAIL PROTECTED] wrote:

Pelo que eu entendi, os A_i são dados e os N_i
 variam sobre as t-uplas de
 naturais cuja soma e' n. Uma prova relativamente
 curta e' a seguinte:
 escreva (1+x)^A=produto((1+x)^A_j,j=1..t) e olhe
 para o coeficiente de x^n
 em cada um dos dois lados: eles são C(A,n) e
 soma(produto(C(A_j,n_j),j=1..t),n_1+...+n_t=n),
 respectivamente.
Abraços,
   Gugu
  
 
 Sejam 
 
 n = n_1 + n_2 +...+ n_t 
 A = A_1 + A_2 +...+ A_t
 
 Entao
 
 soma(produto(C(A_j,n_j),j=1..t),n_1+...+n_t=n) =
 C(A,n)
 
 Alguem pode me dizer se essa conjectura eh
 verdadeira?
 Se for, ela jah foi provada?
 Alguns casos particulares sao faceis de ver, por
 exemplo:
 
 C(A+B,2)=C(A,2)C(A,0)+C(A,1)C(B,1)+C(A,0)C(B,2)
 
 Supondo que:
 
 C(A+B,n)=soma(C(A,i)C(B,n-i),i=1..n)
 
 Eh facil mostrar que 
 
 C(A+B+C,n)=C((A+B)+C,n)=
 =soma(C(A,i)C(B,j)C(C,k),0=i,j,k=n,i+j+k=n)
 
 Ah! O caso C(A+B,n)=soma(C(A,i)C(B,n-i),i=1..n)
 para n=3 jah verifiquei e estah certo, isto eh:
 
 C(A+B,3)=C(A,3)+C(A,2)C(B,1)+C(A,1)C(B,2)+C(B,3)
 
 Abra,cos!
 
 
 ===
 geocities.yahoo.com.br/mathfire2001
 Enciclopedia de Matematica - Aulas
 Formulas para primos - Grupos de Estudo
 Projeto Matematica para Todos
 [EMAIL PROTECTED]
 ===
 
 __
 Converse com seus amigos em tempo real com o Yahoo!
 Messenger 
 http://br.download.yahoo.com/messenger/ 

=
 Instruções para entrar na lista, sair da lista e
 usar a lista em
 http://www.mat.puc-rio.br/~nicolau/olimp/obm-l.html

=
 
 

=
 Instruções para entrar na lista, sair da lista e
 usar a lista em
 http://www.mat.puc-rio.br/~nicolau/olimp/obm-l.html

Re: [obm-l] numeros binomiais, conjectura

2005-06-12 Por tôpico Carlos Gustavo Tamm de Araujo Moreira
   Pelo que eu entendi, os A_i são dados e os N_i variam sobre as t-uplas de
naturais cuja soma e' n. Uma prova relativamente curta e' a seguinte:
escreva (1+x)^A=produto((1+x)^A_j,j=1..t) e olhe para o coeficiente de x^n
em cada um dos dois lados: eles são C(A,n) e
soma(produto(C(A_j,n_j),j=1..t),n_1+...+n_t=n), respectivamente.
   Abraços,
  Gugu
 

Sejam 

n = n_1 + n_2 +...+ n_t 
A = A_1 + A_2 +...+ A_t

Entao

soma(produto(C(A_j,n_j),j=1..t),n_1+...+n_t=n) =
C(A,n)

Alguem pode me dizer se essa conjectura eh verdadeira?
Se for, ela jah foi provada?
Alguns casos particulares sao faceis de ver, por
exemplo:

C(A+B,2)=C(A,2)C(A,0)+C(A,1)C(B,1)+C(A,0)C(B,2)

Supondo que:

C(A+B,n)=soma(C(A,i)C(B,n-i),i=1..n)

Eh facil mostrar que 

C(A+B+C,n)=C((A+B)+C,n)=
=soma(C(A,i)C(B,j)C(C,k),0=i,j,k=n,i+j+k=n)

Ah! O caso C(A+B,n)=soma(C(A,i)C(B,n-i),i=1..n)
para n=3 jah verifiquei e estah certo, isto eh:

C(A+B,3)=C(A,3)+C(A,2)C(B,1)+C(A,1)C(B,2)+C(B,3)

Abra,cos!


===
geocities.yahoo.com.br/mathfire2001
Enciclopedia de Matematica - Aulas
Formulas para primos - Grupos de Estudo
Projeto Matematica para Todos
[EMAIL PROTECTED]
===

__
Converse com seus amigos em tempo real com o Yahoo! Messenger 
http://br.download.yahoo.com/messenger/ 
=
Instruções para entrar na lista, sair da lista e usar a lista em
http://www.mat.puc-rio.br/~nicolau/olimp/obm-l.html
=


=
Instruções para entrar na lista, sair da lista e usar a lista em
http://www.mat.puc-rio.br/~nicolau/olimp/obm-l.html
=


Re: [obm-l] Transcendentes - forma definitiva.

2005-05-19 Por tôpico Carlos Gustavo Tamm de Araujo Moreira
  Oi Cláudio,
  Isso não é exatamente verdade não. A seqüência a(n) converge se e somente
se e^(-e) = x = e^(1/e). Se 0xe^(-e), a seqüência a(n) tem dois valores
de aderência em (0,1). O caso 0x1 da' um pouco mais de trabalho que o aso
x = 1, mas também é legal.
  Abraços,
   Gugu


Esse tamb=E9m =E9 um belo problema:

Prove que se a(1) =3D x  0 e a(n+1) =3D x^a(n), para n =3D 1, ent=E3o a=
 sequ=EAncia ((a(n)) converge se e somente se x =3D e^(1/e).

[]s,
Claudio.

De:[EMAIL PROTECTED]

Para:obm-l@mat.puc-rio.br

C=F3pia:

Data:Wed, 18 May 2005 00:53:25 +

Assunto:Re:[obm-l] Transcendentes - forma definitiva.

 Oi Claudio e demais colegas
 desta lista ... OBM-L,
 
 Resposta correta.
 
 Em linhas gerais, a historia do problema e a seguinte : alguem resolveu=
 um 
 problema mostrando que haviam duas respostas possiveis, uma das quais 
 deveria ser falsa. Uma estudante reclamou querendo saber a opcao corret=
a. Eu 
 invoquei o teorema do Gelfond e identifiquei a resposta correta :
 
 Gelfond =3D raiz(2)^raiz(2) e transcendente =3D e irracional
 
 E entao resolvi construir explicitamente uma sequencia de numeros 
 transcendentes que tinha como limite um numero natural. Aqui entrou o G=
UGU, 
 reclamando que mesmo nao sabendo provar a transcendencia, nao haviam 
 hipoteses suficientes para postular tal transcendencia. A reclamacao de=
le, 
 correta e justificavel, era implicitamente a proposicao de um problema =
: 
 este problema abaixo, onde voce ensaia uma solucao ...
 
 Voce faz a observacao basica e fundamental : fixando a base, o restan=
te ( 
 o expoente ) tende para o mesmo limite. Dai, na sua linguagem, r=3Dt^r.=
 Daqui 
 sai tranquilo o resto. Note que se voce faz uma tal passagem perante al=
gumas 
 assembleias que amam o detalhe, muito provavelmente voce sera linchado =
e 
 execrado... Eu sempre achei notavel a capacidade de algumas pessoas de =

 essencializar
 o trivial e trivializar o essencial. Mas elas existem. E sao muitas !
 
 Note tambem que o pulo logico final precisa ser conectado com o e as =

 demais hipoteses.
 
 Quer descobrir algo que vai lhe surpreender ? Mantenha do lado esquerdo=
 do 
 cerebro o numero e=3D2,71... Com o lado direito estude as sequencias da=
 forma 
 X^X^X^... QUE CONVERGEM. Procure descobrir algo equivalente a um raio =
de 
 convergencia.
 
 E com os melhores votos
 de paz profunda, sou
 
 Paulo Santa Rita
 3,2154,170505
 
 From: claudio.buffara 
 Reply-To: obm-l@mat.puc-rio.br
 To: obm-l 
 Subject: Re:[obm-l] Transcendentes - forma definitiva.
 Date: Tue, 17 May 2005 13:47:18 -0300
 
   Ola Pessoal desta
   lista ... OBM-L,
  
   Esse problema e antigo, bonito e foi proposto aqui nesta lista - se=
 nao 
 me
   falha a memoria - pelo Prof Carlos Gustavo (GUGU), em uma forma men=
os 
 geral.
   Peco desculpas a todos por tantas correcoes.
  
   Seja T um transcendente da forma i^i, onde i e um irracional algebr=
ico.
   Definimos a sequencia :
  
   A(1) =3D T
   A(N+1) =3D T^A(N)
  
   Se LIM A(N)=3Dr, r racional, Considere a afirmacao : Existe N, N
   suficientemente grande, tal que A(N) e algebrico. Voce consegue pr=
ovar 
 ou
   refutar esta afirmacao ? Note que nao e possivel aplicar, DIRETAMEN=
TE, o
   teorema de Gelfond.
  
   Um Abraco a Todos
   Paulo Santa Rita
   3,1242,170505
  
  
 Oi, Paulo:
 
 Se lim A(n) existe e =E9 igual ao racional r, ent=E3o lim A(n+1) =3D r=
.
 Portanto, teremos: r =3D t^r =3D=3D
 t =3D r^(1/r) =3D alg=E9brico =3D=3D
 contradi=E7=E3o, pois estamos supondo que t =E9 transcendente.
 
 Logo, ou lim A(n) n=E3o existe ou existe mas =E9 irracional.
 
 Assim, a senten=E7a:
 lim A(n) =E9 racional =3D=3D A(N) =E9 alg=E9brico para algum N sufici=
entemente 
 grande
 =E9 verdadeira, j=E1 que a sua premissa =E9 falsa.
 
 Era isso o que voc=EA tinha em mente?
 
 []s,
 Claudio.
 

--_=__=_XaM3_.1116385718.2A.615916.42.31897.52.42.007.1250663696
Content-Type: text/html; charset=iso-8859-1
Content-Transfer-Encoding: quoted-printable

DIVEsse tamb=E9m =E9 um belo problema:/DIV
DIVnbsp;/DIV
DIVProve que se a(1) =3D x gt; 0 e a(n+1) =3D x^a(n), para n gt;=3D 1=
, ent=E3o a sequ=EAncia ((a(n)) converge se e somente se x lt;=3D e^(1/e=
)./DIV
DIVnbsp;/DIV
DIV[]s,/DIV
DIVClaudio./DIV
DIVnbsp;/DIV
DIV
TABLE cellSpacing=3D0 cellPadding=3D4 width=3D100% bgColor=3D#f0f0f0 b=
order=3D0
TBODY
TR
TD width=3D70 bgColor=3D#bde9fdFONT face=3DVerdana,Arial,'Trebuchet M=
S' size=3D2BDe:/B/FONT/TD
TDFONT face=3DVerdana,Arial,'Trebuchet MS' size=3D2[EMAIL PROTECTED]
puc-rio.br/FONT/TD/TR/TBODY/TABLE/DIV
DIV
TABLE cellSpacing=3D0 cellPadding=3D4 width=3D100% bgColor=3D#f0f0f0 b=
order=3D0
TBODY
TR
TD width=3D70 bgColor=3D#bde9fdFONT face=3DVerdana,Arial,'Trebuchet M=
S' size=3D2BPara:/B/FONT/TD
TDFONT face=3DVerdana,Arial,'Trebuchet MS' size=3D2[EMAIL PROTECTED]
o.br/FONT/TD/TR/TBODY/TABLE/DIV
DIV
TABLE cellSpacing=3D0 cellPadding=3D4 width=3D100% bgColor=3D#f0f0f0 b=
order=3D0
TBODY
TR
TD width=3D70 bgColor=3D#bde9fdFONT 

Re: [obm-l] Problemas de Algebra

2005-05-14 Por tôpico Carlos Gustavo Tamm de Araujo Moreira

On Fri, May 13, 2005 at 01:48:56PM -0300, Carlos Gustavo Tamm de Araujo 
Moreira wrote:
Oi Claudio,
Qual e' esse problema 26 da secao 2.5 ?
Gostei muito do exemplo do Nicolau. Eu pensei em alguns outros depois de
 responder a mensagem, por exemplo, um grupo G gerado por a e b com b de ordem
 2 e sem outras relacoes. O conjunto H dos elementos cuja representacao
 simplificada e' uma palavra formada pelas letras a, a^(-1) e b tal que o
 numero de a's nas suas k primeiras letras e' sempre maior ou igual ao numero
 de a^(-1)'s, para todo k, e' um subgrupo e aHa^(-1) esta' estritamente
 contido em H - senao b=axa^(-1) para algum x em H, mas x tem que ser
 a^(-1)ba, que nao esta' em H.

Faltou dizer que o número total de a's é 0 (onde a^(-1) conta como -1 a).

   Oi Nicolau, obrigado pela correção - é isso mesmo, o que equivale a dizer que
os elementos de H #e seus inversos# devem ter a propriedade acima sobre o 
número 
acumulado de a's ser sempre não-negativo.
   Abraços,  
 Gugu


Uma fábrica geral de exemplos é a seguinte.
Comece escolhendo um grupo H0 (que será o H)
e um automorfismo f entre H0 e um subgrupo próprio H1.
Considere o conjunto H0 x Z e defina a relação de equivalencia
(h1,n1) ~ (h2,n2) se e somente se h1 = f^(n2-n1)(h2), n2 = n1 ou
h2 = f^(n1-n2)(h1), n1 = n2.
Seja HZ o quociente de H0 x Z por ~.
Podemos identificar naturalmente H0 com {(h,0) em HZ}
e definir f: HZ - HZ por f((h,n)) = (h,n+1).
Defina ainda um produto iem Hz por (h1,n)*(h2,n) = (h1*h2,n),
assim HZ é um grupo e f um automorfismo externo levando H0
em um subgrupo próprio H1. Podemos definir G como
o produto semidireto G = HZ :f Z. Mais precisamente,
os elementos de G são da forma (h,m) = h*g^m, h em HZ, m em Z,
e g é um elemento novo. A multiplicação em G é determinada por
g*h = f(h)*g. Assim HZ é um subgrupo de G e f passa a ser
uma conjugação em G.

[]s, N.
=
Instruções para entrar na lista, sair da lista e usar a lista em
http://www.mat.puc-rio.br/~nicolau/olimp/obm-l.html
=


=
Instruções para entrar na lista, sair da lista e usar a lista em
http://www.mat.puc-rio.br/~nicolau/olimp/obm-l.html
=


Re: [obm-l] Problemas de Algebra

2005-05-13 Por tôpico Carlos Gustavo Tamm de Araujo Moreira
   Oi Claudio,
   De fato esse exemplo com dois elementos corresponde a esse exemplo das
matrizes sobre Z/2Z (em geral corresponde as matrizes como essas com
a(1,1)=1 e a(1,2) em {0,1}).
   Abracos,
 Gugu


O das matrizes tudo bem, mas esse exemplo com dois elementos foi chato!

Muito obrigado.

[]s,
Claudio.

on 13.05.05 00:19, Carlos Gustavo Tamm de Araujo Moreira at [EMAIL PROTECTED]
wrote:

 
 Oi, pessoal:
 
 Preciso de ajuda nos seguintes problemas sobre grupos do Herstein - Topics
 in Algebra:
 
 Secao 2.4:
 
 13) De um exemplo de um conjunto S, fechado em relacao a uma operacao
 associativa * e tal que:
 i) Existe e em S, tal que a*e = a, para todo a em S;
 ii) Para todo a em S, existe y(a) em S tal que y(a)*a = e;
 iii) S nao eh um grupo.
 
 Um exemplo: S={e,a}, e.e=e, a.e=a, e.a=e, a.a=a. Mais geralmente,
 S={matrizes 2x2 com segunda coluna nula e a(1,1) nao nulo; e=(1,0; 0,0)}.
 
 
 
 Secao 2.6:
 
 8) De um exemplo de um grupo G, um subgrupo H, e um elemento a de G tais que
 aHa^(-1) estah propriamente contido em H.
 
 Um tal H, se existir, tem que ser necessariamente infinito, alem de
 nao-abeliano. Eu imagino que deva haver algum grupo de matrizes com esta
 propriedade, mas nao consegui pensar em nenhum.
 
 Esse eu achei mais dif?cil: acho que podemos tomar um grupo gerado por
 elementos a e x(n), com n inteiro, que s? satisfazem as rela??es
 a.x(n).a^(-1)=x(n+1), e H gerado pelos x(n) com n natural (aHa^(-1) vai ser
 gerado pelos x(n+1) com n natural). Talvez haja exemplos mais simples e
 naturais...  
 
 
 []s,
 Claudio.
 


=
Instruções para entrar na lista, sair da lista e usar a lista em
http://www.mat.puc-rio.br/~nicolau/olimp/obm-l.html
=


=
Instruções para entrar na lista, sair da lista e usar a lista em
http://www.mat.puc-rio.br/~nicolau/olimp/obm-l.html
=


Re: [obm-l] Problemas de Algebra

2005-05-13 Por tôpico Carlos Gustavo Tamm de Araujo Moreira
   Oi Claudio,
   Qual e' esse problema 26 da secao 2.5 ?
   Gostei muito do exemplo do Nicolau. Eu pensei em alguns outros depois de
responder a mensagem, por exemplo, um grupo G gerado por a e b com b de ordem
2 e sem outras relacoes. O conjunto H dos elementos cuja representacao
simplificada e' uma palavra formada pelas letras a, a^(-1) e b tal que o
numero de a's nas suas k primeiras letras e' sempre maior ou igual ao numero
de a^(-1)'s, para todo k, e' um subgrupo e aHa^(-1) esta' estritamente
contido em H - senao b=axa^(-1) para algum x em H, mas x tem que ser
a^(-1)ba, que nao esta' em H. Outro exemplo e' G={bijecoes crescentes de R
em R} e H={f(x) em G tal que lim (x-+oo) f(x)/x existe e e' racional 
positivo}. Se a=a(x)=x^3, afa^(-1)(x)=f(x^(1/3))^3, e, se f(x)/x tende a c
racional positivo entao afa^(-1)(x)/x tende a c^3, que tambem e' racional
positivo. Por outro lado, f(x)=2x pertence a H, mas se afa^(-1)(x)=2x entao
f(y)=2^(1/3).y, que nao pertence a H. Eu tentei um pouco achar um exemplo
natural com G={bijecoes de N}, mas so' consegui versoes artificiais de
exemplos anteriores...  
   Abracos,
 Gugu


Maravilha! Muito obrigado.

Com isso, eu fecho as secoes 2.1 a 2.6 do Herstein - Topics in Algebra, com
excecao do problema 26 da secao 2.5 que nem o proprio autor conseguiu fazer
(usando apenas o material das secoes 2.1 a 2.5, claro!).

[]s,
Claudio.

on 13.05.05 09:35, Nicolau C. Saldanha at [EMAIL PROTECTED] wrote:

 On Fri, May 13, 2005 at 12:19:12AM -0300, Carlos Gustavo Tamm de Araujo
 Moreira wrote:
 8) De um exemplo de um grupo G, um subgrupo H, e um elemento a de G tais 
 que
 aHa^(-1) estah propriamente contido em H.
 
 Um tal H, se existir, tem que ser necessariamente infinito, alem de
 nao-abeliano. Eu imagino que deva haver algum grupo de matrizes com esta
 propriedade, mas nao consegui pensar em nenhum.
 
 Esse eu achei mais dificil: acho que podemos tomar um grupo gerado por
 elementos a e x(n), com n inteiro, que so satisfazem as relações
 a.x(n).a^(-1)=x(n+1), e H gerado pelos x(n) com n natural (aHa^(-1) vai ser
 gerado pelos x(n+1) com n natural). Talvez haja exemplos mais simples e
 naturais...  
 
 O exemplo do Gugu é perfeitamente correto. Aqui vai outro,
 talvez mais simples ou mais natural, ou talvez não.
 Tem a vantagem de ser um grupo de matrizes bem conhecido,
 como o Claudio imaginou.
 
 Tome G = SL(2,R), o grupo das matrizes reais 2x2 com determinante igual a 1;
 H é cíclico infinito com elementos [[1,n],[0,1]], n inteiro;
 a é a matriz diagonal [[2,0],[0,1/2]];
 fazendo a conta temos a[[1,n],[0,1]]a^(-1) = [[1,4n],[0,1]]
 donde aHa^(-1) é um subgrupo próprio (de índice 4) de H.
 
 []s, N.
 =
 Instruções para entrar na lista, sair da lista e usar a lista em
 http://www.mat.puc-rio.br/~nicolau/olimp/obm-l.html
 =
 


=
Instruções para entrar na lista, sair da lista e usar a lista em
http://www.mat.puc-rio.br/~nicolau/olimp/obm-l.html
=


=
Instruções para entrar na lista, sair da lista e usar a lista em
http://www.mat.puc-rio.br/~nicolau/olimp/obm-l.html
=


Re: [obm-l] identidade

2005-05-12 Por tôpico Carlos Gustavo Tamm de Araujo Moreira
Vamos lá: (z^m-w^m)/(z-w)=z^(m-1)+z^(m-2)w+...+w^(m-1). Isso menos m.w^(m-1) 
da' soma(k=1 a m-1)(w^(m-k-1).(z^k-w^k))=(z-w).soma(k=1 a m-1)(w^(m-k-1).s_k), 
onde s_k=z^(k-1)+z^(k-2)w+...+w^(k-1). Nessa soma cada termo w^(m-j-2).z^j
com 0=j=m-2 aparece m-j-1 vezes (desde k=j+1 até k=m-1), o que mostra ue
essa soma é soma(j=0 a m-2)((m-j-1).w^(m-j-2).z^j), que é igual (fazendo
k=m-j-1) a soma(k=1 a m-1)(k.w^(k-1).z^(m-k-1)), como queríamos.
  Abraços,
Gugu


Ola pessoal.
Como posso chegar na seguinte igualdade (operando apenas com os termos 
do lado esquerdo)


[(z^m-w^m)/(z-w)]-[m*w^(m-1)]=(z-w)*Soma[1=k=m-1](k*w^(k-1)*z^(m-k-1)

(supondo m = 2, e só pra ficar claro; Soma = Somatorio para k indo de 1 
até m-1)

Eu tentei fazer desenvolvendo z^m - w^m, mas sem sucesso no final.
Alguem tem alguma sugestão?

Obrigado

Niski
=
Instruções para entrar na lista, sair da lista e usar a lista em
http://www.mat.puc-rio.br/~nicolau/olimp/obm-l.html
=


=
Instruções para entrar na lista, sair da lista e usar a lista em
http://www.mat.puc-rio.br/~nicolau/olimp/obm-l.html
=


Re: [obm-l] Problemas de Algebra

2005-05-12 Por tôpico Carlos Gustavo Tamm de Araujo Moreira

Oi, pessoal:

Preciso de ajuda nos seguintes problemas sobre grupos do Herstein - Topics
in Algebra:

Secao 2.4:

13) De um exemplo de um conjunto S, fechado em relacao a uma operacao
associativa * e tal que:
i) Existe e em S, tal que a*e = a, para todo a em S;
ii) Para todo a em S, existe y(a) em S tal que y(a)*a = e;
iii) S nao eh um grupo.

   Um exemplo: S={e,a}, e.e=e, a.e=a, e.a=e, a.a=a. Mais geralmente,
S={matrizes 2x2 com segunda coluna nula e a(1,1) nao nulo; e=(1,0; 0,0)}.



Secao 2.6:

8) De um exemplo de um grupo G, um subgrupo H, e um elemento a de G tais que
aHa^(-1) estah propriamente contido em H.

Um tal H, se existir, tem que ser necessariamente infinito, alem de
nao-abeliano. Eu imagino que deva haver algum grupo de matrizes com esta
propriedade, mas nao consegui pensar em nenhum.

   Esse eu achei mais difícil: acho que podemos tomar um grupo gerado por
elementos a e x(n), com n inteiro, que só satisfazem as relações
a.x(n).a^(-1)=x(n+1), e H gerado pelos x(n) com n natural (aHa^(-1) vai ser
gerado pelos x(n+1) com n natural). Talvez haja exemplos mais simples e
naturais...  


[]s,
Claudio.

=
Instruções para entrar na lista, sair da lista e usar a lista em
http://www.mat.puc-rio.br/~nicolau/olimp/obm-l.html
=


=
Instruções para entrar na lista, sair da lista e usar a lista em
http://www.mat.puc-rio.br/~nicolau/olimp/obm-l.html
=


Re: [obm-l] Irredutíveis e Anéis

2005-05-11 Por tôpico Carlos Gustavo Tamm de Araujo Moreira
  Se p divide (a+b.raiz(3))(c+d.raiz(3)), entao p divide
(a^2-3b^2)(c^2-3d^2), e logo p divide um desses fatores, digamos a^2-3b^2.  
Como x^2-3 'e irredutivel, e logo nao tem raiz em Z/pZ, se p divide a^2-3b^2
entao p divide b (senao b e' invertivel em Z/pZ, e a/b e' raiz de x^2-3), e
logo p divide a, donde p divide a+b.raiz(3).
  Abracos,
   Gugu


--_=__=_XaM3_.1115735981.2A.732637.42.23694.52.42.007.339369759
Content-Type: text/plain; charset=iso-8859-1
Content-Transfer-Encoding: quoted-printable

Preciso de ajuda com o exerc=EDcio 3 da se=E7=E3o IV.4 do livro Elementos=
 de =C1lgebra (Arnaldo Garcia e Yves Lequain - Projeto Euclides):

a) Mostre que Z[raiz(3)] =E9 isomorfo a Z[x]/(x^2-3).

b) Seja p um primo de Z. Mostre que p =E9 um elemento primo de Z[raiz(3)]=
 se e somente se o polin=F4mio x^2 - 3 =E9 irredut=EDvel em (Z/pZ)[x].

Eu fiz o item (a) mostrando que o ideal (x^2-3) =E9 o n=FAcleo do homomor=
fismo sobrejetor H:Z[x] - Z[raiz(3)] dado por H(f(x)) =3D f(raiz(3)) e i=
nvocando o teorema dos homomorfismos.

No item (b) eu provei que se p =E9 primo em Z[raiz(3)] ent=E3o x^2 - 3 =E9=
 irredut=EDvel (de fato, eu provei o contrapositivo):
x^2 - 3 n=E3o =E9 irredut=EDvel em Z_p[x]  =3D=3D
x^2 - 3 tem uma raiz em Z_p[x]  =3D=3D
existe um inteiro a tal que a^2 =3D=3D 3 (mod p) =3D=3D

p divide a^2 - 3 =3D (a + raiz(3))(a - raiz(3)) em Z[raiz(3)].

Mas p n=E3o divide a + raiz(3) nem a - raiz(3) pois se dividisse, teria q=
ue dividir os coeficientes de raiz(3) respectivos, iguais a 1 e -1, o que=
 =E9 uma contradi=E7=E3o, pois p =E9 um primo de Z.
Logo, p n=E3o =E9 primo em Z[raiz(3)].

No entanto, n=E3o estou conseguindo provar a rec=EDproca. Imagino que, de=
 alguma forma, eu tenha que usar o item (a).

Qualquer ajuda ser=E1 bem vinda.

[]s,
Claudio.

--_=__=_XaM3_.1115735981.2A.732637.42.23694.52.42.007.339369759
Content-Type: text/html; charset=iso-8859-1
Content-Transfer-Encoding: quoted-printable

DIVPreciso de ajuda comnbsp;o exerc=EDcio 3 da se=E7=E3o IV.4 do livro=
 Elementos de =C1lgebra (Arnaldo Garcia e Yves Lequain - Projeto Euclides=
):/DIV
DIVnbsp;/DIV
DIVa) Mostre que Z[raiz(3)] =E9 isomorfo a Z[x]/(x^2-3)./DIV
DIVnbsp;/DIV
DIVb) Seja p um primo de Z. Mostre que p =E9 um elemento primo de Z[rai=
z(3)] se e somente se o polin=F4mio x^2 - 3 =E9 irredut=EDvel em (Z/pZ)[x=
]./DIV
DIVnbsp;/DIV
DIVEu fiz o item (a) mostrando que o ideal (x^2-3) =E9 o n=FAcleo do ho=
momorfismo sobrejetor H:Z[x] -gt; Z[raiz(3)]nbsp;dado por H(f(x)) =3D f=
(raiz(3)) e invocando o teorema dos homomorfismos./DIV
DIVnbsp;/DIV
DIVNo item (b) eu provei que se p =E9 primo em Z[raiz(3)] ent=E3o x^2 -=
 3 =E9 irredut=EDvel (de fato, eu provei o contrapositivo):/DIV
DIVx^2 - 3 n=E3o =E9 irredut=EDvel em Z_p[x]nbsp; lt;=3D=3Dgt;/DIV=

DIVx^2 - 3 tem uma raiz em Z_p[x]nbsp; lt;=3D=3Dgt;/DIV
DIVexiste um inteiro a tal que a^2 =3D=3Dnbsp;3 (mod p) =3D=3Dgt;/DI=
V
DIVnbsp;/DIV
DIVp divide a^2 - 3 =3D (a + raiz(3))(anbsp;- raiz(3)) em Z[raiz(3)].=
/DIV
DIVnbsp;/DIV
DIVMas p n=E3o divide a + raiz(3) nem a - raiz(3) pois se dividisse, te=
ria que dividir os coeficientes de raiz(3)nbsp;respectivos, iguais a 1 e=
 -1, o que =E9 uma contradi=E7=E3o, pois p =E9 um primo de Z./DIV
DIVLogo, p n=E3o =E9 primo em Z[raiz(3)]./DIV
DIVnbsp;/DIV
DIVNo entanto, n=E3o estou conseguindo provar a rec=EDproca. Imagino qu=
e, de alguma forma, eu tenha que usar o item (a)./DIV
DIVnbsp;/DIV
DIVQualquer ajuda ser=E1 bem vinda./DIV
DIVnbsp;/DIV
DIV[]s,/DIV
DIVClaudio./DIV
DIVnbsp;/DIV

--_=__=_XaM3_.1115735981.2A.732637.42.23694.52.42.007.339369759--

=
Instrugues para entrar na lista, sair da lista e usar a lista em
http://www.mat.puc-rio.br/~nicolau/olimp/obm-l.html
=


=
Instruções para entrar na lista, sair da lista e usar a lista em
http://www.mat.puc-rio.br/~nicolau/olimp/obm-l.html
=


Re: [obm-l] Elementos de um Grupo

2005-04-27 Por tôpico Carlos Gustavo Tamm de Araujo Moreira
   Oi Cláudio,
   De a^(-1)*b^2*a=b^3 segue b^2*a*b^(-2)=a*b. De b^(-1)*a^2*b = a^3 segue
b^(-2)*a^4*b^2=b^(-1)*a^6*b=a^9, donde a^4=b^2*a^9*b^(-2)=(a*b)^9.
Analogamente, b^4=(b*a)^9. Assim, b*a^4=b*(a*b)^9=(b*a)^9*b=b^4*a, donde
a^3=b^3, e de a^(-1)*b^2*a=b^3=a^3 segue b^2=a^3=b^3, donde b=e, e
analogamente a=e.
Abraços,
   Gugu


a e b sao elementos de um grupo e satisfazem a:
a^(-1)*b^2*a = b^3   e   b^(-1)*a^2*b = a^3
Prove que a = b = e = identidade do grupo.

[]s,
Claudio.

=
Instruções para entrar na lista, sair da lista e usar a lista em
http://www.mat.puc-rio.br/~nicolau/olimp/obm-l.html
=


=
Instruções para entrar na lista, sair da lista e usar a lista em
http://www.mat.puc-rio.br/~nicolau/olimp/obm-l.html
=


Re: [obm-l] Serie condicionalmente convergente

2005-04-27 Por tôpico Carlos Gustavo Tamm de Araujo Moreira
   Bem, eu acho que sei fazer. Não sei se isso já foi estudado antes. Vou
dar um tempo para o pessoal pensar, e depois eu escrevo (para quem quiser eu
posso mandar pelo menos o enunciado da caracterização logo em uma mensagem
pessoal).
   Abraços,
Gugu


Uma pergunta que eu acho mais interessante é a seguinte: Caracterize as
 bijecoes f:N-N tais que para toda série condicionalmente convergente
 A1 + A2 + ... + An + ... , a série Af(1) + Af(2) + ... + Af(n) + ...
converge.

Esse é um problema em aberto não é?
   Até hoje nunca vi solução...

[]s


=
Instruções para entrar na lista, sair da lista e usar a lista em
http://www.mat.puc-rio.br/~nicolau/olimp/obm-l.html
=


=
Instruções para entrar na lista, sair da lista e usar a lista em
http://www.mat.puc-rio.br/~nicolau/olimp/obm-l.html
=


Re: [obm-l] Serie condicionalmente convergente

2005-04-25 Por tôpico Carlos Gustavo Tamm de Araujo Moreira
   Oi Claudio,
   DADA uma série condicionalmente convergente, o conjunto das bijeções que
preservam a convergência, como abaixo, certamente depende da série. Por
exemplo, para a série 1-1/raiz(2)+1/raiz(3)-1/raiz(4)+..., a bijeção f dada 
por f(3k-2)=2k-1, f(3k-1)=4k-2, f(3k)=4k, para todo k=1 produz uma série
divergente, mas para 1-1/2+1/3-1/4+... ela preserva a convergência mas muda
o valor da série, enquanto para 1+0-1/2+0+1/3+0-1/4+0+... ela preserva a
convergência e não muda o valor da série. 
   Uma pergunta que eu acho mais interessante é a seguinte: Caracterize as
bijecoes f:N-N tais que para toda série condicionalmente convergente 
A1 + A2 + ... + An + ... , a série Af(1) + Af(2) + ... + Af(n) + ... converge.
   Abraços,
 Gugu


Oi, Paulo:

Voce poderia dar a solucao deste problema?

[]s,
Claudio.

on 01.03.05 13:48, Paulo Santa Rita at [EMAIL PROTECTED] wrote:
 
 Seja A1 + A2 + ... + An + ... uma serie condicionalmente convergente.
 Caracterize as bijecoes
 f:N-N tais que
 Af(1) + Af(2) + ... + Af(n) converge.
 
 Nota : Af(n) = Termo da serie A1 + A2 + ... + An + ... cujo indice e f(n)
 
 SUGESTAO : note que facilmente voce pode criar uma sequencia semelhante a do
 exercicio que voce acabou de resolver ( inversos dos termos de uma PA ) e
 que converge para log(N)/N, qualquer que seja N. Ora, a expressao log(N)/N e
 bem conhecida e esta relacionada com um famoso teorema da teoria dos numeros
 ...
 
 Um Abraco
 Paulo Santa Rita
 3,1343,010305
 

=
Instruções para entrar na lista, sair da lista e usar a lista em
http://www.mat.puc-rio.br/~nicolau/olimp/obm-l.html
=


=
Instruções para entrar na lista, sair da lista e usar a lista em
http://www.mat.puc-rio.br/~nicolau/olimp/obm-l.html
=


Re: [obm-l] Tetei muito

2005-04-23 Por tôpico Carlos Gustavo Tamm de Araujo Moreira
   Vamos ver essa: ax+by=ax+bc/x, que é mínimo quando ax=bc/x, i.e., quando
x=raiz(bc/a), e nesse caso a expressão vale 2.raiz(abc). Da' para ver que
ax+bc/x=2.raiz(abc) via ax+bc/x-2.raiz(abc)=(raiz(ax)-raiz(bc/x))^2.
   Abraços,
  Gugu
 
 
06) Dados a, b e c positivos, determinar x e y tais que xy = c e que ax + by 
seja o menor possível. 

=
Instruções para entrar na lista, sair da lista e usar a lista em
http://www.mat.puc-rio.br/~nicolau/olimp/obm-l.html
=


Re: [obm-l] Trivialidades Nao ! ( CORRECAO )

2005-04-22 Por tôpico Carlos Gustavo Tamm de Araujo Moreira
   Caro Paulo,
   A pergunta deve ser qual é o MAIOR valor possível para s, não ? De fato,
se for mesmo o menor, a resposta é trivialmente 0. X_1 pode ser tão próximo
de 0 quanto queiramos, o que faz X_1/(1+X_1) ainda mais próximo de 0. Vamos
então mudar desse jeito o enunciado, e tentar resolver o problema.
   Seja r_k=X_k/(1+X_1+X_2+...+X_k). Temos X_1=r_1/(1-r_1), e, por indução,
X_k=r_k/(1-r_1)(1-r_2)...(1-r_k), e 1+X_1+...+X_k=1/(1-r_1)(1-r_2)...(1-r_k)
para todo k=1. Assim, se r_k=s para todo k, temos que
2=1+X_1+...+X_n=1/(1-s)^n, donde 1-s=2^(-1/n), e logo s=1-2^(-1/n)=:C.
Esse é o maior valor possível de s, atingido fazendo r_k=C para todo k, e
logo X_k=C/(1-C)^k, para todo k=1. Note que, de fato, temos
X_1+X_2+...+X_n=(C/(1-C)^n).(1-(1-C)^n)/(1-(1-C))=1/(1-C)^n-1=2-1=1.
Note ainda que, se s fosse, em vez do menor, o maior desses números, e 
perguntássemos pelo menor valor possível de s, a resposta seria a mesma.
   Abraços,
Gugu


Ola Pessoal,

Na mensagem abaixo, na questao 3) leia-se na OBS : nao e permitido usar 
CALCULO DIFERENCIAL ! Nesta mesma questão os valores continuam, isto e,  s 
deve ser o menor valor de :

X1/(1+X1) , X2/(1+X1+x2), X3/(1+X2+X3),..., Xi/(1+X1+...+Xi), 
...,Xn/(1+X1+...+Xn)

Desculpas a todos ( e a pressa ) : !

Paulo Santa Rita
5,1052,100305

From: Paulo Santa Rita [EMAIL PROTECTED]
Reply-To: obm-l@mat.puc-rio.br
To: obm-l@mat.puc-rio.br
Subject: [obm-l] Trivialidades Nao !
Date: Thu, 10 Mar 2005 13:18:41 +

Ola Claudio e demais colegas
desta lista ... OBM-L

Realmente : deprimente !

Esta nossa lista foi concebida originalmente como um meio informal onde 
podemos discutir Problemas Olimpicos de Matematica, nao problemas triviais 
de vestibulares.  Existem muitissimos lugares na internet onde se pode 
postar e discutir estes problemas triviais, mas pouquissimos onde se 
discute problemas de Matematica Olimpica. Assim, dado a raridade e a 
importancia deste nosso local, vamos todos contribuir para manter a sua 
caracteristica e objetivos  iniciais.

PROBLEMAS :

1) IMAGINE um quadriculado em forma de quadrado, 1000X1000, portanto com 
100 quadradinhos. Usando somente os algarismos 0,1 e 2 e possivel 
preencher o quadriculado de tal forma que qualquer retangulo 3x4 ( ou 4x3 ) 
contenha 3 algarismos zeros, 4 algarismos um e 5 algarismos dois ? 
Justifique sua resposta.

2) Sejam X1, X2, ..., Xn numeros reais positivos tais que X1 + X2 + ... + 
Xn =1. Seja s o menor dos numeros : X1/(1+X1) , X2/(1+X1+X2) , 
X3/(1+X1+X2+X3). Qual e o menor valor possivel para s ?
OBS : Alunos das Escolas Russas onde caiu esta questao nao sabem ou nao 
podem CALCULO

3) Prove que para todos A, B e C reais positivos vale :
A^3 + B^3 + c^3 + 3ABC = AB(A+B) + BC(B+C) + AC(A+C)

Mais problemas russos em :
http://www.mat.puc-rio.br/~nicolau/psr

Um Grande Abraco em todos !
Paulo Santa Rita
5,1017,100305


From: Claudio Buffara [EMAIL PROTECTED]
Reply-To: obm-l@mat.puc-rio.br
To: obm-l@mat.puc-rio.br
Subject: Re: [obm-l] Equação
Date: Thu, 10 Mar 2005 00:57:15 -0300

Nesse caso eu nao sei o que eh mais deprimente numa lista que trata de
olimpiadas de matematica: alguem escrever x^2+1/(x+1)^2 quando queria 
dizer
(x^2+1)/(x+1)^2 ou alguem nao saber resolver uma misera equacao do 2o. 
grau.

_
Chegou o que faltava: MSN Acesso Grátis. Instale Já! 
http://www.msn.com.br/discador

=
Instruções para entrar na lista, sair da lista e usar a lista em
http://www.mat.puc-rio.br/~nicolau/olimp/obm-l.html
=

_
Chegou o que faltava: MSN Acesso Grátis. Instale Já! 
http://www.msn.com.br/discador

=
Instruções para entrar na lista, sair da lista e usar a lista em
http://www.mat.puc-rio.br/~nicolau/olimp/obm-l.html
=


=
Instruções para entrar na lista, sair da lista e usar a lista em
http://www.mat.puc-rio.br/~nicolau/olimp/obm-l.html
=


Re: [obm-l] Convergencia pontual

2005-04-22 Por tôpico Carlos Gustavo Tamm de Araujo Moreira
   Caro Tertuliano,
   Da' para provar que f é contínua num conjunto denso. Mais do que isso, f tem
que ser contínua num conjunto residual, i.e., que contém uma interseção 
enumerável de abertos densos em [0,1] (lembremos do teorema de Baire: toda 
interseção enumerável de abertos densos (em R ou num intervalo, entre muitas 
outras situações) é densa). Para isso, basta mostrar que para todo n natural o 
conjunto F_n dos x  em [0,1] tais que a oscilaçao de f no ponto x, definida por 
w(f,x)=lim(h-0)(sup(f|[x-h,x+h])-inf(f|[x-h,x+h])) e' = 1/n é um fechado
(isto e' facil e eu deixo como exercício) de interior vazio (note que a
unico dos F_n é o conjunto dos pontos de descontinuidade de f). Para isso,   
se houver um intervalo (a,b) contido em F_n, definimos, para cada k inteiro,
X_k={x em (a,b) | k/4n=f(x)(k+1)/4n}. Pelo teorema de Baire, o fecho de
X_k deve ter interior não vazio para algum k, senão a união dos fechos dos
X_k seria uma união enumerável de fechados com interior vazio, e logo não
poderia conter (a,b). Suponha que o fecho de X_k contém (c,d), que está
contido em (a,b). Para todo x em X_k, como w(f,x)=1/n, x pertence a A ou a
B, onde 
A={x| f(x)(k+1)/4n e para todo h  0 existe u em (x-h,x+h) com f(u)(k+2)/4n}
e B={x| f(x)=k/4n e para todo h  0 existe u em (x-h,x+h) com f(u)(k-1)/4n}.
Assim, o fecho de A ou o fecho de B tem interior não vazio em (c,d).
Suponhamos que o fecho de A contenha (s,t), que está contido em (c,d). Como
f_n tende a f pontualmente, e os f_n são contínuos, dado um intervalo
I=(z,w), com (k+1)/4n=zw=(k+2)/4n, para todo h0 existe N natural tal que
para todo n=N existe um intervalo J contido em (x-h,x+h) com f_n(J)=I.
Fazendo I_1=((k+1)/4n,(3k+4)/12n) e I_2=((3k+5)/12n,(k+2)/4n), segue que,
para todo N natural, os conjuntos Y_N={y em (s,t)| existe n=N t.q. f_n(y)
pertence a I_1} e Z_N={z em (s,t)| existe n=N t.q. f_n(z) pertence a I_2}
são abertos e densos em (s,t), de modo que (de novo pelo Baire), a
interseção W dos Y_N e dos Z_N (para N natural) é densa (de fato residual)
em (s,t). Entretanto, dado w em W, o limite f(w) de f_n(w) deveria pertencer
simultaneamente ao fecho [(k+1)/4n,(3k+4)/12n] de I_1 e ao fecho
[(3k+5)/12n,(k+2)/4n] de I_2, mas isso é um absurdo, pois esses intervalos
não se intersectam.   
   Abraços,
Gugu


Olá para todos!!

Um professor me propos a seguinte questao:

Considere uma sequencia f_n:[0,1] em R, de funcoes
continuas convergindo pontualmente para f:[0,1] em R.
Mostrar que f é continua em muitos pontos do intervalo
[0,1].
(na realidade, desconfio q f seja continua em um
conjunto denso no intervalo [0,1]).

Grato por qualquer soluçao e/ou comentario.


Obs.: o objetivo é mostrar q nao existe uma sequencia
de funcoes continuas convergindo pontualmente para a
funcao caracteristica dos irracionais, que é um
exercicio do Elon. Como essa funcao caracteristica é
descontinua nos  irracionais, mostrar o que foi
proposto acima resolve o problema. 

__

Yahoo! Messenger - Fale com seus amigos online. Instale agora! 
http://br.download.yahoo.com/messenger/
=
Instruções para entrar na lista, sair da lista e usar a lista em
http://www.mat.puc-rio.br/~nicolau/olimp/obm-l.html
=


=
Instruções para entrar na lista, sair da lista e usar a lista em
http://www.mat.puc-rio.br/~nicolau/olimp/obm-l.html
=


Re: [obm-l] PROBLEMA!

2005-04-04 Por tôpico Carlos Gustavo Tamm de Araujo Moreira
   Caros Wilner e Rafael,
   a1=-a,a2=a,a1+a3=2, logo a3=2+a. As proximas menores somas possiveis sao
a1+a4 e a2+a3, e ambas devem valer 4, logo -a+a4=4, donde a4=4+a e 2+2a=4, 
donde a=1. Assim, a1=-1, a2=1, a3=3, a4=5 e, como devemos ter a4+a5=15,
a5=10. Os numeros sao, portanto: -1,1,3,5,10. E da' certo!
   Abracos,
Gugu


  
  Oi Rafael.

  O problema, tal como formulado, não tem solução.  
  Senão vejamos: denominando os números, na ordem  
  crescente, a1,a2,a3,a4,a5,temos
  a1+a2=0  ou a1=-a2 0 ; 
  a4+a5=15(*)  ; a2+a4=a1+a5=4 ;
  Assim, 0a22 = -2a10 ; 2a44 ; 4a56.
  Mas as ultimas duas são imcompatíveis com(*)!

  De onde vc. tirou o problema? Deve haver algum
engano.
  Por falar nisso, o qye vc. quer dizer com
RESPECTIVAMENTE?
   
  Abraços
  
  Wilner



--- Rafael Alfinito Ferreira [EMAIL PROTECTED]
wrote:
 EU TENTEI, TENTEI E ATÉ AGORA NÃO ENTENDI
 
 AÍ VAI:
 
 DADOS 5 NÚMEROS, AS SOMA 2 A 2 SÃO: 0, 2, 4, 4, 6,
 8, 9, 11, 13 E 15 
 RESPECTIVAMENTE.
 DETERMINE OS NÚMEROS.
 
 DESDE JÁ AGRADEÇO.
 

_
 MSN Messenger: converse online com seus amigos .  
 http://messenger.msn.com.br
 

=
 Instruções para entrar na lista, sair da lista e
 usar a lista em
 http://www.mat.puc-rio.br/~nicolau/olimp/obm-l.html

=
 


   
   
   
Yahoo! Acesso Grátis - Internet rápida e grátis. 
Instale o discador agora! http://br.acesso.yahoo.com/
=
Instruções para entrar na lista, sair da lista e usar a lista em
http://www.mat.puc-rio.br/~nicolau/olimp/obm-l.html
=


=
Instruções para entrar na lista, sair da lista e usar a lista em
http://www.mat.puc-rio.br/~nicolau/olimp/obm-l.html
=


Re: [obm-l] Sequencia de numeros compostos

2005-04-01 Por tôpico Carlos Gustavo Tamm de Araujo Moreira
   Da Eureka 18, página 61:

Você sabia…
Que existem infinitos inteiros positivos ímpares k tais que k.2^n+1 é composto
para todo n ? Tais inteiros k são chamados números de Sierpinski. Em 1962,
John Selfridge provou que 78557 é um número de Sierpinski, e conjectura-se
que seja o menor deles. Atualmente há 11 números menores que 78557 sobre os
quais não se sabe se são números de Sierpinski ou não: 4847, 10223, 19249,
21181, 22699, 24737, 27653, 28433, 33661, 55459 e 67607. O número 5359 fazia
parte dessa lista até 6/12/2003, quando Randy Sundquist ( um participante do
Seventeen or Bust, um projeto distribuído para atacar o problema de
Sierpinski) encontrou o primo 5359.2^5054502+1 , que tem 1521561 dígitos e é o 
quarto maior
primo conhecido, e maior primo conhecido que não é de Merssenne. 
Veja: http://www.seventeenorbust.com para mais informações.

Exercício: Prove que 78557 é um número de Sierpinski, e que existem
infinitos números de Sierpinski a partir das congruências
78557.2^0+1=0 (mod 3)
78557.2^1+1=0 (mod 5)
78557.2^7+1=0 (mod 7)
78557.2^11+1=0 (mod 13)
78557.2^3+1=78557.2^39+1=0 (mod 73)
78557.2^15+1=0 (mod 19)
78557.2^27+1=0 (mod 37).


   Abraços,
  Gugu

P.S.: Agora so' faltam 10: em 30/12/2004 foi achado o primo 
28433.2^7830457+1, tirando o 28433 da lista acima.


on 02.10.04 21:13, Qwert Smith at [EMAIL PROTECTED] wrote:

 
 
 
 From: Claudio Buffara [EMAIL PROTECTED]
 
 on 02.10.04 12:05, Qwert Smith at [EMAIL PROTECTED] wrote:
 
 
 From: Claudio Buffara [EMAIL PROTECTED]
 
 
 E o caso de k*2^n + 1? Para que valor de k isso eh sempre composto?
 
 
 Vou escrever so a solucao pro Super Buffara ver se confere...
 o raciocinio escrevo assim ki tiver tempo
 
 para k*2^n + 1 basta k=[(3*5*11*17)*t + 1] ou
 k= 2805*t + 1 com t inteiro  0
 
 Boa tentativa, mas 2806*2^8+1 = 718337 eh primo.
 Por acaso voce usou o TCR?
 
 []s,
 Claudio.
 
 
 Poxa, foi uma bobeira que nao sei explicar...
Nesse caso, use a explicacao padrao: era um teste pra ver se as pessoas
estavem prestando atencao...

 olhando de volta
 no guardanapo onde tinha escrito isso as potencias de 2 eram
 2, 4, 8, 16, 32, 64, 128, 516 (acho ki embolei 256 e 512)
Ja vi piores aqui na lista. Por exemplo, o meu 14 == -1 (mod 13).

 Agora... se 2^8 fosse 516 tinha matado o problema :).
 Nao usei TCR nao, quer dizer acho ki nao
 pelo menos diretamente...fiz meio que reinventando a roda.
 Infelizmente nao conheco a terminologia matematica suficiente
 pra classificar o metodo. Mas vou descrever e vc me diz o que
 que e.  Comecei com a mesma ideia dos outros problemas
 identificar um m onde 2^n = -1 (mod m) pra qualquer n.
 Como eh impossivel passei ao plano B. Idetinficar alguns 'm's
 2^n = -1 (mod m) para parte dos 'n's. Isso na minha opniao eh
 uma aplicacao abaianada (com todo respeito) do TCR.
Possivelmente.

 Dividi on 'n's em 4 conjuntos: [4t], [4t+1], [4t+2] e [4t+3]
 Se existir um grupo finito de 'm's onde 2^n = -1 (mod m_i) em
 todos os casos acima entao k = m_1*m_2*...*m_i + 1.
 
 O '11' da minha resposta foi baseado na lambanca anterior de
 2^8 = 516 = -1 (mod 11).  Agora estou em duvida se da pra
 achar finitos 'm's.  O problema sao os casos onde n eh potencia
 de 2.  Um dia vou aprender matematica e ai vcs vao ver so :).
 Mas espera sentado viu?

Esse eh um teorema provado por Sierpinski: existem infinitos impares k tais
que k*2^n + 1 eh composto. Conjectura-se que o menor k com essa propriedade
eh 78557 = 17*4621. De uma olhada em: http://www.prothsearch.net/sierp.html

[]s,
Claudio.

=
Instruções para entrar na lista, sair da lista e usar a lista em
http://www.mat.puc-rio.br/~nicolau/olimp/obm-l.html
=


=
Instruções para entrar na lista, sair da lista e usar a lista em
http://www.mat.puc-rio.br/~nicolau/olimp/obm-l.html
=


Re: [obm-l] Problemas em aberto - prob 10

2005-01-18 Por tôpico Carlos Gustavo Tamm de Araujo Moreira
^(pq)=(B^p+s.q^r)^q-B^(pq)=s.B^p.q^(r+1) (mod q^(r+2)), pois q é
primo ímpar, donde a maior potência de q que divide A^(pq)-B^(pq) é q^(r+1),
e logo a maior potência de q que divide (A^(pq)-B^(pq))/(A^p-B^p) é q, donde 
(A^(pq)-B^(pq))/(A^p-B^p)=q (pois é uma potência de q). Entretanto,
(A^(pq)-B^(pq))/(A^p-B^p)=A^(p(q-1))+A^(p(q-2))B^p+...+B^(p(q-1))A^(p(q-1))
2^(q-1)q, pois q=3, absurdo.
Abraços,
  Gugu


Acho q vc tem razão... não me ocorre como consertar,
exceto colocando uma restrição adicional. Acho que só
vale para A-B e c, primos entre si.   

[]´s


--- Carlos Gustavo Tamm de Araujo Moreira
[EMAIL PROTECTED] escreveu: 
Caro Demetrio,
No fim da sua explicacao, A-B nao pode ser uma
 potencia de y ? Nesse
 caso, todos os fatores primos de A-B sao fatores
 primos de y.A^(y-1), e eu
 nao entendi como voce conclui.
Abracos,
 Gugu
 
 
  --- Claudio Buffara [EMAIL PROTECTED]
 escreveu: 
 
  *
  
  10) Seja P = A^c - B^c,
  onde:
  A, B e c são inteiros e primos entre si,
  A - B  1, 
  c = n1*n2*...*ni*...nk ,
  (os ni são fatores primos distintos, ou seja, c
 tem
  k fatores
  primos distintos).
  
  Mostre que P é um número composto com, no mínimo,
  k+1
  fatores primos distintos.
  
  *
 
 Deixe eu colocar uma restrição adicional c = impar.
 
 
 Em primeiro lugar é fácil ver que todos os números 
 da forma A^ni - B^ni dividem P. 
 
 Portanto, um caminho seria mostrar que, dados 
 quaisquer números da forma S1 = A^x - B^x e 
 S2 = A^y - B^y, x e y primos entre si, S1 e S2 não 
 podem ser múltiplos, isto é, possuem algum fator 
 primo distinto entre si.
 
 
 
 1** 
 suponha A, B e x,y primos entre si. x e y primos 
 diferentes de 2 e x  y.
 
 Hipótese: se A^x - B^x tem fatores primos em 
 comum com A^y - B^y, estes fatores estão em A - B.
 
 
 Suponha que S1 = A^x - B^x contém um fator em 
 comum com S2 = A^y - B^y.  
 Seja 
 F1 = A^(x-y) * S2 = A^(x-y) * (A^y - B^y) = 
 A^x - [A^(x-y)*(B^y)]. 
 
 Naturalmente F1 contém o mesmo fator em comum 
 com S1 e S2, e portanto F1 - S1 o conterá também.
 
 F1 - S1 = B^x - [A^(x-y)*(B^y)] = 
 B^y * [B^(x-y) - A^(x-y)]. 
 
 Dado que A e B são primos entre si, o fator comum
 não pode estar em B^y, e portanto está em 
 B^(x-y) - A^(x-y). 
 
 Agora pode-se repetir o raciocínio para 
 B^(x-y) - A^(x-y) e A^y - B^y, 
 verificando qual dos dois expoentes é maior. 
 Suponhamos que x-y  y. Neste caso podemos provar 
 que o fator comum também está em B^(x-2y) -
 A^(x-2y). 
 Observe que, caso y  x-y provaríamos para o 
 expoente 2y - x. 
 
 Repetindo o raciocínio interativamente vamos chegar
 
 até o  expoente 1. Note que, como x e y são primos,
 
 a sequencia de expoentes decrescentes não
 coincidirá 
 com y.
 
 Por exemplo x = 19, y = 3.
 19 - 16 - 13 - 10 - 7 - 4 -1
 
 Por exemplo x = 17, y = 3. 
 17 - 14 - 11 - 8 - 5 - 2 -1   
 
 Por exemplo x = 19 y = 11.
 19 - 8 - 3 (11 - 8) - 5 (8 - 3) - 2 (5 - 3) -
 1
 (3 - 2)  
 
 
 2**
 S2 = A^y - B^y também possui ao menos um fator
 primo
 distinto da decomposição  em fatores primos de A -
 B.
 
 Note-se que 
 S2 = (A - B) * F3, onde 
 F3 = A^y-1 + (A^(y-2))*B + ... + B^y-1
 
 Portanto, se a hipótese estiver correta e S2 
 contiver ao menos um fator primo distinto de A - B,
 
 este fator estará em F3
 
 Note-se que F3 tem exatamente y termos. 
 Se a Hipótese estiver incorreta, isto é, se A - B 
 contiver todos os fatores primos de F3, então
 qualquer
 
 combinação linear do tipo  k1*(A - B) + k2*F3
 também 
 conterá todos estes fatores. 
 
 Esta idéia pode ser usada para reduzir-se os termos
 
 de F3 até um único termo que obrigatoriamente teria
 
 de conter todos os fatores primos.
 
 Por exemplo vamos considerar y = 3. 
 Neste caso F3 = A^2 +A*B +B^2
 
 F3 + A(A - B) = A^2 + A*B - A*B + B^2 = 2*A^2 + B^2
 2*A^2 + B^2 + (A + B)*(A - B) = 
 2*A^2 + B^2 + A^2 - B^2 = 3*A^2
 
 y = 5, F3=A^4 +A^3*B +A^2*B^2 +A*B^3 +B^4
 F3 + A^3*(A - B) + A*B^2*(A - B) = 
 2*A^4 +2*A^2*B^2 +B^4 = X1
 
 X1 + 2*A^2*(A + B)*(A - B) = X1 + 2*A^4 -
 2*A^2*B^2=
  4*A^4 + B^4
 
 4*A^4 + B^4 + A^4 - B^4 = 5*A^4
 
 Na verdade, caso (A - B) tenha todos os fatores
 primos
 de F3, é possível transformar F3 em outras
 expressões 
 que devem conter os mesmos fatores primos, através
 de 
 operações elementares, até uma expressão na forma
 
 y*A^y-1 (ou y*B^y-1). Mas vamos recordar que A,B e
 y 
 são primos entre si, portanto não é possível 
 que y*A^y-1 contenha os mesmos fatores primos de A
 -
 B.
 
 
 Em resumo, temos que 
 **1** - Se S1 e S2 possuem fatores primos em comum,
 
 estes fatores estão em A - B. 
 
 **2** - S1 e S2 possuem ao menos um fator primo não
 
 contido em A - B
 
 Logo S1 e S2 possuem ao menos um fator distinto
 entre
 si. 
 
 A extensão para c par é direta fazendo A^2 - B^2 
 = D - C
 
 []´s 
 
 
 
 
 

___
 
 Yahoo! Acesso Grátis - Instale o discador do Yahoo!
 agora. http://br.acesso.yahoo.com/ - Internet

Re: [obm-l] Problemas em aberto

2005-01-18 Por tôpico Carlos Gustavo Tamm de Araujo Moreira

Caros colegas:

Seguem abaixo problemas propostos na lista obm-l desde outubro de 2004 que
ainda nao foram resolvidos:

[]s,
Claudio.


28) Seja A = conjunto dos inteiros positivos livres de quadrados e que tem
um numero ímpar de fatores primos (distintos, claro!)
 
Assim, A contém todos os primos e seu menor elemento composto é 30 = 2*3*5.
 
Calcule o valor de Soma(n em A) 1/n^2.
 
Pode usar, sem demonstrar, que:
Soma(n em N) 1/n^2 = Pi^2/6   e   Soma(n em N) 1/n^4 = Pi^4/90.


Para s1, por fatoração única e convergência absoluta, 
soma(n=1 a infinito)(1/n^s)=produto(p primo)(1+1/p^s+1/p^(2s)+...)=
=produto(p primo)(1-1/p^s)^(-1) (essa é a popular fórmula de Euler).
Por razões análogas, se B = conjunto dos inteiros positivos livres de
quadrados e que têm um numero ímpar de fatores primos distintos, 
soma(n em B)(1/n^2)-soma(n em A)(1/n^2)=produto(p primo)(1-1/p^2)=
=(soma(n=1 a infinito)(1/n^2))^(-1)=6/pi^2, enquanto 
soma(n em B)(1/n^2)+soma(n em A)(1/n^2)=produto(p primo)(1+1/p^2)=
=produto(p primo)((1-1/p^4)/(1-1/p^2))=(pi^2/6)/(pi^4/90)=15/pi^2. Assim,
soma(n em A)(1/n^2)=(15/pi^2-6/pi^2)/2=9/(2.pi^2)=
=0,45594532639051997149745758444377

  Abraços,
Gugu
=
Instruções para entrar na lista, sair da lista e usar a lista em
http://www.mat.puc-rio.br/~nicolau/olimp/obm-l.html
=


Re: [obm-l] Problemas em aberto

2005-01-12 Por tôpico Carlos Gustavo Tamm de Araujo Moreira
   Caro Domingos,
   Note que a diferenca entre as duas somas e' soma(p=n,k=2)[n/p^k]=
soma(p=n)(n/p(p-1))=O(n) (aqui p percorre os primos), donde, como voce
mostrou que uma das somas e' assintoticamente n.loglog(n), a outra
automaticamente tambem e'. Note que voce so' usou ii), que e' mais facil de
provar que i) (veja o Hardy e Wright). Alem disso, para ver que os limites
sao infinitos, basta usar que a serie dos inversos dos primos da' infinito,
o que provavelmente ja' foi provado nesta lista (senao me avisem que eu
provo).
   Abracos,
 Gugu
 




20) Seja f: S = {2, 3, 4, 5, 6, ...} - S a função que leva um número n no
seu número de fatores primos. Por exemplo, f(6) = 2 e f(12) = f(8) = 3.
Quanto vale lim[n-inf] (f(2) + f(3) + ... + f(n))/(n-1)?



A resposta é bonitinha quando f não conta os primos repetidamente...
Vamos usar aquele princípio básico da combinatória que nos ensina a 
contar a mesma coisa de duas maneiras distintas :-)
Note que f(2) + ... + f(n) é equivalente a Soma_{p primo} Piso{n/p}.

Para verificar isso, disponha caixas indexadas por todos os primos = n. 
Olhe para cada elemento i = n, colocando um marcador na caixa p em cada 
primo p que é divisor de i. É evidente que o número de marcadores de 
todas as caixas é f(2) + ... + f(n). Por outro lado, quantos marcadores 
teremos numa caixa p? Cada inteiro i = n que é múltiplo de p colocou 
exatamente um marcador em tal caixa, logo temos Piso{n/p} marcadores em 
tal caixa.

Sempre que o termo p aparecer, fica implícito que p é primo, ok?
Como Piso{n/p}  n/p - 1, temos:
f(2) + ... + f(n)  Soma_{p  = n} [n/p - 1] = -pi(n) + n Soma_{p =n} 
[1/p],
onde pi(n) é a função que conta o número de primos = n.

Resultados clássicos nos grantem que:
i) pi(n) ~ n/(log n)
ii) Soma_{p =n} [1/p] ~ log log n, logo

Soma_{p  = n} [n/p - 1] ~ n[log log n - 1/(log n)].

Isso prova que lim_{n - oo} [f(2) + ... + f(n)]/n = oo. Como a nossa f 
tem valores sempre menores ou iguais aos valores da f do problema 
original, o resultado vale também para o problema original.

Se f conta os primos de forma repetida então a nossa contagem passa a ser
f(2) + ... + f(n) = Soma_{k = 0..oo} Soma_{p primo} Piso{n/p^k}.
Talvez alguém da lista tenha paciência de analisar assintoticamente o 
mostrinho...

[ ]'s
=
Instruções para entrar na lista, sair da lista e usar a lista em
http://www.mat.puc-rio.br/~nicolau/olimp/obm-l.html
=


=
Instruções para entrar na lista, sair da lista e usar a lista em
http://www.mat.puc-rio.br/~nicolau/olimp/obm-l.html
=


Re: [obm-l] Problemas em aberto - prob 10

2005-01-12 Por tôpico Carlos Gustavo Tamm de Araujo Moreira
   Caro Demetrio,
   No fim da sua explicacao, A-B nao pode ser uma potencia de y ? Nesse
caso, todos os fatores primos de A-B sao fatores primos de y.A^(y-1), e eu
nao entendi como voce conclui.
   Abracos,
Gugu


 --- Claudio Buffara [EMAIL PROTECTED]
escreveu: 

 *
 
 10) Seja P = A^c - B^c,
 onde:
 A, B e c são inteiros e primos entre si,
 A - B  1, 
 c = n1*n2*...*ni*...nk ,
 (os ni são fatores primos distintos, ou seja, c tem
 k fatores
 primos distintos).
 
 Mostre que P é um número composto com, no mínimo,
 k+1
 fatores primos distintos.
 
 *

Deixe eu colocar uma restrição adicional c = impar.


Em primeiro lugar é fácil ver que todos os números 
da forma A^ni - B^ni dividem P. 

Portanto, um caminho seria mostrar que, dados 
quaisquer números da forma S1 = A^x - B^x e 
S2 = A^y - B^y, x e y primos entre si, S1 e S2 não 
podem ser múltiplos, isto é, possuem algum fator 
primo distinto entre si.



1** 
suponha A, B e x,y primos entre si. x e y primos 
diferentes de 2 e x  y.

Hipótese: se A^x - B^x tem fatores primos em 
comum com A^y - B^y, estes fatores estão em A - B.


Suponha que S1 = A^x - B^x contém um fator em 
comum com S2 = A^y - B^y.  
Seja 
F1 = A^(x-y) * S2 = A^(x-y) * (A^y - B^y) = 
A^x - [A^(x-y)*(B^y)]. 

Naturalmente F1 contém o mesmo fator em comum 
com S1 e S2, e portanto F1 - S1 o conterá também.

F1 - S1 = B^x - [A^(x-y)*(B^y)] = 
B^y * [B^(x-y) - A^(x-y)]. 

Dado que A e B são primos entre si, o fator comum
não pode estar em B^y, e portanto está em 
B^(x-y) - A^(x-y). 

Agora pode-se repetir o raciocínio para 
B^(x-y) - A^(x-y) e A^y - B^y, 
verificando qual dos dois expoentes é maior. 
Suponhamos que x-y  y. Neste caso podemos provar 
que o fator comum também está em B^(x-2y) - A^(x-2y). 
Observe que, caso y  x-y provaríamos para o 
expoente 2y - x. 

Repetindo o raciocínio interativamente vamos chegar 
até o  expoente 1. Note que, como x e y são primos, 
a sequencia de expoentes decrescentes não coincidirá 
com y.

Por exemplo x = 19, y = 3.
19 - 16 - 13 - 10 - 7 - 4 -1

Por exemplo x = 17, y = 3. 
17 - 14 - 11 - 8 - 5 - 2 -1   

Por exemplo x = 19 y = 11.
19 - 8 - 3 (11 - 8) - 5 (8 - 3) - 2 (5 - 3) - 1
(3 - 2)  


2**
S2 = A^y - B^y também possui ao menos um fator primo
distinto da decomposição  em fatores primos de A - B.

Note-se que 
S2 = (A - B) * F3, onde 
F3 = A^y-1 + (A^(y-2))*B + ... + B^y-1

Portanto, se a hipótese estiver correta e S2 
contiver ao menos um fator primo distinto de A - B, 
este fator estará em F3

Note-se que F3 tem exatamente y termos. 
Se a Hipótese estiver incorreta, isto é, se A - B 
contiver todos os fatores primos de F3, então qualquer

combinação linear do tipo  k1*(A - B) + k2*F3 também 
conterá todos estes fatores. 

Esta idéia pode ser usada para reduzir-se os termos 
de F3 até um único termo que obrigatoriamente teria 
de conter todos os fatores primos.

Por exemplo vamos considerar y = 3. 
Neste caso F3 = A^2 +A*B +B^2

F3 + A(A - B) = A^2 + A*B - A*B + B^2 = 2*A^2 + B^2
2*A^2 + B^2 + (A + B)*(A - B) = 
2*A^2 + B^2 + A^2 - B^2 = 3*A^2

y = 5, F3=A^4 +A^3*B +A^2*B^2 +A*B^3 +B^4
F3 + A^3*(A - B) + A*B^2*(A - B) = 
2*A^4 +2*A^2*B^2 +B^4 = X1

X1 + 2*A^2*(A + B)*(A - B) = X1 + 2*A^4 - 2*A^2*B^2=
 4*A^4 + B^4

4*A^4 + B^4 + A^4 - B^4 = 5*A^4

Na verdade, caso (A - B) tenha todos os fatores primos
de F3, é possível transformar F3 em outras expressões 
que devem conter os mesmos fatores primos, através de 
operações elementares, até uma expressão na forma 
y*A^y-1 (ou y*B^y-1). Mas vamos recordar que A,B e y 
são primos entre si, portanto não é possível 
que y*A^y-1 contenha os mesmos fatores primos de A -
B.


Em resumo, temos que 
**1** - Se S1 e S2 possuem fatores primos em comum, 
estes fatores estão em A - B. 

**2** - S1 e S2 possuem ao menos um fator primo não 
contido em A - B

Logo S1 e S2 possuem ao menos um fator distinto entre
si. 

A extensão para c par é direta fazendo A^2 - B^2 
= D - C

[]´s 


   
   
   
___ 
Yahoo! Acesso Grátis - Instale o discador do Yahoo! agora. 
http://br.acesso.yahoo.com/ - Internet rápida e grátis
=
Instruções para entrar na lista, sair da lista e usar a lista em
http://www.mat.puc-rio.br/~nicolau/olimp/obm-l.html
=


=
Instruções para entrar na lista, sair da lista e usar a lista em
http://www.mat.puc-rio.br/~nicolau/olimp/obm-l.html
=


Re: [obm-l] Problemas em aberto

2005-01-12 Por tôpico Carlos Gustavo Tamm de Araujo Moreira
   Caro Domingos,
Você observou quef(2) + ... + f(n) é equivalente a Soma_{p primo} Piso{n/p},
mas isso é n.soma{p primo, p=n}(1/p) + O(n), donde isso dividido por n é 
soma{p primo, p=n}(1/p) + O(1), que tende a infinito pois a serie dos
inversos dos primos diverge.
Abraços,
  Gugu



Carlos Gustavo Tamm de Araujo Moreira wrote:

   Caro Domingos,
   Note que a diferenca entre as duas somas e' soma(p=n,k=2)[n/p^k]=
soma(p=n)(n/p(p-1))=O(n) (aqui p percorre os primos), donde, como voce
mostrou que uma das somas e' assintoticamente n.loglog(n)


Já imaginava que fosse dar a mesma coisa :-)

, a outra
automaticamente tambem e'. Note que voce so' usou ii), que e' mais facil de
provar que i) (veja o Hardy e Wright). Alem disso, para ver que os limites
sao infinitos, basta usar que a serie dos inversos dos primos da' infinito,
o que provavelmente ja' foi provado nesta lista (senao me avisem que eu
provo).


Você está dizendo que dá pra provar que o limite é +oo somente usando 
que a soma dos inversos dos primos diverge? Como seria a prova?

Abraços,

Domingos.
=
Instruções para entrar na lista, sair da lista e usar a lista em
http://www.mat.puc-rio.br/~nicolau/olimp/obm-l.html
=


=
Instruções para entrar na lista, sair da lista e usar a lista em
http://www.mat.puc-rio.br/~nicolau/olimp/obm-l.html
=


[obm-l] Jornadas de iniciacao cientifica no IMPA

2004-08-25 Por tôpico Carlos Gustavo Tamm de Araujo Moreira
   Caros amigos da lista,
   O Marcelo Viana, que esta' coordenando as jornadas de iniciação
científica no IMPA me pediu para dar publicidade ao evento, que o Domingos
mencionou. Na página abaixo ha' informações detalhadas: 
http://www.impa.br/Conferencias/Jornadas_IC/
Vou citar um trecho dela: 
As Jornadas de Iniciação Científica têm por objetivo reunir no IMPA os
melhores estudantes de iniciação científica em Matemática do Brasil, visando
proporcionar-lhes acesso ao ambiente de pesquisa do instituto e
incentivá-los a prosseguir estudos de pós-graduação e a carreira matemática. 

Propostas de apresentação de trabalho, incluindo monografia (mínimo 20
páginas) e histórico escolar, devem ser enviadas a icimpaimpa . br até
31/08/2004. Este prazo foi prorrogado até 30 de setembro de 2004. 

Os melhores trabalhos serão premiados e seus autores convidados a
apresentá-los no 25o Colóquio Brasileiro de Matemática, em 2005. 

PRÊMIO DE INICIAÇÃO CIENTÍFICA 

MINICURSOS 

PALESTRAS DE DIVULGAÇÃO 

APRESENTAÇÕES DE INICIAÇÃO CIENTÍFICA 

POSTERS 

 Abraços,
 Gugu


perfeito!

tem vários outros fatos interessantes que eu aprendi recentemente na 
minha iniciação científica.
estou escrevendo uma monografia pra participar da jornada de IC no IMPA.
se você (ou mais alguém) tiver interesse em ver, eu coloco na web.

[ ]'s
=
Instruções para entrar na lista, sair da lista e usar a lista em
http://www.mat.puc-rio.br/~nicolau/olimp/obm-l.html
=


=
Instruções para entrar na lista, sair da lista e usar a lista em
http://www.mat.puc-rio.br/~nicolau/olimp/obm-l.html
=


Re: [obm-l] questao simples do bartle

2004-08-11 Por tôpico Carlos Gustavo Tamm de Araujo Moreira
   E' costume usar a notacao A^B para o conjunto de todas as funcoes de B em
A. Quando A e' um corpo isso e' um espaco vetorial sobre A.
   Abracos,
Gugu

   

Pessoal, este problema tirado do capitulo 8 (The Topology of Cartesian 
Spaces) me parece ser simples por ser um dos primeiros do capitulo. Eu 
realmente não entendi o enunciado. Me desculpem pelo ingles, se alguem 
quiser eu traduzo o enunciado.

Let S = {1,2,...,p}, for some p E N. Show that the vector space R^S
is essentially the same as the space R^p

Gostaria que alguem por favor me explicasse o que exatamente ele quer no 
problema ou seja, acredito que basta explicar como se mostra que um 
espaço vetorial é essencialmente o mesmo que um outro e tambem o que é 
R^S. S é um conjunto...soa estranho, estou acosumado com R^2, R^3 e de 
associar a ideia de produto cartesiano mas como imaginar para R^S onde S 
é um conjunto de numeros naturais?

obrigado
-- 
Niski - http://www.linux.ime.usp.br/~niski

[upon losing the use of his right eye]
Now I will have less distraction
Leonhard Euler

=
Instruções para entrar na lista, sair da lista e usar a lista em
http://www.mat.puc-rio.br/~nicolau/olimp/obm-l.html
=


=
Instruções para entrar na lista, sair da lista e usar a lista em
http://www.mat.puc-rio.br/~nicolau/olimp/obm-l.html
=


Re: [obm-l] RSA

2004-04-26 Por tôpico Carlos Gustavo Tamm de Araujo Moreira
 Eu também certamente ja' escrevi muita bobagem nesta lista...
 Abracos,
  Gugu


On Wed, Apr 21, 2004 at 02:15:38PM -0300, Domingos Jr. wrote:
 Tenho certeza de que apresentando idéias você será levado a sério, mesmo que
 essas idéias estejam erradas. Todo mundo aqui já postou coisa errada, exceto
 o Nicolau, o Gugu, o... ah, esses caras não contam.

Se você olhar os arquivos verá que eu já publiquei uma quantidade enorme
de erros bobos. Alguns eu mesmo acho quando a mensagem volta, outros alguém
acha, outros provavelmente estão enterrados nos arquivos e nunca ninguém achou.

[]s, N.
=
Instruções para entrar na lista, sair da lista e usar a lista em
http://www.mat.puc-rio.br/~nicolau/olimp/obm-l.html
=


=
Instruções para entrar na lista, sair da lista e usar a lista em
http://www.mat.puc-rio.br/~nicolau/olimp/obm-l.html
=


Re: [obm-l] f''(t) + (f'(t))^2 - -infinito

2004-04-19 Por tôpico Carlos Gustavo Tamm de Araujo Moreira
   Acho que nao da' nao. Nao existe nem uma funcao g (nesse caso g=f')
derivavel de [0,infinito) em R com g'(t)+(g(t))^2  -1 para todo t grande:
nesse caso teriamos g'(t)  -1 para todo t grande, donde g(t) tende a
-infinito quando t- infinito, e logo, para t grande, g(t) e' negativo,
mas tambem teriamos g'(t)/g(t)^2  -1 para todo t grande,
ou seja, (1/g(t))'  1, donde 1/g(t) deve tender a +infinito, absurdo.
E' claro que podemos trocar -1 por qualquer coisa negativa...
   Abracos,
Gugu


on 13.04.04 17:39, Danilo notes at [EMAIL PROTECTED] wrote:


Pessoal ser=E1 que  algu=E9m pode me ajudar no problema abaixo ?

Construir uma fun=E7=E3o f  de classe  C^1  definida no intervalo  [  0 ,
infinito )   e tal que  w(t) =3D  (derivada segunda de f(t) )  +   ( derivada
primeira de f(t) ) ^ 2tende  a  menos infinito quando t  tende a mais
infinito =20

Abs.  =20


Oi, Danilo:

Ainda nao consegui achar uma funcao que satisfaca ao enunciado, mas achei
uma que chega perto:

f : [0,+infinito) - R, definida por:
f(0) =3D 0;
f(t) =3D sen(t^2)/t  se t  0

f eh continua em t =3D 0.

Para t  0, (f(t) - f(0))/(t - 0) =3D sen(t^2)/t^2 =3D=3D
f'(0+) =3D lim(t - 0+) (f(t) - f(0))/(t - 0) =3D 1

Alem disso, se t  0, f'(t) =3D 2*cos(t^2) - sen(t^2)/t^2  =3D=3D
lim(t - 0) f'(t) =3D 1 =3D=3D
f' eh continua para t =3D 0 =3D=3D
f eh de classe C^1.

t  0 =3D=3D f''(t) =3D -4*t*sen(t^2) -  2*cos(t^2)/t + 2*sen(t^2)/t^3

Assim, vemos que f'(t) eh limitada e que f''(t) atinge valores
arbitrariamente pequenos (ou seja, negativos e de modulo arbitrariamente
grande). Logo, f''(t) + (f'(t))^2 tambem atinge valores arbitrariamente
pequenos, apesar de nao tender a -infinito quando t tende a +infinito pois
sen(t^2) se anula para infinitos valores de t (mais precisamnte, para todo =
t
da forma raiz(k*Pi), com k inteiro positivo).


[]s,
Claudio.

=
Instruções para entrar na lista, sair da lista e usar a lista em
http://www.mat.puc-rio.br/~nicolau/olimp/obm-l.html
=


Re: [obm-l] Teoria dos Numeros

2004-04-19 Por tôpico Carlos Gustavo Tamm de Araujo Moreira
   Caros Claudio e Dirichlet,
   Bacana esse problema. 
   Vamos la': Dadas essas condicoes, se a pertence a X entao
b(n)=a^(n+2)+4.(a^n+a^(n-1)+...+a+1)=a^(n+2)+4.(a^(n+1)-1)/(a-1) pertence a
X para todo n, mas para todo primo q (digamos q=b(0)=a^2+4), b(n) (mod q) e'
periodica com periodo divisor de q-1, pelo pequeno teorema de Fermat, donde,
em particular, b(a^2+3) e' multiplo de b(0)=a^2+4, e como claramente
b(a^2+3)  a^2+4, b(a^2+3) nao e' primo, absurdo. Assim, X tem que ser vazio.
   Abracos,
 Gugu

Obs.: A primeira condicao e' um caso particular da segunda, nao ?
 

on 19.04.04 12:54, Ricardo Bittencourt at [EMAIL PROTECTED] wrote:

 Johann Peter Gustav Lejeune Dirichlet wrote:
 
 (a,b = ab+4 e a^2+4)
 Mas espere, 6m-1=m-1 mod 5.Logo (6m-1)(6n-1)+4=mn-m-n mod 5.Sera que da
 para arrancar alguem mod 5?
 Se mn=m+n mod 5 entao nao da primo
 
 Eu já consegui mostrar que todos os elementos do
 conjunto são da forma 30k+23, mas ainda não cheguei em
 nenhuma contradição.
 
Mais ainda: eles tem que ser da forma 60k + 53. Infelizmente, eu acho que
esse caminho nao vai levar aa solucao, mas pelo menos aumenta a minha
conviccao de que X eh vazio.

Alias, uma pergunta pro Dirichlet: de onde voce tirou esse problema?

[]s,
Claudio.


=
Instruções para entrar na lista, sair da lista e usar a lista em
http://www.mat.puc-rio.br/~nicolau/olimp/obm-l.html
=


=
Instruções para entrar na lista, sair da lista e usar a lista em
http://www.mat.puc-rio.br/~nicolau/olimp/obm-l.html
=


Re: [obm-l] metodo do pto fixo ou aproximacoes sucessivas ou substituicoes

2004-04-19 Por tôpico Carlos Gustavo Tamm de Araujo Moreira
   Caro Niski,
   Desculpe, so' agora vi a sua mensagem.
   Temos f'(x)=2+sen(x), que e' sempre maior que 0 entre 0 e pi/2, donde f
e' crescente, e logo, como f(0)=-1 e f(pi/2)=pi, f tem uma unica raiz entre
0 e pi/2. 
   Como phi'(x)=-sen(x)/2, que tem modulo sempre menor ou igual a 1/2, segue
que |x(i+2)-x(i+1)|=|phi(x(i+1))-phi(x(i))|=(1/2).|x(i+1)-x(i)|, pelo
teorema do valor médio. Assim, por inducao, para todo n natural temos
|x(n+1)-x(n)|=(1/2)^n.|x(1)-x(0)|, e dai' segue que  x(n) e' uma sequencia 
de Cauchy, e logo converge a um certo c real, e como x(n+1)=phi(x(n)) e phi
e' contínua segue que c=phi(c)=cos(c)/2, donde f(c)=2c-cos(c)=0. Como, se x
esta' em I entao cos(x)/2 tambem esta' em I, segue que c esta' em I, donde c
e' a única raiz x' de f(x) em I.
   Por outro lado eu não vejo porquê o fato de a seqüência x(n) convergir
para x(0)=0 e para x(0)=pi/2 garante que x(n) convirja para todo x(0) em I
sem consideraçoes adicionais.
   O fato crucial que usamos sobre a função phi e' que existe k com 0k1
tal que |phi(y)-phi(x)|=k.|y-x| para quaisquer x,y em I (i.e., phi e' uma
contração), além de phi(I) estar contido em I. Isso garante que o método das
aproximações sucessivas converge (para todo x(0) em I) para o único ponto
fixo de phi em I, i.e., o único c em I com phi(c)=c, sempre que I for um
intervalo fechado (e limitado).
   Abraços,
Gugu


Pessoal, gostaria da ajuda de voces para saber se eu estou pensando 
corretamente.

Seja a funcao f(x) = 2x - cos(x) que possui uma raiz x' em I = [0, pi/2]
é pedido para provar que para *qualquer* x[0] em I, a sequencia
x[i] = phi(x[i-1]) = cos(x[i-1])/2 , i = 1 converge para x'.

Bom pessoal no item anterior, eu provei que o processo iterativo
x[0] = 0, x[i] = phi(x[i-1]) = cos(x[i-1])/2 , i = 1 convergia para x' 
(Eu prove isto mostrando  que x' é ponto fixo de phi, que phi e phi' sao 
continuas em I, que o modulo da derivada de phi é menor que 1 e que se 
x[0] esta em I entao x[i] tb esta para qualquer i =1)

Eu vi que para x[0] = 0, o processo convergia pois 0 é o extremo de I 
mais proximo da raiz (tomei o ponto medio de I...e etc) Então para 
provar que para qualquer x[0] em I a sequencia converge, não basta eu 
mostrar que para x[0] = pi/2 a sequencia tambem converge? Se for isso é 
facil pq phi(pi/2) = 0, assim x[1] = 0 e como eu ja vi que para x[0] = 0 
a sequencia convege, obviamente para x[1] = 0 ela tambem convergeria.
Isso esta certo?

Outra pergunta, de modo geral, dado que x[i] = phi(x[i-1]) = uma funcao 
qualquer  com i = 1, existe algum bom caminho para ver se existe um 
x[0] em I tal que sequencia convirja para x'?

Obrigado a todos!

-- 
Niski - http://www.linux.ime.usp.br/~niski

When we ask advice, we are usually looking for an accomplice.
Joseph Louis LaGrange

=
Instruções para entrar na lista, sair da lista e usar a lista em
http://www.mat.puc-rio.br/~nicolau/olimp/obm-l.html
=

=
Instruções para entrar na lista, sair da lista e usar a lista em
http://www.mat.puc-rio.br/~nicolau/olimp/obm-l.html
=


Re: [obm-l] Somatorios de k^6 e de k^8

2004-04-06 Por tôpico Carlos Gustavo Tamm de Araujo Moreira

On Wed, Mar 31, 2004 at 12:10:11AM -0300, Rafael wrote:
 É verdade, Nicolau, para o proposto, não houve qualquer erro. Entretanto,
 lendo com mais atenção, surgiram-me duas perguntas:
 
 1) Qual é a vantagem de se calcular a soma até n (exclusive)?

Os números de Bernoulli usuais são os que aparecem na minha outra mensagem.
Ou seja, a fórmula fica mais coerente com a definição usual de B_n
com a soma até n *exclusive*.

A fórmula fica mais simples mesmo se tomarmos 

f_m(n) = (0^m + 2*1^m + 2*2^m + ... + 2*(n-1)^m + n^m)/2

Neste caso o polinômio fica sendo par ou ímpar dependendo 
se n for ímpar ou par, respectivamente.
 
 2) Sobre a definição proposta: S_m(n) = 0^m + 1^m + ... + (n-1)^m, em sua
 mensagem anterior, é considerado que S_0(n) = n. Isso só é verdade, de
 acordo com a definição, se 0^0 = 1, o que é uma convenção. Lembro-me de já
 ter lido que nem sempre é possível afirmar isso, ou melhor, que somente uma
 função analítica permite a conclusão, em geral, de que 0^0 = 1. O mesmo
 seria válido para: 0/0, 0*oo, oo/oo, 1^oo, oo - oo. Você poderia explicar e
 dar detalhes sobre isso?

Você pode dizer que 0^0 = 1 é uma convenção, como você pode dizer que 0! = 1
é uma convenção. Para mim não é não, é um caso particular tanto da definição
combinatória de a^b para a e b naturais (a^b é o número de funções de B em A
onde |A| = a e |B| = b) quanto da definição recursiva (a^0 = 1,
a^(b+1) = a*(a^b)). Mas o fato é que 0^0 = 1 é uma convenção universal,
tanto quanto eu saiba.

Você parece estar falando em limites em parte do seu texto. Não é verdade
que se lim_{x - 0} f(x) = 0 e lim_{x - 0} g(x) = 0 então sempre
lim_{x - 0} ((f(x))^(g(x))) = 1, nem se f e g forem analíticas.
   
   Bem, se f e g sao analiticas nao-constantes numa vizinhanca de 0 e se anulam em 0
(e f e' nao-negativa numa vizinhanca de 0) entao vale 
lim_{x - 0} ((f(x))^(g(x))) = 1.
   Abracos,
Gugu



[]s, N.
=
Instruções para entrar na lista, sair da lista e usar a lista em
http://www.mat.puc-rio.br/~nicolau/olimp/obm-l.html
=


=
Instruções para entrar na lista, sair da lista e usar a lista em
http://www.mat.puc-rio.br/~nicolau/olimp/obm-l.html
=


Re: [obm-l] PROBLEMA DO JORNALEIRO!

2004-03-26 Por tôpico Carlos Gustavo Tamm de Araujo Moreira
   Caro Jorge Luis,
   Se ele comprar n jornais, o valor esperado do lucro e' 
soma(k=1 a n)(0,01*0,4*k)+0,01*(100-n)*0,4*n-0,15*n=
0,004*(n(n+1)/2+n*(100-n))-0,15*n=-0,002*n^2+0,252*n, que e' maximo para
n=0,252/0,004=63. Assim, ele deve adquirir 63 jornais (a menos que eu tenha
errado a conta...).
   Abracos,
Gugu


Oi, Pessoal! Alguém poderia me ajudar no problema abaixo. Grato pela atenção!


Suponhamos que um vendedor de jornais compre jornais no começo do dia a 15 
centavos cada um e os venda a 40 centavos cada. Se ele compra mais jornais do 
que pode vender, a sobra deve ser descartada como perda total. Suponha que haja 
uma mesma probabilidade 0,01 de ter uma demanda de um jornal, dois, 
três,...,100 jornais. Quantos jornais ele deve adquirir no começo do dia a fim 
de maximizar seu lucro esperado?


Abraços!



WebMail UNIFOR - http://www.unifor.br
=
Instruções para entrar na lista, sair da lista e usar a lista em
http://www.mat.puc-rio.br/~nicolau/olimp/obm-l.html
=

=
Instruções para entrar na lista, sair da lista e usar a lista em
http://www.mat.puc-rio.br/~nicolau/olimp/obm-l.html
=


Re: [obm-l] limite de sin(n)^n

2003-11-02 Por tôpico Carlos Gustavo Tamm de Araujo Moreira
Caros Salvador et al,
Essa serie converge sim, mas nao e' muito facil provar. A minha solucao
usa o fato de pi ser diofantino (o que tem a ver com a linha que o Salvador
sugeriu - a ideia principal e' que aproximacoes racionais boas nao sao
frequentes demais): de fato, para todo racional p/q com q suficientemente
grande, |pi-p/q|1/q^24, como esta' provado no livro Pi and the AGM : a
study in analytic number theory and computational complexity, de Jonathan
M. Borwein e Peter B. Borwein (parece que o primeiro resultado desse tipo se 
deve a Mahler, com 1/q^42 em vez de 1/q^24, e o melhor conhecido e' algo
como 1/q^14,65, provado por Chudnovsky e Chudnovsky). Vamos la':
   Primeiro note que se n e' grande e |2/pi-m/n|  d  1/n entao
|pi-2n/m|  d.n.pi/m  5d, donde 5d  1/m^24, e logo d  1/n^25. Sabemos,
pelo teorema de Dirichlet, que para todo inteiro positivo N, existem q com
1=q=N e p inteiros com |2.q/pi-p|1/N. O fato acima mostra que devemos ter
necessarimente q  N^(1/24). Vamos agora usar argumentos da prova de que se
x e' irracional entao nx(mod 1) e' uniformemente distribuida para tentar
estimar o tamanho do conjunto dos n tais que 2n/pi esta' perigosamente perto
de algum inteiro (como o Salvador sugeriu): 
   Dado k inteiro, existem inteiros positivos p e q=2^(k/72) tais que 
|2.q/pi-p|1/2^(k/72) (e, alem disso, |2.q/pi-p|  1/q^24=1/2^(k/3)). Assim, 
se n percorre os termos de uma PA de razao q contida em [1,2^k], 
n.2/pi(mod 1) percorre uma PA (mod 1, i.e., no circulo) com razao entre 
1/2^(k/3) e 1/2^(k/72). Como o numero de termos de uma tal PA de valores de
n e' essencialmente n/q=2^(71k/72)1/2^(k/3), n.2/pi (mod 1) da' no maximo
(2^k/q)/2^(k/72) voltas no circulo, e portanto o numero de valores de n em
uma tal PA com |2.n/pi-r|1/2^(k/3) para algum inteiro r e' no maximo
2.(2^k/q)/2^(k/72) (pois em cada volta passamos no maximo duas vezes perto
do 0), donde, como temos q tais PA's (uma para cada classe de conguencia
mod. q), o numero total de tais n com n=2^k e' limitado por 2.2^(71k/72) 
 2^(72k/73). Assim, em particular, existem no maximo 2^(72k/73) valores de
n com 2^(k-1)=n=2^k tais que |2.n/pi-r|1/2^(k/3). A soma de
(1/n).((2+sen(n))/3)^n para esses n e' limitada pela soma de 1/n para 
esses n, que e' no maximo 2^(72k/73).1/2^(k-1)=2/2^(k/73)  1/2^(k/74). 
   Para os outros valores de n entre 2^(k-1) e 2^k, temos 
|n-r.pi/2|=(pi/2)/2^(k/3)1/2^(k/3), e logo, 
como sen(pi/2+d) = 1-d^2/3 se d e' suficientemente pequeno,
(1/n).((2+sen(n))/3)^n=(1/2^(k-1)).(1-1/(3.2^(2k/3)))^(2^(k-1))  
 (1/2^(k-1)).e^(-2^(k-1)/(3.2^(2k/3))) = (1/2^(k-1)).e^(-2^(k/3))/6), e
logo sua soma para esses n e' no maximo 2^(k-1).(1/2^(k-1)).e^(-2^(k/3))/6) =
= e^(-2^(k/3))/6)  1/2^k. 
   Assim, a soma os termos da nossa serie com 2^(k-1)=n  2^k e' no maximo
1/2^(k/74)+1/2^k  1/2^(k/75), cuja serie em k converge, o que prova a
convergencia da serie original.
   Abracos,
 Gugu




Caro Gugu,

Desculpe incomoda-lo com isso, mas queria saber se voce pensou sobre a
convergencia daquela soma, onde aparecia

((2+sin(x))/3)^n


Eu estou pensando em algo um pouquinho mais simples,


soma de |sin(n)|^n  e acho que isto esta ligado ao
--
n

seguinte problema:


Pegue todos os pares (n,m) tais que |2/pi*n-m|1/(n^0.5)  (*)

O que se pode dizer da soma do inverso dos n que satisfazem (*) ?



Desculpe se te incomodo com isso.



Um abraco,

Salvador




Oi Cláudio!

Não sei a resposta. Eu deveria ter dito mais sobre o problema quando fiz a
pergunta. Pelo que ouvi dizer, este é um problema que um professor copiou
mal de um livro e propôs a seus alunos. (o problema original era trivial)
Ele tentou e não conseguiu resolver o problema. O problema já passou por
muita gente, segundo me contaram até numa das edições da revista AMM, e
ainda não encontraram a solução.

A mim, parece que a série converge. Eu propus na lista por que sei que você,
e outros, iriam se interessar, já que ela parece ter tudo a ver com a
questão de seqüências equidistribuídas.

Ele não me parece tão difícil, o que você acha?

Abraço, Duda.


From: Claudio Buffara [EMAIL PROTECTED]
 on 20.10.03 01:36, Eduardo Casagrande Stabel at [EMAIL PROTECTED]
wrote:

  Oi Pessoal!
 
  E quanto à SOMA{ (1/n)*[(2 + sen(n))/3]^n , n=1, 2, ... } ?
 
  Abraço, Duda.
=
Instruções para entrar na lista, sair da lista e usar a lista em
http://www.mat.puc-rio.br/~nicolau/olimp/obm-l.html
=


Re: [obm-l] Particao de R

2003-10-02 Por tôpico Carlos Gustavo Tamm de Araujo Moreira
Uma bem classica e' A={numeros diofantinos} e B=Q U {numeros de Liouville}. 
Um numero irracional x e' de Liouville se |x-p/q|1/q^n tem solucao racional
p/q com q=2 para todo n natural, e e' diofantino caso contrario.
Abracos,
 Gugu


Oi, pessoal:

Alguem saberia exibir uma particao de R (conjunto dos reais) em dois
conjuntos A e B tais que, para todo intervalo aberto I, A inter I e B inter
I sao nao-enumeraveis?

Um abraco,
Claudio.

=
Instruções para entrar na lista, sair da lista e usar a lista em
http://www.mat.puc-rio.br/~nicolau/olimp/obm-l.html
=

=
Instruções para entrar na lista, sair da lista e usar a lista em
http://www.mat.puc-rio.br/~nicolau/olimp/obm-l.html
=


Re: [obm-l] Sequência Equidistribuída

2003-09-19 Por tôpico Carlos Gustavo Tamm de Araujo Moreira
  Oi Claudio,
  Nao e' nao. De fato, n (mod 2.pi) e' uniformemente distribuida em
[0,2.pi], e isso implica que cos(n) e' distribuido em [-1,1] de acordo com a
imagem da medida de Lebesgue normalizada em [0,pi] pela funcao cos(x), ou
seja, a probabilidade de termos -1=a=cos(n)=b=1 e'
(arccos(a)-arccos(b))/pi. Falando nisso ha' um teorema segundo o qual se
P(x) e' um polinomio com algum coeficiente nao-constante irracional entao
({P(n)}),n natural e' uniformemente distribuida em [0,1]. Em particular n^2
(mod 2.pi) e' uniformemente distribuida em [0,2.pi] (compare com
(n/2.pi)^2, que e' uniformemente distribuida em [0,1]). Isso implicva a sua
conjectura sobre essa sequencia com cos(n^2), nao ?
  Abracos,
   Gugu
  

Oi, Salvador:

Esse teorema é bem interessante.

Acho que ele está relacionado ao seguinte fato:
Na sequência x(n) = 2^n, a probabilidade do algarismo da esquerda da
representação decimal de x(n) ser igual a k (1=k=9) é igual a
log_10((k+1)/k). Ou seja, nessa sequência, pouco mais de 30% dos termos
começam com o algarismo 1. Por outro lado, menos de 5% deles começam com
9. Essa é a tal lei de Benford.

Claro que, como log_10(2) é irracional, a sequência y(n) = log_10(x(n)) mod
1 = n*log_10(2) mod 1 é equidistribuida.



Será que x(n) = cos(n) é equidistribuída?

Um abraço,
Claudio.

- Original Message -
From: Salvador Addas Zanata [EMAIL PROTECTED]
To: Lista OBM [EMAIL PROTECTED]
Sent: Friday, September 19, 2003 11:56 AM
Subject: Re: [obm-l] Valores de aderencia



 Oi amigos,

 Existe um troco chamado teorema da equidistribuicao de Weyl, que diz o
 seguinte: Se uma sequencia a_n em [0,1] por exemplo, satisfizer uma serie
 de relacoes, entao ela eh equidistribuida.

 Por exemplo, a_n=n.w mod 1, com w irracional eh equidistribuida, o que
 quer dizer que ela se espalha uniformemente em [0,1].

 Se alguem quiser, posso dar as condicoes precisas que a seq. deve
 satisfazer. Eh claro que dizer que uma seq. eh equidistribuida eh muito
 mais forte que dizer que ela eh densa.


 Um abraco,

 Salvador




 On Thu, 18 Sep 2003, Claudio Buffara wrote:

  Oi, pessoal:
 
  Sabemos que x(n) = cos(n) tem subsequencias que convergem para qualquer
  ponto no intervalo [-1,1].
 
  Pergunta:
  O que eh que a funcao cosseno tem de especial para que isso aconteca, ou
  seja, que propriedade(s) uma funcao real precisa ter para gerar
sequencias
  com subsequencias convergindo para qualquer ponto da imagem da funcao?
 
  Sobre a funcao cosseno eu consigo pensar em 4 coisas:
  1) Ela eh limitada;
  2) Ela eh periodica de periodo irracional;
  3) Ela eh continua;
  4) Ela eh uma sobrejecao em [-1,1].
 
  O meu chute eh que (1) e (3) sao irrelevantes, que (2) eh uma condicao
  suficiente mas nao necessaria, pois acho que y(n) = cos(n^2) tambem tem
  subsequencias convergindo para qualquer ponto de [-1,1], e que (4) eh
uma
  condicao necessaria mas nao suficiente, pois f(x) = sen(pi*x) tambem eh
uma
  sobrejecao em [-1,1] mas z(n) = sen(pi*n) eh constante e igual a zero.
 
  Como sempre, qualquer ajuda serah bem-vinda.
 
  Um abraco,
  Claudio.
 
 
=
  Instruções para entrar na lista, sair da lista e usar a lista em
  http://www.mat.puc-rio.br/~nicolau/olimp/obm-l.html
 
=
 

 =
 Instruções para entrar na lista, sair da lista e usar a lista em
 http://www.mat.puc-rio.br/~nicolau/olimp/obm-l.html
 =

=
Instruções para entrar na lista, sair da lista e usar a lista em
http://www.mat.puc-rio.br/~nicolau/olimp/obm-l.html
=

=
Instruções para entrar na lista, sair da lista e usar a lista em
http://www.mat.puc-rio.br/~nicolau/olimp/obm-l.html
=


Re: [obm-l] RE: [obm-l] Questão de Análise

2003-09-16 Por tôpico Carlos Gustavo Tamm de Araujo Moreira
   Oi Artur,

Oi Duda!
Se X_1,... e X_n estao em P(A), entao cada X_i esta contido s em Uniao =
X_i.
Pelas condicoes dadas, segue-se que F(Uniao X_i) estah contido em cada =
um
dos F(X_i). Logo, F(Uniao X_i) estah contido em Interseccao F(X_i). Alem
disto, temos que Interseccao F(X_i) esta contido em cada um dos F(x_i), =
o
que acarreta que cada F(F(X_i)) =3D X_i esteja contido em F(Interseccao
F(X_i)). Prosseguindo, temos que Uniao X_i esta contido em F(Interseccao
F(X_i), o que implica que F(F(Interseccao F(X_i)) =3D Interseccao F(X_i)
esteja contido em F(Uniao X_i), Assim  concluimos que F(Uni=E3o
X_i) =3D Interse=E7=E3o F(X_i) --Ufa! .Interessante observar que isto eh =
valido
mesmo para subcolecoes nao numeraveis de P(A).=20

Agora, temos que Interseccao X_i estah contido em cada X_i, de modo que =
cada
F(X_i) estah contido em F(Interseccao X_i). Logo, Uniao F(X_i) esta =
contido
em F(Interseccao X_i). Alem disto, cada F(X_i) estah contido  em Uniao
F(x_i), de modo que F(Uniao F(X_i)) esta contido em cada um dos =
F(F(X_i)) =3D
X_i. Segue-se que F(Uniao F(X_i)) esta contido em Interseccao (X_i), do =
que
concluimos que F(Interseccao X_i) esta contido em F(F(Uniao F(X_i))) =3D =
Uniao
F(X_i). E assim, segue-se que F(Interse=E7=E3o X_i) =3D Uni=E3o F(X_i), =
completando
a prova. Verificamos de novo que isto eh valido mesmo para subcolecoes =
nao
numeraveis de P(A).

Das condicoes dadas segue-se que F eh bijetora. Sendo 0 o conjunto =
vazio,
temos para todo X de P(A) que 0 estah contido em X e que, portanto, F(X)
esta contido em F(0). Mas como F eh bijetora, para algum X temos F(X) =
=3D A,
de modo que F(0) =3D A. Logo, F(A) =3D F(F(0)) =3D 0. Isto nao prova, =
mas
desconfio que F eh a funcao complemento.

  Acho que nao e' sempre assim nao: seja f uma involucao de A e F(X)={f(x),
x no complementar de A}. Entao F satisfaz as condicoes do enunciado.
  Abracos,
   Gugu



Um abraco!
Artur=20

 Ol=E1 Pessoal!
=20
 Estou resolvendo o livro do Elon de An=E1lise e h=E1 um exerc=EDcio =
que n=E3o
 estou
 conseguindo resolver.
=20
 Seja A um conjunto e P(A) o conjunto das partes de A. Considere uma =
fun=E7=E3o
 f:P(A)-P(A) que satisfaz as propriedades: se X est=E1 contido em Y =
(ambos
 de
 P(A)) ent=E3o F(Y) est=E1 contido em F(X); e F(F(X)) =3D X. Mostrar =
que F(Uni=E3o
 X_i) =3D Interse=E7=E3o F(X_i) e tamb=E9m F(Interse=E7=E3o X_i) =3D =
Uni=E3o F(X_i).
=20
 Uma fun=E7=E3o que satisfaz essas condi=E7=F5es =E9 F(X) =3D =
Complementar X.
=20


--=_NextPart_000_0006_01C37BF4.472A5860
Content-Type: application/ms-tnef;
   name=winmail.dat
Content-Transfer-Encoding: base64
Content-Disposition: attachment;
   filename=winmail.dat

eJ8+IjsEAQaQCAAEAAABAAEAAQeQBgAI5AQAAADoAAEIgAcAGElQTS5NaWNy
b3NvZnQgTWFpbC5Ob3RlADEIAQ2ABAACAgACAAEGAAcAAQEGgAMADgAAANMHCQAQ
AAEALQIAIwEBA5AGAMQJAAAuCwACAAELACMAAAMAJgAACwApAAAD
AC4AAAIBMQABGACIHK2dGmnOQL2OqXUAnoiTJDwmAAMANgAAHgBwAAEA
AAAbW29ibS1sXSBRdWVzdONvIGRlIEFu4Wxpc2UAAAIBcQABFgHDfA0wO+z0EVOo
XU0Vus68Qn6SA7cAAAIBHQwBGQAAAFNNVFA6QVJUVVJAT1BFTkRGLkNPTS5CUgALAAEO
AEAABg4AriNJDXzDAQIBCg4BGACIHK2dGmnOQL2OqXUAnoiTwoMAFA4A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Re: [obm-l] F(F(x)) = x e combinatoria

2003-09-16 Por tôpico Carlos Gustavo Tamm de Araujo Moreira
  Oi Claudio,  
  Vamos la':


Oi, Artur e Duda:

Esse problema do livro do Elon me sugeriu dois problemas de combinatoria.
 
1) Seja A um conjunto qualquer e F: A - A uma funcao tal que, para todo x
em A vale F(F(x)) = x. F eh chamada uma involucao em A. Eh facil ver que
toda involucao em A eh uma bijecao.

Se A for finito e |A| = n, entao existem n! bijecoes de A em A.

Pergunta: Qual o numero de involucoes em A?

   Seja a_n esse numero. Entao a_(n+1)=a_n+n.a_(n-1) (veja para onde vai o
n+1: se fica fixo caimos no caso anterior, senao escolhemos um dos outros n
elementos para permutar com ele e caimos no caso n-1 para o resto).
Considerando f(x)=serie(a_n.x^n/n!), isso da'  f'(x)=f(x)+x.f(x), e olhando
para os primeiros valores, temos f(x)=e^(x+x^2/2).  Se eu nao errei as
contas, f(n) deve ser assintoticamente algo como raiz(2).(n/e)^(n/2).
Veja A85 na encyclopedia of integer sequences, ou, mais diretamente,
http://www.research.att.com/cgi-bin/access.cgi/as/njas/sequences/eismum.cgi
   
*

2) Seja A um conjunto finito com |A| = n (e portanto |P(A)| = 2^n).
Seja F: P(A) - P(A) uma funcao tal que, para todos X e Y em P(A):
F(F(X)) = X   
e   
X contido em Y == F(Y) contido em F(X).

Pergunta: Quantas funcoes de P(A) em P(A) existem com essas duas
propriedades?

   Primeiro vou caracterizar tais f. Elas devem ser do tipo que eu descrevi
na minha mnensagem anterior (mesmo no caso infinito): deve existir uma
involucao f de X tal que F(X)={f(y), y nao esta' em X}. Assim, a resposta e'
o mesmo a_n do item anterior. Para provar isso, note primeiro que dado a em
A existe f(a) em A com F({a})=A\{f(a)}. De fato, se F({a}) esta' contido em
A\{c,d}, {a} contem F(A\{c,d}), que contem estritamente F(A\{c}), que contem
estritamente F(A)=vazio (de fato F e' bijecao), absurdo. Como F({a}) esta'
contido estritamente em F({}), segue nossa afirmacao. Como F(X) e' a
intersecao dos F({x}) para x em X, basta provar que f e' involucao, mas se
f(f(a)) nao e' a, como F({f(a)})=A\{f(f(a))}, temos que a pertence a
F({f(a)}), mas entao A\{f(a)}=F({a}) conteria F(F({f(a)}))={f(a)}, absurdo.

   Abracos,
 Gugu
 



Um abraco,
Claudio.



=
Instruções para entrar na lista, sair da lista e usar a lista em
http://www.mat.puc-rio.br/~nicolau/olimp/obm-l.html
=

=
Instruções para entrar na lista, sair da lista e usar a lista em
http://www.mat.puc-rio.br/~nicolau/olimp/obm-l.html
=


Re: [obm-l] Sequencias convergentes

2003-09-16 Por tôpico Carlos Gustavo Tamm de Araujo Moreira
   Bem, voce quase ja' provou isso: a monotonicidade mostra que a(n)
converge a a e b(b) a b com a=1=b. Para n grande trocamos um par perto de
(a,b) por um par perto de (a.b,b) ou por um par perto de (a,a.b). Assim,
devemos ter a.b=a, donde b=1 ou a.b=b, donde a=1. Assim, a=1 ou b=1. Se a=1
e b1, para n grande acabamos caindo sempre no segundo caso, mas ai
a(n+1)=a(n), e logo a(n) fica constante e menor que 1 a partir desse ponto,
e logo converge a essa constante menor que 1, contradizendo a=1. O outro
caso (a1 e b=1) e' analogo, e portanto devemos ter a=b=1.
   Abracos,
Gugu
  

Oi, pessoal:

Noutro dia o Marcio Cohen deu uma bela demonstracao, usando fracoes
continuas, de que o conjunto {n*a - m; a irracional positivo, m,n: inteiros
positivos} eh denso em R.

A demonstracao do Marcio pode ser adaptada para se provar o seguinte:

Sejam a, b reais tais que 0  a  1  b e a^m*b^n  1, para quaisquer m, n
inteiros positivos.
Sejam as sequencias (a(k)) e (b(k)) definidas por:
a(1) = a; b(1) = b
Para n = 1:
a(n)*b(n)  1 == a(n+1) = a(n)*b(n)  e  b(n+1) = b(n);
a(n)*b(n)  1 == a(n+1) = a(n)  e  b(n+1) = a(n)*b(n).
Prove que: lim a(n) = lim b(n) = 1.

Levando em conta que a^m*b^n  1 para quaisquer inteiros positivos m e n se
e somente se log_b(a) eh irracional, nos caimos no problema anterior e
acabou...

Eu gostaria de ver uma demonstracao mais elementar deste resultado.

Por exemplo, eh facil ver que a(n) e b(n) sao monotonas e limitadas. Assim,
falta provar que sup(a(n)) = inf(b(n)) = 1.

Qulquer dica serah bem-vinda.

Um abraco,
Claudio.

=
Instruções para entrar na lista, sair da lista e usar a lista em
http://www.mat.puc-rio.br/~nicolau/olimp/obm-l.html
=

=
Instruções para entrar na lista, sair da lista e usar a lista em
http://www.mat.puc-rio.br/~nicolau/olimp/obm-l.html
=


[obm-l] Para os cariocas (treinamento)

2003-08-29 Por tôpico Carlos Gustavo Tamm de Araujo Moreira
   Caros colegas,
   No segundo semestre havera' reunioes semanais de treinamento olimpico no
IMPA, alternadamente as segundas e as tercas feiras. Na proxima terca, 2 de
setembro, a reuniao sera' comandada pelo Marcio Cohen. A reuniao seguinte
sera' na segunda, 8 de setembro, comandada pelo Luciano.
   Abracos,
Gugu
=
Instruções para entrar na lista, sair da lista e usar a lista em
http://www.mat.puc-rio.br/~nicolau/olimp/obm-l.html
=


Re: [obm-l] News from IMC!!!!!!! / Dirichlet

2003-07-28 Por tôpico Carlos Gustavo Tamm de Araujo Moreira
  Caros Paulo e Okakamo,
   Nao sei a qual mensagem do Dirichlet voces se referem, mas a que eu vi (e
da qual reproduzo abaixo uma parte) sobre o problema 6 menciona
explicitamente que e' do Tengan a solucao que ele copia. Se for assim acho
que voces estao exagerando um pouco...
   Abracos,
Gugu


Date: Mon, 21 Jul 2003 15:43:53 -0300 (ART)
From: [** iso-8859-1 charset **] Johann Peter Gustav Lejeune
Dirichlet[EMAIL PROTECTED]
Subject: Re: [obm-l] E-mail do Tengan sobre o IMO 6
To: [EMAIL PROTECTED]

---Executing: shownonascii
This message contains non-ASCII text, but the iso-8859-1 font
has apparently not yet been installed on this machine.
(There is no directory named /usr/local/lib/fonts.)
What follows may be partially unreadable, but the English (ASCII) parts
should still be readable.

Legal,esta ideia e parecida com a minha.Mas uma
coisa:alguem pode ser mais explicito nesta parte
de olhar a raiz primitiva de q?E como e que a
ordem e exatamente p?

 --- edmilson motta [EMAIL PROTECTED] escreveu:
 Ei pessoal,
 
 voces notaram que o problema 6 da prova e' uma
 versao simplificada de um problema que eu e o
 Ed
 mandamos em uma das listas de treinamento do
 ano
 passado?  O problema da lista era algo assim:
 
 Sejam a,r1 e p um primo.  Prove que existe um
 primo q tal que (a mod q) tem ordem p^r.

Ola Prof Okakamo e demais
colegas desta lista ... OBM-L,

Protesto !

Eu, em OFF, estava dizendo ao Gugu que aguardassemos mais um tempo antes de 
enviar solucoes das questoes IMO, para que os demais participantes, 
sobretudo os que se preparam para as olimpiadas, tivessem oportunidade de 
enviar as suas solucoes ou parte delas. Eu francamente acreditava e continuo 
acreditando, que muitos membros serios, estudantes, tem totais condicoes de 
resolver qualquer das questoes, alem da 4 ( que e trivial ).

Como nao conhecia o ridiculo plagio do Dirichlet, disse : Olha ai Gugu, viu 
a solucao do Dirichlet ? Isso e uma prova de que devemos aguardar um pouco 
mais !

Agora o Prof Okakamo esclarece tudo ... Que feio ! O Plagiario e o ladrao da 
inteligencia ! Isso e um crime que consta na lei do direito autoral e cabe 
denuncia ao Ministerio Publico Federal : Eu sou do Ministerio Publico 
Federal e uma das minhas funcoes e justamente combater crimes ciberneticos ! 
Nao e muito mais facil mergulhar dentro de si mesmo e escutar o nosso Prof 
interno ? A solucao vem normalmente !

Em sintese, se o Prof Okakamo nao esta enganado, eu me associo a sua 
indignacao e repudio atitudes tao baixas quanto esta !

Um Abraco a Todos !
Paulo Santa Rita
1,1737,270703




From: okakamo kokobongo [EMAIL PROTECTED]
Reply-To: [EMAIL PROTECTED]
To: [EMAIL PROTECTED]
Subject: [obm-l] News from IMC!!!
Date: Sun, 27 Jul 2003 13:56:23 -0300 (ART)

   Oi pessoal pessoal da lista,

  Finalmente consegui internet aqui na Romenia (estou
com os alunos da IMC,
junto com o Luciano), tirando duvidas das solucoes dos
meus problemas
propostos (eu e o Luciano estamos tentando explicar a
ideia de alguns
problemas para o pessoal da banca).


  Gostaria de fazer um pequeno protesto:
O participante da lista Dirichlet estah plagiando
ideias de meu caro
e estimado pupilo Eduardo Tengan, que resolveu o
problema 6 da IMO (de uma
forma elegante) e o participante simplesmente copiou e
colou sem a minima
vergonha. Considero essa atitude desprezivel,
ridicula. E ja nao eh a
primeira vez que isso acontece. Ficar se gabando de
que uma coisa que
nao se fez eh simplesmente estupido. Seja honesto
(inclusive no nome)
O

_
MSN Messenger: converse com os seus amigos online.  
http://messenger.msn.com.br

=
Instruções para entrar na lista, sair da lista e usar a lista em
http://www.mat.puc-rio.br/~nicolau/olimp/obm-l.html
=

=
Instruções para entrar na lista, sair da lista e usar a lista em
http://www.mat.puc-rio.br/~nicolau/olimp/obm-l.html
=


Re: [obm-l] News from IMC!!!!!!! / Dirichlet

2003-07-28 Por tôpico Carlos Gustavo Tamm de Araujo Moreira
   Oi Paulo,   
   O que o Okakamo disse no e-mail dele foi Problema 6:
(...)
 e estimado pupilo Eduardo Tengan, que resolveu o
 problema 6 da IMO (de uma
 forma elegante) e o participante simplesmente copiou e
(...)
   Como o Tengan mandou uma solucao do problema 6 para a lista, por
intermedio do Edmilson (que o Dirichlet copiou e colou, citando a fonte), 
eu achei que era isso. Porque voce afirma que o Okakamo se referiu ao 
problema 3, e nao ao 6 ?
   Abracos,
Gugu


Ola Prof Gugu e demais
colegas desta lista ... OBM-L,

A mensagem a que o Prof Okakamo se refere e :


 INICIO DA MENSAGEM DO DIRICHLET


LEGALLEGALLEGAL!Consegui fechar o problema 3 da 
IMO!!Confiram ai pra ver se nao tem nenhum erro.

Agradeço ao Paulo Santa Rita pelas suas dicas de vetores e ao Marcio Afonso 
Assad Cohen pela ideia de usar desigualdades(e ao problema cinco da IMO da 
India pela fabulosa ideia) e ao Tengan por ter me ensinado a aplicar a 
tecnica  leve tudo ate as ultimas consequencias!!!.

O nosso hexagono tera o nome bastante original de ABCDEF cujos pontos medios 
sao o hexagono nao menos original MNPQRS,com M em AB,N em BC e assim por 
diante.
O enunciado afirma que MQ=(raiz(2) 3)/2 *(AB+DE)= e assim por diante.Devemos 
mostrar que o hexagono e equiangulo.
Seja t=sen 30°=COS 60º=(raiz(2) 3)/2.
Começaremos demonstrando que os lados opostos são paralelos.

Considere a origem em qualquer lugar.O ponto medio de AB e (A+B)/2 entao o 
vetor que liga os pontos medios de AB e DE e (A+B-D-E)/2.Assim 
|A+B-D-E|=2t*(|A-B|+|D-E|).Para melhorar a joça vamos tentar usar a 
desigualdade triangular para vetores.Sabemos que |A-B|+|D-E|=|A-B-D+E| com 
igualdade se e somente se AB//DE(entendeu o paralelismo ai?).Assim 
|AD+BE|=2t|AD-BE|.
Quadrando e usando produto interno euclidiano:
AD,AD+2DA,EB+EB,EB =
3*(AD,AD-2DA,EB+EB,EB) ou equivalentemente AD,AD+EB,EB 
=4*AD,EB.Analogamente
EB,EB²+|FC,FC|²=4*EB,FC
FC,FC²+AD,AD²=-4*FC,DA.
Somando,obtemos 2*(DA-EB+FC)²=0.Logo
DA-EB+FC=0,e os lados opostos são paralelos(verifique as igualdades!).

0=A-D-B+E+C-F=BA+DC+FE,logo os vetores AB,CD e EF podem fechar um 
triangulo.Isto ja e mais um ponto ao nosso favor!
Se transladarmos EF na direção ED,obtemos CG no prolongamento do lado 
BC,devido ao paralelismo.Em miudos, desenhamos os paralelogramos 
ADCH,ABEI,CFEG.Devido aos paralelismos produzimos um triângulo GHI com o 
nosso ABCDEF dentro.Basta demonstrar que o triangulo GHI e equilatero.Para 
tal:

DEVER DE CASA:cada mediana de um certo triangulo mede
sen 60°  vezes o lado correspondente.Calcule os angulos do 
triangulo.(DICA:umas SLCs podem ajudar,ou use Stewart)

A PARTIR DAQUI FAÇA UM BOM DESENHO!

Os triangulos HAB,FIE,CDG e HIG são semelhantes por paralelismo.E pelo mesmo 
motivo os tres primeiros triangulos sao congruentes e homoteticos ao 
quarto(se lembra dos vetores?).Assim IE=DG e EQ=DQ,logo IQ=QG e assim sendo 
Q e ponto medio de GI.Pela homotetia e visivel que H,M,Q  se alinham.
Vamos calcular a razao HQ/IG:
HQ/IG=HM/AB=(HM-HQ)/(AB-GI)=MQ/(AB+DE)=cos 60°,e de acordo com o dever de 
casa(voce fez?),COMEMORE!!

Sera que ressa vai pra Eureka! ?Vou enviar JA!!!

UM ABRAÇAO!Ass.:Johann


** FIM DA MENSAGEM DO DIRICHLET



From: Carlos Gustavo Tamm de Araujo Moreira [EMAIL PROTECTED]
Reply-To: [EMAIL PROTECTED]
To: [EMAIL PROTECTED]
Subject: Re: [obm-l] News from IMC!!! / Dirichlet
Date: Mon, 28 Jul 2003 10:54:21 -0300 (EST)

   Caros Paulo e Okakamo,
Nao sei a qual mensagem do Dirichlet voces se referem, mas a que eu vi 
(e
da qual reproduzo abaixo uma parte) sobre o problema 6 menciona
explicitamente que e' do Tengan a solucao que ele copia. Se for assim acho
que voces estao exagerando um pouco...
Abracos,
 Gugu


Date: Mon, 21 Jul 2003 15:43:53 -0300 (ART)
From: [** iso-8859-1 charset **] Johann Peter Gustav Lejeune
Dirichlet[EMAIL PROTECTED]
Subject: Re: [obm-l] E-mail do Tengan sobre o IMO 6
To: [EMAIL PROTECTED]

---Executing: shownonascii
This message contains non-ASCII text, but the iso-8859-1 font
has apparently not yet been installed on this machine.
(There is no directory named /usr/local/lib/fonts.)
What follows may be partially unreadable, but the English (ASCII) parts
should still be readable.

Legal,esta ideia e parecida com a minha.Mas uma
coisa:alguem pode ser mais explicito nesta parte
de olhar a raiz primitiva de q?E como e que a
ordem e exatamente p?

  --- edmilson motta [EMAIL PROTECTED] escreveu:
  Ei pessoal,
 
  voces notaram que o problema 6 da prova e' uma
  versao simplificada de um problema que eu e o
  Ed
  mandamos em uma das listas de treinamento do
  ano
  passado?  O problema da lista era algo assim:
 
  Sejam a,r1 e p um primo.  Prove que existe um
  primo q tal que (a mod q) tem ordem p^r.
 
 Ola Prof Okakamo e demais
 colegas desta lista ... OBM-L,
 
 Protesto !
 
 Eu, em OFF, estava dizendo ao Gugu que aguardassemos mais um tempo antes 
de
 enviar solucoes das

Re: [obm-l] Log-Integral e Numeros Primos

2003-07-25 Por tôpico Carlos Gustavo Tamm de Araujo Moreira
   Caro Claudio,
   Tanto o liminf quanto limsup acima sao sabidamente infinitos. Sabe-se que
liminf(X(n)log(n)/(n^(1/2).log log log (n)))=-1/2 e que
limsup(X(n)log(n)/n^(1/2).log log log(n)))=+1/2. Isso e' um teorema do
Littlewood (vi isso no livro do A. E. Ingham, The distribution of prime
numbers, Cambridge University Press.
   Abracos,
Gugu
  

Caros colegas da lista:

Alguem saberia dizer se a sequencia:
X(n) = Pi(n) - Li(n) eh limitada e se sao conhecidos o lim inf e lim sup?

onde:
Pi(n) = no. de primos = n;
Li(n) = log-integral de n = Integral(2 a n) dx/ln(x)

OBS: Sabe-se que lim Pi(n)/Li(n) = 1 e que X(n) muda de sinal infinitas
vezes.

Um abraco,
Claudio.

=
Instruções para entrar na lista, sair da lista e usar a lista em
http://www.mat.puc-rio.br/~nicolau/olimp/obm-l.html
=

=
Instruções para entrar na lista, sair da lista e usar a lista em
http://www.mat.puc-rio.br/~nicolau/olimp/obm-l.html
=


Re: [obm-l] Elevador

2003-07-18 Por tôpico Carlos Gustavo Tamm de Araujo Moreira
  Caro Domingos,
  Voce pode esquecer as minhas tres primeiras linhas: elas so' servem como
explicacao de como eu cheguei a essa solucao (e alias nao estao bem
escritas: eu devia ter dito que o conjunto dos elevadores (ou, mais
propriamente, o conjunto dos conjuntos de andares nos quais para cada
elevador) e' {(rx{0})U(rx{1}), r em R}).
  Nas tres linhas seguintes eu descrevo uma solucao explicita com 14
andares, listando os conjuntos de andares em que para cada elevador. Essa
solucao nao e' unica: dada qualquer permutacao de {1,2,...,14}, podemos
aplicar essa permutacao a todos os elementos de todos os conjuntos que eu
listei e obter outra solucao. Isso da' muitas solucoes (mas nao sei se e'
possivel classificar todas asd solucoes de um jeito simples usando esse tipo
de equivalencia).
  Abracos,
   Gugu
 

Não entendi quase nada do que você falou, mas percebi o meu erro:

Devemos ter 13 elementos distintos, pois o 1º andar deve poder ser levado a
todos os demais...
Isso nos diz que há 2 andares que são repetidos, ie: eles aparecem em
exatamente dois elevadores dentre {E1, E2, E3}.

Eu acabei negligenciando o caso em que não há dois andares repetidos, mas
apenas 1 que aparece nos 3 elevadores.

A propósito, essa é a única solução com N = 14?

[ ]'s

--- x ---

 Oi Nicolau,
 Acho que consegui uma solucao com 14 andares: e´ que P:=P2(Z/(2)) tem 7
elementos e 7 retas (seja R o conjunto de suas retas). Tomamos como conjunto
dos andares Px{0,1}, e como conjunto dos elevadores a diagonal de RxR.
Da´ para escrever mais explicitamente isso: os andares sao 1,2,...,14.
Os elevadores sao {1,2,3,8,9,10},{1,4,5,8,11,12},{1,6,7,8,13,14},
{2,4,6,9,11,13},{2,5,7,9,12,14},{3,4,7,10,11,14} e {3,5,6,10,12,13}.
 Abracos,
  Gugu

Citando Nicolau C. Saldanha [EMAIL PROTECTED]:

=
Instruções para entrar na lista, sair da lista e usar a lista em
http://www.mat.puc-rio.br/~nicolau/olimp/obm-l.html
=


=
Instruções para entrar na lista, sair da lista e usar a lista em
http://www.mat.puc-rio.br/~nicolau/olimp/obm-l.html
=


Re: [obm-l] Problemas da IMO

2003-07-16 Por tôpico Carlos Gustavo Tamm de Araujo Moreira
   Caros Ed et al,
   Eu queria agradecer (com algum atraso; eu estava na SBPC em Recife, com o
Paulo Jose', e nao estava facil conseguir computador) as mensagens (um tanto
exageradas, como a sua e a do Wagner) sobre ter entrado um probleminha nosso 
na IMO. E' claro que eu tambem fiquei contente...
   Essa prova da IMO deu trabalho. Acho (e espero) que as notas de corte
devam estar baixas esse ano... E' para mandar solucoes (mesmo feias) ?
   Abracos,
Gugu


Parabéns, Gugu.

Isso só confirma que você é um dos melhores criadores
de problemas (no bom sentido) do mundo. 
Os últimos bancos já indicavam que era só uma questão
de tempo (para quem não sabe, o Gugu já colocou vários
problemas nas short lists).

O Brasil confirma que está evoluindo em todos os
sentidos!!

Abraços, Ed.




--- [EMAIL PROTECTED] wrote:
 
 
 Prova da IMO retirada do Site
 http://www.mathlinks.go.ro/
 
 O Problema 1 é nois que mandou...
 
 
 First Day - 44th IMO 2003 Japan 
 
 1. Let A be a 101-element subset of the set
 S={1,2,3,...,100}. Prove that 
 there exist numbers t_1, t_2, ..., t_{100} in S such
 that the sets 
 
 Aj = { x + tj | x is in A } for each j = 1, 2, ...,
 100 
 
 are pairwise disjoint. 
 
 
 2. Find all pairs of positive integers (a,b) such
 that the number 
 
 a^2 / ( 2ab^2-b^3+1) is also a positive integer. 
 
 3. Given is a convex hexagon with the property that
 the segment connecting the 
 middle points of each pair of opposite sides in the
 hexagon is  sqrt(3) / 2 
 times the sum of those sides' sum. 
 
 Prove that the hexagon has all its angles equal to
 120. 
 
 
 Second Day - 44th IMO 2003 Japan 
 
 4. Given is a cyclic quadrilateral ABCD and let P,
 Q, R be feet of the 
 altitudes from D to AB, BC and CA respectively.
 Prove that if PR = RQ then the 
 interior angle bisectors of the angles  ABC and 
 ADC are concurrent on AC. 
 
 5. Let x1 = x2 = ... = xn be real numbers, n2. 
 
 a) Prove the following inequality: 
 
 (sum  ni,j=1 | xi - xj | ) 2 = 2/3 ( n^2 - 1 )sum
 ni,j=1 ( xi - xj)^2 
 
 b) Prove that the equality in the inequality above
 is obtained if and only if 
 the sequence (xk) is an arithemetical progression. 
 
 6. Prove that for each given prime p there exists a
 prime q such that n^p - p 
 is not divisible by q for each positive integer n. 
 
 
 
 -
 This mail sent through IMP: http://horde.org/imp/

=
 Instruções para entrar na lista, sair da lista e
 usar a lista em
 http://www.mat.puc-rio.br/~nicolau/olimp/obm-l.html

=


__
Do you Yahoo!?
SBC Yahoo! DSL - Now only $29.95 per month!
http://sbc.yahoo.com
=
Instruções para entrar na lista, sair da lista e usar a lista em
http://www.mat.puc-rio.br/~nicolau/olimp/obm-l.html
=

=
Instruções para entrar na lista, sair da lista e usar a lista em
http://www.mat.puc-rio.br/~nicolau/olimp/obm-l.html
=


Re: [obm-l] Mol fatorial

2003-07-05 Por tôpico Carlos Gustavo Tamm de Araujo Moreira
   Uma ideia e' usar a formula de Stirling: n! e' aproximadamente
n^n.e^(-n).raiz(2.n.pi) (a razao tende a 1 quando n tende a infinito).
Segundo o Mathematica, isso(com alguma estimativa razoanel para a diferenca
entre a razao acima e 1, que de fato e' menor que 1/n) implica que um mol
fatorial tem 14053871809631452780257321 digitos (fiz umas contas usando o
Mathematica).
   Abracos,
Gugu


a quest=E3o eh seca:

Calcule quantos digitos possui um mol fatorial. (Seja um mol =3D =
6,02*10^23)

tentei aproximar Integral de log(n) com log(n!) mas nao sei se da muito =
certo, alguem tem alguma id=E9ia?
=
Instruções para entrar na lista, sair da lista e usar a lista em
http://www.mat.puc-rio.br/~nicolau/olimp/obm-l.html
=


Re: [obm-l] Nenhuma ajuda mesmo

2003-07-02 Por tôpico Carlos Gustavo Tamm de Araujo Moreira
   Cara Alininha,
   Na verdade eu acho que nao entendi bem o seu enunciado: Voce usa o nome A
para dois conjuntos: o subconjunto convexo de X dado inicialmente e 
A= {(a,t) tal que a pertence a a e f(a)= t}. Por outro lado, voce define o
conjunto B mas depois nao fala mais nele... A qual conjunto A se refere a
ultima frase ? Que papel tem o conjunto B no problema ?
   Abracos,
Gugu


Sei que o problema é um pouco off-topic mas aqui me 
parece o único lugar onde posso obter ajuda para os meus 
estudos.

Qualquer ajuda para resolver o problema abaixo será 
excelente. Já esgotei meu conhecimento.

---
Abaixo repito o problema
---
Acredito que seja uma aplicação imediata do Teorema de 
Hahn-Banach na forma da separação, entretanto, como 
surge um produto cartesiano de dois espaços não consegui 
(para minha tristeza) escrever a solução.
O problema é o seguinte: 


X é um espaço normado REAL, A 
é um subconjunto convexo de X com o elemento neutro de X 
pertencente a A. Consideremos ainda uma função côncava f 
satisfazendo f(a) = M ||a|| para todo a em A (||a|| é 
norma de a) e os subconjunto do produto cartesiano de X 
com R:
A= { (a,t) tal que a pertence a a e f(a)= t}
B= { (x,t) tal que x pertence a X e M||x||t}

Queremos mostrar que existe um elemento x* do dual de X 
tal que ||x*||=M e f(a)=x*(a) com a de A e x de X.

Serei muito grata pela ajuda.

Alininha


 
__
Seleção de Softwares UOL.
10 softwares escolhidos pelo UOL para você e sua família.
http://www.uol.com.br/selecao


=
Instruções para entrar na lista, sair da lista e usar a lista em
http://www.mat.puc-rio.br/~nicolau/olimp/obm-l.html
=

=
Instruções para entrar na lista, sair da lista e usar a lista em
http://www.mat.puc-rio.br/~nicolau/olimp/obm-l.html
=


Re: [obm-l] Dica de problema.

2003-07-02 Por tôpico Carlos Gustavo Tamm de Araujo Moreira
   Oi Nicolau,
   E' sabido como essa sequencia cresce assintoticamente ?
   Abracos,
Gugu

On Wed, Jul 02, 2003 at 10:06:48AM -0300, [EMAIL PROTECTED] wrote:
  Caros colegas, aqui vai um bom exercicio de contagem.
  Seja S uma matriz nxn com entradas em {1,... ,n} que nao possui elementos 
 repetidos em suas filas (linhas e colunas). 
  Quantas matrizes existem com tal propriedade?

Uma matriz com estas propriedades é chamada de quadrado latino.
Não existe fórmula simples para o número de quadrados latinos de tamanho n.
Na verdade, o valor do que você pede para n = 10 só foi calculado em 1990.
Os primeiros valores são:

1,2,12,576,161280,812851200,61479419904000,108776032459082956800,
5524751496156892842531225600,998243765821303987172506475692032

Você pode ler mais sobre esta seqüência nas páginas abaixo:

http://www.research.att.com/cgi-bin/access.cgi/as/njas/sequences/eisA.cgi?Anum=A002860
http://mathworld.wolfram.com/LatinSquare.html
http://www.combinatorics.org/Volume_2/PostScriptfiles/v2i1n3.ps

[]s, N.
=
Instruções para entrar na lista, sair da lista e usar a lista em
http://www.mat.puc-rio.br/~nicolau/olimp/obm-l.html
=

=
Instruções para entrar na lista, sair da lista e usar a lista em
http://www.mat.puc-rio.br/~nicolau/olimp/obm-l.html
=


Re: [obm-l] Re: [obm-l]_Olá!

2003-06-28 Por tôpico Carlos Gustavo Tamm de Araujo Moreira
   Definicao nao se demonstra, mas vou mencionar dois fatos a favor de
definir 0^0=1:
i) Pelo binomio de Newton, 0^k=(1-1)^k=Soma(j=0 ate' k)(C(k,j).(-1)^j, para
todo k natural. Fazendo k=0, temos 0^0=C(0,0).(-1)^0=0!/(0!.0!)=1 (note que
1=1!=1.0! mostra que 0!=1 e' a definicao natural de 0!). Em geral, para
aplicacoes em combinatoria, parece natural definir 0^0=1.
ii) lim(x-0)(x^x)=1, e ,mais geralmente, para todo real a  0,
lim(x-0)(x^(x^a))=1. Assim, em muitos problemas de calculo, ou analise,como 
queiram (eu arriscaria dizer na maioria) o valor mais natural de 0^0 e' 1.
   Abracos,
Gugu


Nicolau, confesso que não tinha conhecimento desta definição para 0^0. Fui 
tentar ajudar e acabei atrapalhando. Obrigado pelo esclarecimento. 
Abraços. Fabio. 



 Em 27 Jun 2003, [EMAIL PROTECTED] escreveu: 

On Fri, Jun 27, 2003 at 04:45:10AM -0300, Fabio Henrique wrote: 
 Thiago, desculpe me intrometer. O que você diz é verdade. Por isso, 0/0 é 
 INDETERMINADO. Pode-se estender este raciocínio para 0^0. Pense comigo: 
0^0 
 = 0^k/0^k com k diferente de zero. Mas 0^k = 0. Logo, 0^0 = 0/0 que é 
 indeterminado. 
 
Esse papo de indeterminado só deve ser usado quando se fala de limites. 
 
O usual é não definir 0/0 e definir 0^0 = 1. 
 
De novo, isso são definições. 
 
[]s, N. 
= 
Instruções para entrar na lista, sair da lista e usar a lista em 
http://www.mat.puc-rio.br/~nicolau/olimp/obm-l.html 
= 
 
-- 

_
Voce quer um iGMail protegido contra vírus e spams? 
Clique aqui: http://www.igmailseguro.ig.com.br
Ofertas imperdíveis! Link: http://www.americanas.com.br/ig/
Ofertas imperdíveis!

=
Instruções para entrar na lista, sair da lista e usar a lista em
http://www.mat.puc-rio.br/~nicolau/olimp/obm-l.html
=

=
Instruções para entrar na lista, sair da lista e usar a lista em
http://www.mat.puc-rio.br/~nicolau/olimp/obm-l.html
=


Re: [obm-l] Inscriçoes para OBM

2003-06-26 Por tôpico Carlos Gustavo Tamm de Araujo Moreira
   Caro Thiago,
   O ideal seria convencer a sua universidade a se cadastrar, o que nao e'
dificil e e' gratuito. Assim, outros alunos poderiam fazer a prova. Se isso
nao for possivel, voce pode fazer a prova em outra universidade cadastrada
(mas nesse caso voce deve entrar antes em contato com o coordenador da
olimpiada na universidade onde voce for fazer a prova). Mas antes tente
convencer algum professor seu a cadastrar sua universidade...
   Ha' apenas um nivel universitario da OBM, ou seja, todos os
universitarios fazem as mesmas provas e disputam entre si.
   Abracos,
Gugu
 

Olá galera. Minha universidade nao esta cadastrada na
OBM. Como eu poderia fazer a inscriçao e onde eu faria
as provas e até quando é a incricao?
Na OBM todos os universitarios concorrem contra todos
ou existe niveis ?
Obrigado

___
Yahoo! Mail
Mais espaço, mais segurança e gratuito: caixa postal de 6MB, antivírus, proteção 
contra spam.
http://br.mail.yahoo.com/
=
Instruções para entrar na lista, sair da lista e usar a lista em
http://www.mat.puc-rio.br/~nicolau/olimp/obm-l.html
=

=
Instruções para entrar na lista, sair da lista e usar a lista em
http://www.mat.puc-rio.br/~nicolau/olimp/obm-l.html
=


Re: [obm-l] 2 Problemas

2003-06-25 Por tôpico Carlos Gustavo Tamm de Araujo Moreira
Caros colegas,
Seguem abaixo (no texto) comentarios sobre o segundo problema que eu
propus.
Abracos,
 Gugu 


   Caros colegas,
   Coloquei na minha pagina (www.impa.br/~gugu , mais precisamente em
www.impa.br/~gugu/ChebSum2.ps ) uma versao atualizada da nota que eu
mencionei abaixo (que agora esta' organizada de um jeito um pouco diferente,
incluindo a Proposicao 1, que implica esses resultados sobre o modulo maximo
que eu mencionei abaixo, e incorporando correcoes achadas pelo Fabio).
   Ja' que ninguem escreveu sobre os problemas da minha mensagem anterior
vou escrever solucoes resumidas.
   Abracos,
Gugu


Caros colegas,
Coloquei na minha pagina (www.impa.br/~gugu , mais precisamente em
www.impa.br/~gugu/ChebSum.ps ) uma nota que prova que o polinomio maximo do 
problema 2 do Duda e' o n-esimo polinomio de Chebyshev P_n (na nota eu chamo
de T_n), como eu mencionei abaixo (de fato eu enunciei um 
resultado um pouco mais geral); sobre o modulo maximo dos coeficientes
tambem e' o P_n - isso ja' era conhecido (ver o livro do Rivlin citado na
nota), e tambem segue da prova do teorema da nota. A prova e' relativamente 
elementar: eu so' uso interpolacao de Lagrange. Como eu nao achei esse 
resultado na literatura, acho que vou submeter a alguma revista para ver o 
que acontece (em particular para ver se isso ja' e' conhecido ou nao...).
Abracos,
Gugu

P.S.: Alguem fez os exercicios que eu propus na mensagem abaixo ? 


   Caro Duda,
   O problema 2 e' realmente muito interessante. Acho que para todo n o
maximo e' atingido pelo n-esimo polinomio de Chebyshev P_n(x) (que e'
definido por cos(nx)=P_n(cos(x)), e satisfaz a recorrencia
P_(n+1)(x)=2x.P_n(x)-P_(n-1)(x), P_0(x)=1, P_1(x)=x). O valor da soma dos
modulos dos coeficientes de P_n e' s_n:=((1+raiz(2))^n+(1-raiz(2))^n)/2
(note que (s_n) satisfaz s_(n+1)=2.s_n+s_(n-1)).
   Os polinomios de Chebyshev sao extremais em muitos sentidos, sendo o mais
popular e mais importante o seguinte: se P e' um polinomio real de grau n=1
tal que |P(x)|=1 para -1=x=1, entao o coeficiente lider (de x^n) de P tem
modulo no maximo 2^(n-1), que e' atingido por P_n. E' um bom exercicio
provar isso (Sugestao: P_n(x) tem modulo 1 para os seguintes n+1 valores de
x no intervalo [-1,1]: cos(k.pi/n), com 0=k=n).

   De fato, se P e' um tal polinomio cujo coeficiente lider A tem modulo
maor que 2^(n-1) entao (2^(n-1)/A).P(x) e' um polinomio com coeficiente
lider 2^(n-1) de grau n tal que |P(x)|  1 para -1=x=1. Assim,
Q(x)=P_n(x)-P(x) e' um polinomio de grau = n-1 (pois o coeficiente lider de 
P_n e' 2^(n-1), e como P_n(cos(k.pi/n))=cos(k.pi)=(-1)^k (e |P(cos(k.pi/n)|1), 
o sinal de Q(cos(k.pi/n)) e' (-1)^k, para 0=k=n, donde Q(x) tem pelo menos
uma raiz entre cos((k-1).pi/n) e cos(k.pi/n), para 1=k=n, e logo tem pelo
menos n raizes, o que e' absurdo, pois seu grau e' menor que n.  

   Usando esse resultado (e a prova dele), nao e' muito dificil mostrar que
a soma dos modulos dos coeficientes de um polinomio P(x) de grau n tal que
|P(x)|=1 para -1=x=1 e' no maximo s_n+2.s_(n-1)2.s_n (exercicio). 

  Acho que vou deixar isso para depois (nao estou conseguindo lembrar do meu
argumento...), mas e' claro que isso segue do fato abaixo...

   Bom, eu acho que eu me enganei: o argumento que eu tinha nao da'
exatamente isso, mas uma coisa mais fraca (lembrando sempre que uma
estimativa mais forte,e de fato a mais forte possivel, que e' s_n, esta'
provada nessa nota que eu coloquei na minha pagina): Se x_n e' o maior valor 
possivel da soma dos modulos dos coeficientes de um polinomio P(x) de grau n
com |P(x)|=1 para -1=x=1, entao x_n  7.s_n.
   Para isso, vamos provar primeiro que x_n=s_n+2.x_(n-1) (isso parece com
o que eu tinha enunciado, mas infelizmente nao e' a mesma coisa...). De
fato, se P(x) e' um tal polinomio de grau n com coeficiente lider A, temos
|A|=2^(n-1) pelo problema anterior. Consideremos entao o polinomio
Q(x)=(P(x)-(A/2^(n-1)).P_n(x))/2. Temos que Q(x) e' um polinomio de grau
=n-1, pois o coeficiente lider de P_n e' 2^(n-1), e |Q(x)|=1 para -1=x=1,
pois |A/2^(n-1)|=1 e |P_n(x)|=1 para -1=x=1. Assim, por hipotese de
inducao, a soma dos modulos dos coeficientes de Q(x) e' menor ou igual a
x_(n-1), e portanto, como P(x)=2.Q(x)+(A/2^(n-1)).P_n(x), a soma dos modulos 
dos coeficientes de P(x) e' no maximo 2.x_(n-1)+s_n (pois a soma dos modulos 
dos coeficientes de P_n(x) e' s_n, e |A/2^(n-1)|=1).
   Agora, como s_n = s_(n-1), e s_(n+1)=2.s_n+s_(n-1), temos s_(n+1)=3.s_n,
para todo n=1, e portanto s_(n+2)=2.s_(n+1)+s_n = 2.s_(n+1)+s_(n+1)/3 =
(7/3).s_(n+1), para todo n=1. Por outro lado, x_0=1, donde x_1=s_1+2.x_0=3,
e logo x_n  7.s_n para n=1 e n=2. Temos entao, por inducao, para n=3,
x_n=s_n+2.x_(n-1)=s_n+14.s_(n-1)=s_n+14.(3/7).s_n=7.s_n, c.q.d.
 

Eu acho que sei provar que o maximo de fato e' s_n, mas isso da' mais
trabalho.

   Vejam a nota mencionada acima!

   Abracos,
   Gugu



Caros colegas da 

Re: [obm-l] Re: [obm-l] Livro - Teoria dos Números

2003-06-24 Por tôpico Carlos Gustavo Tamm de Araujo Moreira
   Voce tambem podia dar uma olhada no meu livro com o Nicolau Primos de
Mersenne (e outros primos muito grandes), que esta' disponivel na minha
pagina www.impa.br/~gugu e na do Nicolau www.mat.puc-rio.br/~nicolau 
Tambem tem o livro Introducao a teoria dos numeros (com enfase em
aproximacoes diofantinas), texto de um minicurso que eu dei no Peru, 
tambem na minha pagina.
   Abracos,
   Gugu


Recomendo-lhe dois livros:

SANTOS, José Plínio de Oliveira -  Introdução à Teoria dos Números  - 
Coleção Matemática Universitária - SBM

MILIES, César Polcino  , COELHO, Sônia Pitta - Números: Uma Introdução à 
Matemática - edUSP

O segundo trata de temas mais básicos, o priimeiro faz interessantes 
ligações entre combinatória e teoria dos números e  avança até o estudo de 
raízes primitivas, resíduos quadráticos e fraçoes parciais.

OBS: ambos tem preço acessível, algo em torno de R$25,00, cada.

Frederico.


From: Victor Luiz [EMAIL PROTECTED]
Reply-To: [EMAIL PROTECTED]
To: [EMAIL PROTECTED]
Subject: [obm-l] Livro - Teoria dos Números
Date: Mon, 23 Jun 2003 21:05:43 -0300

-BEGIN PGP SIGNED MESSAGE-
Hash: SHA1

Indo de carona nessas recomendações de livros que têm sido enviados para a
lista eu gostaria de que me recomendassem algum livro básico que trate da
teoria dos números.


Obrigado,

Victor Luiz Salgado de Lima.

- 
Spam sux. www.wecanstopspam.org
-BEGIN PGP SIGNATURE-
Version: GnuPG v1.2.1 (MingW32) - GPGOE 0.4.1

iD8DBQE+95XWpBwZ7xrHmVsRApEpAJ9FFOr2m5dx/QvAYcVTMYFNUmR4+ACeN5a9
yK0v7zUzhowGTGO2kYButko=
=Xqs/
-END PGP SIGNATURE-

=
Instruções para entrar na lista, sair da lista e usar a lista em
http://www.mat.puc-rio.br/~nicolau/olimp/obm-l.html
=

_
MSN Hotmail, o maior webmail do Brasil.  http://www.hotmail.com

=
Instruções para entrar na lista, sair da lista e usar a lista em
http://www.mat.puc-rio.br/~nicolau/olimp/obm-l.html
=

=
Instruções para entrar na lista, sair da lista e usar a lista em
http://www.mat.puc-rio.br/~nicolau/olimp/obm-l.html
=


Re: [obm-l] Dicas: Análise Funcional

2003-06-21 Por tôpico Carlos Gustavo Tamm de Araujo Moreira
Cara Alininha,
Use o fato de que um funcional linear que nao e' continuo nao e'
limitado, ou seja, voce pode encontrar elementos v de X com |v|  1 e |f(v)|
tao grande quanto voce quiser para mostrar que, dado x em X existem
elementos do nucleo de f (a imagem inversa de 0) arbitrariamente proximos de
x, somando a x elementos pequenos de X escolhidos convenientemente.
   Abracos,
   Gugu


Amigos,

estou inciandos meus estudos de análise funcional sem 
muito background matemático e por isso estou encontrado 
muitas dificuldades.

Gostaria que me dessem, se possível, algumas dicas para 
provar:

Seja X um espaço normado. Se f é  um funcional linear 
NÃO contínuo Então a imagem inversa de 0 é denso em X

Quero resolver sozinha e por isso gostaria apenas de 
algumas dicas...

Muito obrigada.

 
__
Seleção de Softwares UOL.
10 softwares escolhidos pelo UOL para você e sua família.
http://www.uol.com.br/selecao


=
Instruções para entrar na lista, sair da lista e usar a lista em
http://www.mat.puc-rio.br/~nicolau/olimp/obm-l.html
=

=
Instruções para entrar na lista, sair da lista e usar a lista em
http://www.mat.puc-rio.br/~nicolau/olimp/obm-l.html
=


Re: [obm-l] 2 Problemas

2003-06-13 Por tôpico Carlos Gustavo Tamm de Araujo Moreira
Caros colegas,
Coloquei na minha pagina (www.impa.br/~gugu , mais precisamente em
www.impa.br/~gugu/ChebSum.ps ) uma nota que prova que o polinomio maximo do 
problema 2 do Duda e' o n-esimo polinomio de Chebyshev P_n (na nota eu chamo
de T_n), como eu mencionei abaixo (de fato eu enunciei um 
resultado um pouco mais geral); sobre o modulo maximo dos coeficientes
tambem e' o P_n - isso ja' era conhecido (ver o livro do Rivlin citado na
nota), e tambem segue da prova do teorema da nota. A prova e' relativamente 
elementar: eu so' uso interpolacao de Lagrange. Como eu nao achei esse 
resultado na literatura, acho que vou submeter a alguma revista para ver o 
que acontece (em particular para ver se isso ja' e' conhecido ou nao...).
Abracos,
Gugu

P.S.: Alguem fez os exercicios que eu propus na mensagem abaixo ? 


   Caro Duda,
   O problema 2 e' realmente muito interessante. Acho que para todo n o
maximo e' atingido pelo n-esimo polinomio de Chebyshev P_n(x) (que e'
definido por cos(nx)=P_n(cos(x)), e satisfaz a recorrencia
P_(n+1)(x)=2x.P_n(x)-P_(n-1)(x), P_0(x)=1, P_1(x)=x). O valor da soma dos
modulos dos coeficientes de P_n e' s_n:=((1+raiz(2))^n+(1-raiz(2))^n)/2
(note que (s_n) satisfaz s_(n+1)=2.s_n+s_(n-1)).
   Os polinomios de Chebyshev sao extremais em muitos sentidos, sendo o mais
popular e mais importante o seguinte: se P e' um polinomio real de grau n=1
tal que |P(x)|=1 para -1=x=1, entao o coeficiente lider (de x^n) de P tem
modulo no maximo 2^(n-1), que e' atingido por P_n. E' um bom exercicio
provar isso (Sugestao: P_n(x) tem modulo 1 para os seguintes n+1 valores de
x no intervalo [-1,1]: cos(k.pi/n), com 0=k=n).
   Usando esse resultado (e a prova dele), nao e' muito dificil mostrar que
a soma dos modulos dos coeficientes de um polinomio P(x) de grau n tal que
|P(x)|=1 para -1=x=1 e' no maximo s_n+2.s_(n-1)2.s_n (exercicio). Eu
acho que sei provar que o maximo de fato e' s_n, mas isso da' mais trabalho.
   Abracos,
   Gugu



Caros colegas da lista,

tenho dois problemas a propor. O primeiro é filosófico e pedagógico. O
segundo é sobre polinômios. Lá vão eles:

Problema 1.  O que vocês acham sobre o método de ensino da matemática,
deve-se começar pelo concreto e ir em direção ao abstrato mais geral, ou
seguir o caminho inverso? Por exemplo, estudar topologia geral depois
estudar topologia no R^n e em seguida topologia R, como coisas particulares.
Ou começar pelo R, depois R^n e a visão mais geral. Eu tenho a impressão que
partir de idéias muito abstratas e gerais é uma abordagem que não dá bons
resultados, pois começa-se a partir de um ponto onde a confusão é muito mais
natural de acontecer. Particularmente, eu nunca comecei desse modo, sempre
estudei as coisas na ordem usual. O Halmos, por exemplo, defende a idéia de
estudar espaços de Hilbert simultaneamente a espaços vetoriais de dimensão
finita.

A ordem de criação da matemática é do mais concreto para o mais abstrato,
pelo que compreendo. Depois que se viram muitas estruturas de
características similares (espaços vetoriais com certas propriedades, p.e.),
se retira uma noção mais geral e abstrata (espaços de Hilbert) e daí
retiram-se propriedades mais fracas mas muito gerais. E a teoria continua
num crescendum, até que, tavez, todos os campos se unifiquem numa grande
visão, essa é a minha esperança. A criação da matemática, pelo que tenho na
minha memória, segue a direção concreto -- abstrato. Mas isso não implica
que o ensino deva seguir essa mesma direção. Existem experiências de ensino
ou alguém já estudou algum assunto seguindo a ordem inversa? O que relata
dessas experiências?

Problema 2. Se um polinômio p(x) de grau n (particularmente gostaria de
saber sobre o caso n=3) é tal que |p(t)| = 1 para |t| = 1, então o que
podemos dizer sobre os coeficientes de p(x)? Qual o máximo módulo que eles
podem ter? E sobre a soma em módulo dos coeficientes, qual o máximo?

Este problema me surgiu na aula de Análise no R^n. É certo que tal máximo de
fato existe, pois todas as normas em R^n são equivalentes, mas determinar o
máximo me parece um problema interessante. Não sei se ele tem uma resposta
simples, acho que não, mas pode-se fazer algumas estimativas do máximo.

Um Abraço a todos,
Duda.

=
Instruções para entrar na lista, sair da lista e usar a lista em
http://www.mat.puc-rio.br/~nicolau/olimp/obm-l.html
=

=
Instruções para entrar na lista, sair da lista e usar a lista em
http://www.mat.puc-rio.br/~nicolau/olimp/obm-l.html
=


=
Instruções para entrar na lista, sair da lista e usar a lista em
http://www.mat.puc-rio.br/~nicolau/olimp/obm-l.html

Re: [obm-l] Primos em PA

2003-06-12 Por tôpico Carlos Gustavo Tamm de Araujo Moreira


- Original Message -
From: Carlos Gustavo Tamm de Araujo Moreira [EMAIL PROTECTED]
To: [EMAIL PROTECTED]
Sent: Thursday, June 12, 2003 1:09 AM
Subject: Re: [obm-l] Primos em PA


Caro Claudio,
O teorema de Dirichlet claramente implica a afirmacao do problema 8.
Por
 outro lado, dados inteiros primos entre si a e b, com b1, a afirmacao do
 problema 8 implica que para todo n existe um primo congruente a a+b.n
modulo
 b^n. Esse primo claramente tambem e' congruente a a modulo b, e nao e'
dificil
 ver que dessse jeito geramos infinitos primos, e portanto existem
infinitos
 primos congruentes a a modulo b. Isso resolve o problema 8 abaixo.

Oi, Gugu:

Desculpe a minha lerdeza mental, mas o fato de existirem infinitos primos
congruentes a a modulo b não é justamente a conclusão do teorema de
Dirichlet?

Ou seja, a meu ver você acabou de provar que se mdc(a,b) = 1 e se existe um
primo da forma a + bn, então existem infinitos primos dessa forma. Ou estou
enganado?

   Bem, Claudio, o que eu provei foi que se mdc(a,b)=1 e se, PARA QUAISQUER
A e B com mdc(A,B)=1 existe algum primo congruente a A modulo B entao
existem infinitos primis congruentes a a modulo b. Na prova desse fato eu
uso infinitos valores be B (B=b^n), apesar de a e b estarem fixos. E' um
pouco diferente... Para conseguir infinitos primos congruentes a 2 modulo 5
eu precisaria de conseguir algum primo em certas classes de congruencia
modulo 5, modulo 25, modulo 125, modulo 625, etc, e nao apenas saber que
existe algum primo congruente a 2 modulo 5.
   Abracos,
Gugu


 Apesar
 disso, sem o teorema de Dirichlet, continuamos sem conhecer uma prova
 simples da existencia de infinitos primos congruentes a 2 modulo 5...

De acordo com o que você provou, não.
Basta tomar a = 2, b = 5 e verificar que mdc(a,b) = 1 e que a + b*1 = 7 é
primo.

Abracos,
Gugu

 
Um abraço,
Claudio.

=
Instruções para entrar na lista, sair da lista e usar a lista em
http://www.mat.puc-rio.br/~nicolau/olimp/obm-l.html
=

=
Instruções para entrar na lista, sair da lista e usar a lista em
http://www.mat.puc-rio.br/~nicolau/olimp/obm-l.html
=


Re: [obm-l] Integral[Sen(x)/(1+x)](Desafio)

2003-06-10 Por tôpico Carlos Gustavo Tamm de Araujo Moreira
   Caro Leandro,
   Eu nao lembro de ter feito essa conta, mas posso fazer. Por outro lado,
nao e' Tammus, e' Tamm... :-)
   Derivando Si(x+1)Cos(1) - Ci(x+1)Sen(1) obtemos
Cos(1).Sen(x+1)/(x+1)-Sen(1)(1/(x+1)+((Cos(x+1)-1)/(x+1))=
=(Cos(1).Sen(x+1)-Sen(1).Cox(x+1))/(x+1)=Sen((x+1)-1)/(x+1)=Sen(x)/(x+1).
   Abracos,
Gugu

   

O que eu acho que ele quer Claudio e chegar nesse resultado. Porem, se
nao me engano, o Carlos Tammus (Gugu) ja fez essa resolucao
anteriormente.=20

=20

Caso ele queira somente verificar, entao esta correto !!!=20

=20

Saudacoes Rubro-Negras,

=20

Leandro.=20

=20

-Original Message-
From: [EMAIL PROTECTED]
[mailto:[EMAIL PROTECTED] On Behalf Of claudio.buffara
Sent: Tuesday, June 10, 2003 9:35 AM
To: obm-l
Subject: Re:[obm-l] Integral[Sen(x)/(1+x)](Desafio)

=20

=C9 s=F3 derivar o lado direito, levando em conta que:

d(ln(x))/dx =3D 1/x=20

e

d(integral(f(t), t=3D0..x)/dx =3D f(x)

=20

Um abra=E7o,

Claudio.

=20


De:

[EMAIL PROTECTED]


Para:

[EMAIL PROTECTED]

=20


C=F3pia:

[EMAIL PROTECTED]


Data:

Tue, 10 Jun 2003 11:58:02 -0300 (ART)

=20


Assunto:

[obm-l] Integral[Sen(x)/(1+x)](Desafio)


=20

=20

 Algu=E9m se habilita a demonstrar que

=20

 int[Sen(x)/(1+x), x] =3D Si(x+1)Cos(1) - Ci(x+1)Sen(1)

=20

 Si: Seno integral

 Ci: Cosseno integral

=20

 Estas integrais s=E3o definidas para todo x complexo=20

 como segue

=20

 Si(t) =3D int[Sen(t)/t, t=3D0..x]

 Ci(t) =3D gamma + ln(x) + int[(Cos(t)-1)/t, t=3D0..x]

=20

 Obrigado, desde j=E1.

=20

=20


___

 Yahoo! GeoCities

 Tudo para criar o seu site: ferramentas f=E1ceis de usar, espa=E7o de
sobra e acess=F3rios.

 http://br.geocities.yahoo.com/


=3D=3D=3D=3D=3D=3D=3D=3D=3D=3D=3D=3D=3D=3D=3D=3D=3D=3D=3D=3D=3D=3D=3D=3D=3D=
=3D=3D=3D=3D=3D=3D=3D=3D=3D=3D=3D=3D=3D=3D=3D=3D=3D=3D=3D=3D=3D=3D=3D=3D=3D=
=3D=3D=3D=3D=3D=3D=3D=3D=3D=3D=3D=3D=3D=3D=3D=3D=3D=3D=3D=3D=3D=3D
=3D

 Instru=E7=F5es para entrar na lista, sair da lista e usar a lista em

 http://www.mat.puc-rio.br/~nicolau/olimp/obm-l.html


=3D=3D=3D=3D=3D=3D=3D=3D=3D=3D=3D=3D=3D=3D=3D=3D=3D=3D=3D=3D=3D=3D=3D=3D=3D=
=3D=3D=3D=3D=3D=3D=3D=3D=3D=3D=3D=3D=3D=3D=3D=3D=3D=3D=3D=3D=3D=3D=3D=3D=3D=
=3D=3D=3D=3D=3D=3D=3D=3D=3D=3D=3D=3D=3D=3D=3D=3D=3D=3D=3D=3D=3D=3D
=3D

=20

=20


--=_NextPart_000_0022_01C32F3D.395FB530
Content-Type: text/html;
   charset=iso-8859-1
Content-Transfer-Encoding: quoted-printable

html

head
META HTTP-EQUIV=3DContent-Type CONTENT=3Dtext/html; =
charset=3Diso-8859-1


meta name=3DGenerator content=3DMicrosoft Word 10 (filtered)

style
!--
 /* Font Definitions */
 @font-face
   {font-family:Tahoma;
   panose-1:2 11 6 4 3 5 4 4 2 4;}
@font-face
   {font-family:Trebuchet MS\; verdana\; arial;
   panose-1:0 0 0 0 0 0 0 0 0 0;}
 /* Style Definitions */
 p.MsoNormal, li.MsoNormal, div.MsoNormal
   {margin:0in;
   margin-bottom:.0001pt;
   font-size:12.0pt;
   font-family:Times New Roman;}
a:link, span.MsoHyperlink
   {color:blue;
   text-decoration:underline;}
a:visited, span.MsoHyperlinkFollowed
   {color:purple;
   text-decoration:underline;}
span.EmailStyle17
   {font-family:Arial;
   color:navy;}
@page Section1
   {size:8.5in 11.0in;
   margin:1.0in 1.25in 1.0in 1.25in;}
div.Section1
   {page:Section1;}
--
/style

/head

body lang=3DEN-US link=3Dblue vlink=3Dpurple

div class=3DSection1

p class=3DMsoNormalfont size=3D2 color=3Dnavy face=3DArialspan =
lang=3DPT-BR
style=3D'font-size:10.0pt;font-family:Arial;color:navy'O que eu acho =
que ele
quer Claudio e chegar nesse resultado. Porem, se nao me engano, o Carlos =
Tammus
(Gugu) ja fez essa resolucao anteriormente. /span/font/p

p class=3DMsoNormalfont size=3D2 color=3Dnavy face=3DArialspan =
lang=3DPT-BR
style=3D'font-size:10.0pt;font-family:Arial;color:navy'nbsp;/span/fo=
nt/p

p class=3DMsoNormalfont size=3D2 color=3Dnavy face=3DArialspan =
lang=3DPT-BR
style=3D'font-size:10.0pt;font-family:Arial;color:navy'Caso ele queira =
somente
verificar, entao esta correto !!! /span/font/p

p class=3DMsoNormalfont size=3D2 color=3Dnavy face=3DArialspan =
lang=3DPT-BR
style=3D'font-size:10.0pt;font-family:Arial;color:navy'nbsp;/span/fo=
nt/p

p class=3DMsoNormalfont size=3D2 color=3Dnavy face=3DArialspan =
lang=3DPT-BR
style=3D'font-size:10.0pt;font-family:Arial;color:navy'Saudacoes =
Rubro-Negras,/span/font/p

p class=3DMsoNormalfont size=3D2 color=3Dnavy face=3DArialspan =
lang=3DPT-BR
style=3D'font-size:10.0pt;font-family:Arial;color:navy'nbsp;/span/fo=
nt/p

p class=3DMsoNormalfont size=3D2 color=3Dnavy face=3DArialspan =
lang=3DPT-BR
style=3D'font-size:10.0pt;font-family:Arial;color:navy'Leandro. =
/span/font/p

p class=3DMsoNormalfont size=3D2 color=3Dnavy face=3DArialspan =
lang=3DPT-BR
style=3D'font-size:10.0pt;font-family:Arial;color:navy'nbsp;/span/fo=
nt/p

p class=3DMsoNormalfont size=3D2 face=3DTahomaspan =

Re: [obm-l] Questão do Colégio Naval 1981

2003-06-09 Por tôpico Carlos Gustavo Tamm de Araujo Moreira
   Eu acho que quando k^3 = N  (k+1)^3 o resto da extracao da raiz cubica de
m deve ser N-k^3. Isso de o resto ser o maior possivel deve querer dizer que
N=(k+1)^3-1. Assim, 3.k^2+3.k=126, k^2+k-42=0, donde k=6 e N=342, letra b).
   Abracos,
Gugu


Amigos da OBM lista,
gostaria de uma ajuda para resolver uma questão da prova do Colégio Naval de 
1980/81:


10) Ao extrairmos a raiz cúbica do número natural N verificamos que o resto 
era o maior possível e igual a 126. A soma dos algarismos de N é:
a) 11 b) 9 c) 8 d) 7 e) 6


Afinal de contas, o que é resto da raiz cúbica de número natural.


Até mais,
Marcelo Rufino de Oliveira

_
Add photos to your messages with MSN 8. Get 2 months FREE*. 
http://join.msn.com/?page=features/featuredemail

=
Instruções para entrar na lista, sair da lista e usar a lista em
http://www.mat.puc-rio.br/~nicolau/olimp/obm-l.html
=

=
Instruções para entrar na lista, sair da lista e usar a lista em
http://www.mat.puc-rio.br/~nicolau/olimp/obm-l.html
=


Re: [obm-l] numeros primos

2003-06-06 Por tôpico Carlos Gustavo Tamm de Araujo Moreira
   Caro Rafael,
   Tem uma fatoracao que e' assim: x^4+4.y^4=(x^2+2.y^2)^2-(2xy)^2=
=(x^2+2xy+2.y^2)(x^2-2xy+2.y^2). No nosso caso, sendo n impar, n=2k+1,
temos n^4+4^n=n^4+4.(2^k)^4=(n^2+2^(k+1).n+2^(2k+1))(n^2-2^(k+1).n+2^(2k+1)),
que e' sempre composto se k=1.
   Abracos,
Gugu


Oi Pessoal!

O número de valores de n para os quais n^4 + 4^n é um
número primo é:
a)1b)2c)3d)4e)5

Eu acho que quase resolvi a questão, mas ainda falta
uma coisa. Eu fui usar regras de divisibilidade e
potências. Sabendo que qualquer número elevado à
quarta sempre termina em 0, 1, 5 ou 6 e que 4 elevado
a qualquer potência sempre termina em 4 (expoente
ímpar) ou 6 (expoente par), pude ir fazendo algumas
contas.

Como n não pode ser par senão n^4 + 4^n é par, só
poderia ser um número terminado em 1, 3, 5, 7 ou 9.
Mas no caso de n terminar com 1, 3, 7 ou 9, a soma n^4
+ 4^n termina em 5 então não é primo, a não ser no
caso n = 1:
n^4 + 4^n = 5

Esse foi o único caso que achei. Mas não consegui
provar que para os números terminados em 5 a soma n^4
+ 4^n não é primo:
5^4 + 4^5 = 1649 = 17.97

E para o próximo caso, n = 15 já fica difícil ficar
fazendo as contas no braço.

Se alguém tiver outra idéia para resolver essa questão
agradeceria. Principalmente se não tivesse nada muito
elaborado sobre congruências para poder explicar para
um aluno do segundo grau.

Abraços,

Rafael.

___
Yahoo! Mail
Mais espaço, mais segurança e gratuito: caixa postal de 6MB, antivírus, proteção 
contra spam.
http://br.mail.yahoo.com/
=
Instruções para entrar na lista, sair da lista e usar a lista em
http://www.mat.puc-rio.br/~nicolau/olimp/obm-l.html
=

=
Instruções para entrar na lista, sair da lista e usar a lista em
http://www.mat.puc-rio.br/~nicolau/olimp/obm-l.html
=


Re: [obm-l] Duvidas

2003-06-06 Por tôpico Carlos Gustavo Tamm de Araujo Moreira
   Caro Wagner,
   De fato eu ainda nao achei nenhuma evidencia de que o Poncelet soubesse
como provar o seu porisma. Seria bom se alguem tivesse alguma boa referencia
sobre isso...As provas que eu e o Nicolau conhecemos nao sao nada
elementares (a mais simples usa fatos sobre superficies de Riemann -
nao e' isso, Nicolau ?).
   Abracos,
Gugu
 

A palavra porisma eh, na verdade grega.
Euclides escreveu um livro com exatamente este titulo:
Porismas, mas esta perdido.  Infelizmente nao sabemos
o que ele continha.
Sobre o significado que dei dessa palavra, quero dizer que
copiei do dicionario Pillet et Dumoulin editado no
seculo 19.
Eh claro que o porisma de Poncelet nao eh nada facil, mas
talvez essa palavra tambem tenha sido usada como sinonimo
de conjectura, algo que nao eh ainda um teorema.

Abracos,

Wagner.


--
From: Nicolau C. Saldanha [EMAIL PROTECTED]
To: [EMAIL PROTECTED]
Subject: Re: [obm-l] Duvidas
Date: Tue, Jun 3, 2003, 9:16 PM


 On Tue, Jun 03, 2003 at 08:47:46PM -0300, Eduardo Wagner wrote:
 Porismo nao consta dos nossos dicionarios.
 Porismo vem do frances porisme que significa
 uma afirmacao muito facil de demonstrar. Pode ser um lema
 ou um corolario, algo que nao tem o status de teorema.

 O porismo de Poncelet é tudo, menos muito fácil de demonstrar.

 Para quem não sabe, o porismo diz o seguinte:

 Sejam C1 e C2 dois círculos, C2 dentro de C1 mas não concêntricos.
 Seja P0 um ponto de C1; por P0 trace uma tangente a C2 para obter P1,
 a outra interseção desta tangente com C1, trace outra tangente a C2
 para obter P2 e assim por diante como na figura em anexo.
 Suponha que Pn = P0 onde n é um inteiro positivo. Comece agora com outro
 ponto Q0 em C1 e repita a construção para obter Q1, Q2, ..., Qn.

 Prove que Qn = Q0.

 A mesma coisa vale para elipses ou cônicas em geral
 e isso segue facilmente do caso com círculos usando
 transformações projetivas.

 []s, N.
 
=
Instruções para entrar na lista, sair da lista e usar a lista em
http://www.mat.puc-rio.br/~nicolau/olimp/obm-l.html
=

=
Instruções para entrar na lista, sair da lista e usar a lista em
http://www.mat.puc-rio.br/~nicolau/olimp/obm-l.html
=


Re: [obm-l] Duvidas

2003-06-06 Por tôpico Carlos Gustavo Tamm de Araujo Moreira
   O Tomei tem alguma referencia sobre a prova do Poncelet ? Que tecnicas
ela usa ?
   Abracos,
Gugu


On Thu, Jun 05, 2003 at 05:22:55PM -0300, Carlos Gustavo Tamm de Araujo Moreira wrote:
Caro Wagner,
De fato eu ainda nao achei nenhuma evidencia de que o Poncelet soubesse
 como provar o seu porisma. Seria bom se alguem tivesse alguma boa referencia
 sobre isso...As provas que eu e o Nicolau conhecemos nao sao nada
 elementares (a mais simples usa fatos sobre superficies de Riemann -
 nao e' isso, Nicolau ?).

É. Mas o Carlos Tomei afirma que Poncelet provou esta coisa sim,
mas que a demonstração dele é bem complicada e que definitivamente
não é geometria no sentido grego da palavra.

[]s, N.
=
Instruções para entrar na lista, sair da lista e usar a lista em
http://www.mat.puc-rio.br/~nicolau/olimp/obm-l.html
=

=
Instruções para entrar na lista, sair da lista e usar a lista em
http://www.mat.puc-rio.br/~nicolau/olimp/obm-l.html
=


Re: [obm-l] Primos numa PA

2003-06-06 Por tôpico Carlos Gustavo Tamm de Araujo Moreira
Caro Claudio,
Eu estou convencido de que isso e' tao dificil quanto o teorema de
Dirichlet. Falando nisso, alguem sabe uma prova elementar e relativamente
simples de que existem infinitos primos da forma 5k+2 (isso certamente
seguiria do problema abaixo) ?
Abracos,
 Gugu


HelpCaros colegas da lista:

Aqui vai um problema que eu vi num livro de teoria dos n=FAmeros a =
n=EDvel elementar e que continua em aberto aqui na lista:

Prove que:
Se:
a e b s=E3o inteiros com mdc(a,b) =3D 1=20
e=20
existe um primo da forma am + b (m inteiro)
Ent=E3o:
existem infinitos primos desta forma.

Naturalmente a conclus=E3o =E9 o famoso teorema de Dirichlet dobre =
primos numa PA, cuja demonstra=E7=E3o =E9 bem dif=EDcil. No entanto, =
dado o n=EDvel do livro onde eu vi o problema, n=E3o creio que a =
solu=E7=E3o seja muito sofisticada.

Qualquer ajuda ser=E1 grandemente apreciada.

Um abra=E7o,
Claudio.

=
Instruções para entrar na lista, sair da lista e usar a lista em
http://www.mat.puc-rio.br/~nicolau/olimp/obm-l.html
=


Re: [obm-l] f(f(x))_=_x^2_-_1996_é_impossível

2003-06-06 Por tôpico Carlos Gustavo Tamm de Araujo Moreira
   Boa ideia: g(x)=x^2-1996 tem dois pontos fixos e dois pontos periodicos 
de periodo 2 (pontos x e y tais que f(x)=y e f(y)=x) - desenhem o grafico de
g(x) para ver isso. Por outro lado, como f(f(x))=g(x), um ponto periodico de
periodo 2 de g e' um ponto periodico de periodo 4 de f: x, f(x), f(f(x))=y,
f(f(f(x)))=f(y) sao distintos e f(f(f(f(x=x. Mas entao teriamos pelo
menos 4 pontos periodicos de periodo 2 para g (x, f(x), y e f(y)), mas temos
apenas 2, absurdo.
   Abracos,
Gugu

Na verdade não estou ajudando em nada, mas já procurou por pontos fixos? Pontos do 
tipo f(x)=x

Cláudio_(Prática) [EMAIL PROTECTED] wrote:Caros colegas:
 
Estou com dificuldades num problema que caiu no Torneio das Cidades de 1996:
 
Provar que não existe nenhuma função f: R - R tal que:
f(f(x)) = x^2 - 1996.
 
Agradeço qualquer ajuda.
 
Um abraço,
Claudio.




=
Instruções para entrar na lista, sair da lista e usar a lista em
http://www.mat.puc-rio.br/~nicolau/olimp/obm-l.html
=


Re: [obm-l] problema

2003-06-05 Por tôpico Carlos Gustavo Tamm de Araujo Moreira
   Caro Claudio,
   E' interessante notar que isso da' uma prova da desigualdade das medias
aritmetica e geometrica usando a desigualdade do rearranjo (nesse caso na
versao que determina o menor produto interno possivel de um vetor por um
rearranjo seu) : sejam x(1),...,x(n) positivos. Nao ha' perda de
generalidade em supor que seu produto e' 1 (senao dividimos todos pela sua
media geometrica). Seja a(1)=1, a(2)=x(1), a(3)=x(1).x(2), ... ,
a(n)=x(1)x(2)...x(n-1). Note que a(n)=1/x(n). Temos entao, pela desigualdade
abaixo, n=a(1)/a(n)+a(2)/a(1)+a(3)/a(2)+...+a(n)/a(n-1)=
=x(n)+x(1)+x(2)+...+x(n-1), que e' o que queriamos provar.
   Abracos,
   Gugu


Oi, Ricardo:

Na verdade, isso sai por uma aplicacao direta da desigualdade do =
rearranjo.

Suponhamos s.p.d.g. que 0  a(1) =3D a(2) =3D ... =3D a(n).

Entao, 0  1/a(n) =3D 1/a(n-1) =3D ... =3D 1/a(1)

A desigualdade do rearranjo diz que, para qualquer reordenacao b(1), =
..., b(n) dos a(i), vale o seguinte:
a(1)*(1/a(1)) + a(2)*(1/a(2)) + ... + a(n)*(1/a(n)) =3D b(1)*(1/a(1)) + =
b(2)*(1/a(2)) + ... + b(n)*(1/a(n))

Ou seja:
1 + 1 +  + 1 =3D n =3D b(1)/a(1) + b(2)/a(2) + ... + b(n)/a(n)

Um abraco,
Claudio.
  - Original Message -=20
  From: Ricardo Prins=20
  To: [EMAIL PROTECTED]
  Sent: Thursday, June 05, 2003 9:48 PM
  Subject: [obm-l] problema


  =E9, morgado, n=E3o consegui. desisto.=20

  prove que, se b(1),b(2),b(3),...,b(n) =E9 uma reordena=E7=E3o dos =
n=FAmeros positivos a(1),a(2),...,a(n), ent=E3o

  b(1)/a(1) + b(2)/a(2) + ... + b(n)/a(n) =3D n

  bom, a dica foi usar desigualdade das m=E9dias...t=E1... somat=F3rio =
dos a(i)/n =3D raiz en=E9sima do produt=F3rio dos a(i)...mas n=E3o =
consigo pensar em mais nadatentei indu=E7=E3o tb n=E3o saiu...o que =
fa=E7o?
=
Instruções para entrar na lista, sair da lista e usar a lista em
http://www.mat.puc-rio.br/~nicolau/olimp/obm-l.html
=


Re: [obm-l] Duvidas

2003-06-05 Por tôpico Carlos Gustavo Tamm de Araujo Moreira
  Caro Duda,
  Um texto basico qualquer sobre superficies de Riemann e' suficiente. No
curso que eu fiz no IMPA (ha' algum tempo...) nos usamos o Farkas-Kra. Na
verdade a solucao que eu conheco e' do Nicolau. Se nao me engano a ideia e'
complexificar os circulos e considerar o conjunto dos pares de pontos (p,q)
onde p pertence ao circulo de fora e q ao de dentro de modo que a reta
pq e' tandente ao circulo de dentro. Eu pus essas aspas pois ao
complexificarmos os circulos (i.e., ao considerarmos conjuntos definidos
pelas mesmas equacoes que as dos circulos, mas considerando as variaveis
complexas), eles passam a se intersectar, e na verdade esses pontos de
intersecao tem um papel especial: a pontos genericos p d circulo de fora
correspondem dois pontos q no de dentro, mas se p e' um ponto de intersecao
dos dois circulos entao so' corresponde a ele o ponto p. Esses pares (p,q)
formam uma superficie de Riemann M, e f(p,q)=p e' uma funcao de grau 2 de M
no circulo de fora (que, complexificado, e' uma esfera de Riemann). Como f
tem dois pontos de ramificacao (os dois pontos de intersecao dos circulos),
o teorema de Riemann-Hurwitz mostra que M tem que ser um toro.  Agora
consideramos a funcao g que leva (p,q) em (p',q'), onde p' e p sao os pontos
de intersecao de pq com o circulo de fora e p'q e p'q' sao as duas
tangentes ao circulo de dentro passando por p'. Isso da' um automorfismo
(bijecao analitica com inversa analitica) de M. Como todo automorfismo de M,
que e' um toro, se levanta a uma translacao de C, e como g tem um ponto
periodicos de periodo n, entao g^n e' a identidade de M, e acabou. Nao e'
isso, Nicolau ?
   Abracos,
Gugu


Olá Gugu,

o porismo de Poncelet é um resultado muito interessante. Eu gostaria de
saber mais a respeito, se você souberem dar referências sobre quais assuntos
preciso saber para estudá-lo e onde posso encontrar sua demonstração, eu
ficaria grato.

Outro dia fiz a pergunta sobre qual a soma máxima dos coeficientes dos
módulos de um polinômio de grau = n com a propriedade |p(x)|=1 para todo
|x|=1. Você respondeu que possivelmente eram os polinômios de Chebychev. Eu
recebi sua resposta, e fui estudar em um livro de Teoria da Aproximação
alguns fatos básicos, antes de estudar os polinômios de Chebychev. Em alguns
dias, responderei à sua mensagem.

Grato!
Um abraço,
Duda.

From: Carlos Gustavo Tamm de Araujo Moreira [EMAIL PROTECTED]
Caro Wagner,
De fato eu ainda nao achei nenhuma evidencia de que o Poncelet soubesse
 como provar o seu porisma. Seria bom se alguem tivesse alguma boa
referencia
 sobre isso...As provas que eu e o Nicolau conhecemos nao sao nada
 elementares (a mais simples usa fatos sobre superficies de Riemann -
 nao e' isso, Nicolau ?).
Abracos,
 Gugu

 
 A palavra porisma eh, na verdade grega.
 Euclides escreveu um livro com exatamente este titulo:
 Porismas, mas esta perdido.  Infelizmente nao sabemos
 o que ele continha.
 Sobre o significado que dei dessa palavra, quero dizer que
 copiei do dicionario Pillet et Dumoulin editado no
 seculo 19.
 Eh claro que o porisma de Poncelet nao eh nada facil, mas
 talvez essa palavra tambem tenha sido usada como sinonimo
 de conjectura, algo que nao eh ainda um teorema.
 
 Abracos,
 
 Wagner.
 
 
 --
 From: Nicolau C. Saldanha [EMAIL PROTECTED]
 To: [EMAIL PROTECTED]
 Subject: Re: [obm-l] Duvidas
 Date: Tue, Jun 3, 2003, 9:16 PM
 
 
  On Tue, Jun 03, 2003 at 08:47:46PM -0300, Eduardo Wagner wrote:
  Porismo nao consta dos nossos dicionarios.
  Porismo vem do frances porisme que significa
  uma afirmacao muito facil de demonstrar. Pode ser um lema
  ou um corolario, algo que nao tem o status de teorema.
 
  O porismo de Poncelet é tudo, menos muito fácil de demonstrar.
 
  Para quem não sabe, o porismo diz o seguinte:
 
  Sejam C1 e C2 dois círculos, C2 dentro de C1 mas não concêntricos.
  Seja P0 um ponto de C1; por P0 trace uma tangente a C2 para obter P1,
  a outra interseção desta tangente com C1, trace outra tangente a C2
  para obter P2 e assim por diante como na figura em anexo.
  Suponha que Pn = P0 onde n é um inteiro positivo. Comece agora com
outro
  ponto Q0 em C1 e repita a construção para obter Q1, Q2, ..., Qn.
 
  Prove que Qn = Q0.
 
  A mesma coisa vale para elipses ou cônicas em geral
  e isso segue facilmente do caso com círculos usando
  transformações projetivas.
 
  []s, N.
 
 =
 Instruções para entrar na lista, sair da lista e usar a lista em
 http://www.mat.puc-rio.br/~nicolau/olimp/obm-l.html
 =

 =
 Instruções para entrar na lista, sair da lista e usar a lista em
 http://www.mat.puc-rio.br/~nicolau/olimp/obm-l.html

Re: [obm-l] Duvidas

2003-06-05 Por tôpico Carlos Gustavo Tamm de Araujo Moreira
   O caso n=3 (ou uma coisa equivalente) caiu, se eu nao me engano, na 10a. 
OBM (que eu fiz; na verdade esse foi o problema que eu nao fiz), e segue
tambem, se eu nao me engano, da formula para a distancia entre o
circuncentro e o incentro de um triangulo (acho que e' raiz(R^2-2Rr)).
   Abracos,
Gugu 

On Thu, Jun 05, 2003 at 01:31:09AM -0300, Claudio Buffara wrote:
 on 04.06.03 16:49, Cláudio (Prática) at [EMAIL PROTECTED]
 wrote:
  
  Uma versão mais simples desse resultado seria:
  Dadas duas circunferências C1 e C2, com C2 interna a C1, se existe um
  triângulo que está ao mesmo tempo inscrito em C1 e circunscrito a C2 (ou
  seja, os 3 lados tangentes a C2), então qualquer triângulo circunscrito a C2
  estará inscrito em C1.
 
 Nao precisa nem responder. No caminho pra casa percebi que tinha falado
 besteira. Acho que tambem eh necessario que o segundo triangulo, alem de
 circunscrito a C2, tenha 2 vertices sobre C1 - entao, o 3o. vertice tambem
 estara sobre C1.

O caso n=3 do porismo de Poncelet segue de Pascal (ou algum destes
outros teoremas sobre cônicas, eu confundo tudo). O difícil é o caso geral.

[]s, N.
=
Instruções para entrar na lista, sair da lista e usar a lista em
http://www.mat.puc-rio.br/~nicolau/olimp/obm-l.html
=

=
Instruções para entrar na lista, sair da lista e usar a lista em
http://www.mat.puc-rio.br/~nicolau/olimp/obm-l.html
=


Re: Re: [obm-l] Problema 5 da OBMU

2003-06-03 Por tôpico Carlos Gustavo Tamm de Araujo Moreira
   Oi Claudio,
   Bom problema. De fato, sup(A)=e. Voce(s) quer(em) pensar mais ou quer(em)
ver uma solucao ?
   Abracos,
Gugu
 

Oi, Gugu e Luis:

Baseado na ultima mensagem (do Gugu) temos um novo 
problema derivado desse:

Qual a maior base de logaritmos para a qual a serie converge 
ou, mais precisamente, seja:
A = {b em R tais que para logs na base b a serie converge}
Quem eh sup(A) ?

Pelo que o Gugu disse, 2 = sup(A) = e.
 
Um abraco,
Claudio.
-- Cabeçalho inicial  ---

De: [EMAIL PROTECTED]
Para: [EMAIL PROTECTED]
Cópia: 
Data: Fri, 30 May 2003 04:04:13 -0300 (EST)
Assunto: Re: [obm-l] Problema 5 da OBMU

Caro Luis,
Nao sei se o Rousseau entendeu o enunciado. Acho que ele 
esta' pensando
 que o numero de logs e' constante, e ai o resultado e' bem 
classico. Nesse
 problema o numero de logs em cada termo depende de n (ou 
de x, na integral). 
 Ou seja: a funcao e' 1/x entre 1 e e, 1/(x.log(x)) entre e e e^e e 
assim por
 diante. E' um fato interessante (que a meu ver mostra o quao 
delicada e'a
 questao da convergencia dessa serie) que, se trocarlos log 
(logaritmo natural) 
 por lg (logaritmo na base 2), a serie, em vez de divergir, 
converge.
Abracos,
 Gugu
 
 
 Sauda,c~oes,
 
 Apaguei a msg original do Gugu. O que sobrou foi:
 
 Caros colegas,
 Para uma serie cuja discussao sobre convergencia e'
 mais delicada, vejam o problema 5 da ultima OBM
 universitaria.
 Trata-se da serie
 Soma(n=1)(1/(n.log(n).log log (n). ... . log log ... log(n))),
 onde os logaritmos sao naturais, e o numero de termos
 no produto depende de n:
 paramos no ultimo log log ... log(n) que e' maior ou igual a
 1.
 
 Antes de ver a solução proposta na Eureka, propus o
 problema pro prof. Rousseau. Sua resposta demorou um
 pouco por razões que não vêm ao caso mas chegou.
 
 Recentemente falou-se do teste da integral numa outra
 série . pelo Salvador???
 Mais um uso do mesmo teste.
 
 []'s
 Luis
 
 ===
 The series with nth term
 1/(n log n log log n \cdots log
 Let log_k x = log \cdots \log x with
 k iterates.  To see that \int_a^R dx/(x log_k x )
 tends to infinity with R, argue by induction and set x = e^u.
 Then the integral in question becomes
 \int_{log a}^{log R} du/(u \log_{k-1} u)
 which tends to infinity with R by induction.
 
 Alternatively, one can avoid integrals by using
 the Cauchy condensation test.
 
 Cecil
 ===
 
 
 
=

 Instruções para entrar na lista, sair da lista e usar a lista em
 http://www.mat.puc-rio.br/~nicolau/olimp/obm-l.html
 
=

 
 
==
===
 Instruções para entrar na lista, sair da lista e usar a lista em
 http://www.mat.puc-rio.br/~nicolau/olimp/obm-l.html
 
==
===
 
 

=
Instruções para entrar na lista, sair da lista e usar a lista em
http://www.mat.puc-rio.br/~nicolau/olimp/obm-l.html
=

=
Instruções para entrar na lista, sair da lista e usar a lista em
http://www.mat.puc-rio.br/~nicolau/olimp/obm-l.html
=


Re: [obm-l] Problemas sobre sequencias recorrentes

2003-06-03 Por tôpico Carlos Gustavo Tamm de Araujo Moreira
   Caros colegas,
   A solucao do Marcio para o problema 3 abaixo esta' otima, mas pelo que eu
entendi do enunciado, ele calculou 1-q_n, onde q_n e' a probabilidade
pedida. Assim, q_n e' igual a
1-((10+5sqrt(2))/16).((2+2sqrt(2))/5)^n-((10-5sqrt(2))/16).((2-2sqrt(2))/5)^n.
Como eu tinha combinado com o Marcio vou fazer o problema 2. Eu escrevi
para o Morgado sobre o enunciado desse problema, pois eu achava esquisito
que o sujeito recebesse mensalmente mas so' aplicasse anualmente (ele devia
guardar uma parte da renda no colchao todo mes e so' aplica-la no fim do
ano...). A resposta do Morgado foi a seguinte:

Esse problema da renda eh um problema do Boole!
Saxoes costumam discutir salarios anuais. Agora, esse problema estah mal
transcrito. Tudo era anual!

   Assim, latinizando o problema (i.e., trocando anualmente por
mensalmente), e trocando mes i por mes j, para nao confundir com a
taxa, ele fica assim:
2) o salario de carmelino no mes n e' s_n=a+bn. Sua renda mensal e' formada
pelo salario e pelos juros de suas aplicacoes financeiras.Ele poupa
mensalmente 1/p de sua renda e investe sua poupanca a juros mensais de taxa
i. Determine a renda de carmelino no mes j.

   Agora vamos la': 
   Sejam r_j a renda e T_j o total aplicado no mes j. Temos, para todo j=1,
(1)   r_j=s_j+i.T_(j-1)=a+bj+i.T_(j-1)  ;
(2)   T_j=T_(j-1)+r_j/p , 
   com T_0=0.
   De (1), temos T_(j-1)=(r_j-a-bj)/i, e T(j)=(r_(j+1)-a-b(j+1))/i.
Substituindo em (2), obtemos r_j/p=T_j-T_(j-1)=(r_(j+1)-r_j-b)/i, donde
r_(j+1)=((i+p)/p).r_j+b. A solucao geral desse tipo de recorrencia e'
r_j=A.((i+p)/p)^j+B, para alguma escolha das
constantes A e B (pois (i+p)/p nao e' 1; um jeito de ver isso e' notar que a
recorrencia implica r_(j+2)-r_(j+1)=((i+p)/p).(r_(j+1)-r_j), e logo
d_j=r_(j+1)-r_j e' uma PG de razao (i+p)/p, bastando soma-la para obter uma
formula para r_j). Resolvendo o sistema
A.((i+p)/p)+B = r_1 = a+b
A.((i+p)/p)^2+B = r_2 = a+2b+(i/p).(a+b),
obtemos A=p(ai+b(p+i))/(i(p+i)) e B=-bp/i, e portanto
r_j=p(ai+b(p+i))/(i(p+i)).((i+p)/p)^j-bp/i.

   Abracos,
Gugu


Re: [obm-l] Raiz QuadradaCumprindo o prometido.. :)
2) Algo que eh util em varios problemas de recorrencia eh ir calculando =
os 1o termos da sequencia para ver o que esta acontecendo:
No mes 0, Carmelino (C) nada fez e portanto tem M(0) =3D S(0) =3D a (M =
de montante total).
Ja no mes 1, C ganha S(1) =3D a + b de salario e ele ganha i*M(0) de =
juros (pois os juros incidem sobre=20

3) Seja p(n) a resposta pedida. p(1)=3Dp(2)=3D1, pq pra ganhar taca sao =
necessarios ao menos 3 torneios. Vamos olhar para p(3):
Para que o torneio nao termine no 3o torneio, eh razoavel dividir a =
situacao em dois casos:
a) Se o vencedor do 2o torneio for diferente do vencedor do 1o (o =
que ocorre com probabilidade 4/5), entao o vencedor do torneio3 pode ser =
qq um que a taca nao sera ganha.
b) Se por outro lado, o vencedor V(2) do 2o torneio for igual ao do =
1o - V(1) (o que ocorre com prob. 1/5), entao para a taca nao ser ganhar =
no 3o,basta que V(3) seja diferente de V(2) (o que ocorre com prob. =
4/5).=20
Somando, veja que p(3) =3D 4/5  + 1/5 * 4/5  =3D 24/25

Agora p(4):
a) Se V(2) !=3D V(1) (prob. 4/5), entao o problema agora eh: Qual a =
probabilidade de, nos torneios 2,3,4, a taca nao ser dada para ninguem. =
E isso eh justamente p(3) (apenas os torneios mudaram de nome).=20
b) Se V(2)=3DV(1) (prob. 1/5), entao o torneio 3 tem que ser ganho =
por um time diferente de V(1) (prob. 4/5), e dai basta que a taca nao =
seja ganha nos torneios 3,4 (essa probabilidade eh p(2)=3D1).
Portanto, p(4) =3D 4/5 * p(3) + 1/5 * 4/5 * p(2)

Esse raciocinio pode ser generalizado de maneira analoga:
Para calcular p(n+2), divida em dois casos:
a) Se V(2)!=3DV(1) (prob. 4/5), entao basta saber a probabilidade de =
a taca nao ser ganha nos torneios 2,3,...,n+2, que eh p(n+1).
b) Se V(2)=3DV(1) (prob. 1/5), entao deve-se ter V(3)!=3DV(2) =
(prob.4/5) e dai a taca nao deve ser ganha nos torneior 3,4,...,n+2, o q =
ocorre com prob. p(n).
Logo, p(n+2) =3D 4/5 * p(n+1) + 1/5 * 4/5 * p(n), i.e, 25p(n+2) =3D =
20p(n+1) + 4p(n).
Resolvendo a eq. caracteristica correspondente: 25t^2 - 20t - 4 =3D 0, =
donde t =3D (2 +- 2sqrt(2)) / 5.
Logo, p(n) =3D A* [(2+2sqrt(2))/5]^n + B*[(2-2sqrt(2))/5]^n=20
Analisando p(1) e p(2) (ou mais simples ainda, note que pondo n=3D0 na =
recorrencia, tem-se p(0) =3D (25-20)/4 =3D 5/4 para a recorrencia fazer =
sentido em 0):
A+B =3D 5/4
2*(A+B) + 2sqrt(2)*(A-B) =3D 5 =3D A-B =3D 5sqrt(2)/8   =20
Logo A =3D [10+5sqrt(2)]/16 e B =3D [10-5sqrt(2)]/16.

Isso fecha o problema..

O 2 ja tem dono, serah feito em breve :))  O fato de ele receber =
mensalmente e soh poupar anualmente parece deixar a coisa um pouquinho =
mais chata de ser escrita.



  - Original Message -=20
  From: Marcio=20
  To: [EMAIL PROTECTED]
  Sent: Saturday, May 24, 2003 9:45 AM
  Subject: Re: [obm-l] Problemas sobre sequencias 

Re: [obm-l] integral

2003-06-03 Por tôpico Carlos Gustavo Tamm de Araujo Moreira
   De fato o Mathematica 3.0 for Solaris diz o seguinte:
In[1]:= Integrate[Sin[x]/(1+x),x]

Out[1]= -(CosIntegral[1 + x] Sin[1]) + Cos[1] SinIntegral[1 + x]

In[2]:= ? SinIntegral
SinIntegral[x] gives the sine integral Integrate[Sin[t]/t, {t, 0, x}].

In[3]:= ? CosIntegral
CosIntegral[x] gives the cosine integral EulerGamma + Log[x] +
   Integrate[(Cos[t] - 1)/t, {t, 0, x}],
  
   e nao a resposta mencionada junto com o enunciado...

   Abracos,
Gugu



Nao consegui achar a primitiva desta funcao. A funcao apresentada pelo
Mathematica decididamente nao eh uma primitiva de sen(x)/(1+x), pois sua
derivada eh cos(x-Log(1+x)). (1-1/(1+x)) = (cos(x-Log(1+x))). (x/(1+x)),
bem diferente de sen(x)/(1+x).
Artur 

-Original Message-
From: [EMAIL PROTECTED] 
[mailto:[EMAIL PROTECTED] On Behalf Of carlos augusto
Sent: Monday, June 02, 2003 4:05 PM
To: [EMAIL PROTECTED]
Cc: [EMAIL PROTECTED]
Subject: [obm-l] integral


Sou aluno do 1º período do curso de ciência da
computação, e não consegui responder a seguinte
questão.

Resolver a integral: 
  /
  | Sen(x)
  | -- dx 
  | 1 + x
  /
resposta: Sen(x - Log(1 + x)) by Mathematica.


___
Yahoo! Mail
Mais espaço, mais segurança e gratuito: caixa postal de 6MB, 
antivírus, proteção contra spam. http://br.mail.yahoo.com/ 
===
==
Instruções para entrar na lista, sair da lista e usar a lista 
em http://www.mat.puc-rio.br/~nicolau/olimp/obm-l.html
===
==


=
Instruções para entrar na lista, sair da lista e usar a lista em
http://www.mat.puc-rio.br/~nicolau/olimp/obm-l.html
=

=
Instruções para entrar na lista, sair da lista e usar a lista em
http://www.mat.puc-rio.br/~nicolau/olimp/obm-l.html
=


Re: [obm-l] Problemas sobre sequencias recorrentes

2003-06-03 Por tôpico Carlos Gustavo Tamm de Araujo Moreira
   So' uma observacao:
   Essa solucao que eu escrevi esta' admitindo que o primeiro salario e' o
do mes 1. Se for o do mes 0 a resposta muda um pouco: passaremos a ter
A+B = r_0 = a
A.((i+p)/p)+B = r_1 = a+b+(i/p).a,  
donde A=(bp+ai)/i e B=-bp/i, donde 
r_j=((bp+ai)/i).((i+p)/p)^j-bp/i.
   Esse resultado saiu mais bonitinho, o que me faz gostar dessa
interpretacao... Seria interessante saber a interpretacao do Morgado...
   Abracos,
Gugu 


   Caros colegas,
   A solucao do Marcio para o problema 3 abaixo esta' otima, mas pelo que eu
entendi do enunciado, ele calculou 1-q_n, onde q_n e' a probabilidade
pedida. Assim, q_n e' igual a
1-((10+5sqrt(2))/16).((2+2sqrt(2))/5)^n-((10-5sqrt(2))/16).((2-2sqrt(2))/5)^n.
Como eu tinha combinado com o Marcio vou fazer o problema 2. Eu escrevi
para o Morgado sobre o enunciado desse problema, pois eu achava esquisito
que o sujeito recebesse mensalmente mas so' aplicasse anualmente (ele devia
guardar uma parte da renda no colchao todo mes e so' aplica-la no fim do
ano...). A resposta do Morgado foi a seguinte:

Esse problema da renda eh um problema do Boole!
Saxoes costumam discutir salarios anuais. Agora, esse problema estah mal
transcrito. Tudo era anual!

   Assim, latinizando o problema (i.e., trocando anualmente por
mensalmente), e trocando mes i por mes j, para nao confundir com a
taxa, ele fica assim:
2) o salario de carmelino no mes n e' s_n=a+bn. Sua renda mensal e' formada
pelo salario e pelos juros de suas aplicacoes financeiras.Ele poupa
mensalmente 1/p de sua renda e investe sua poupanca a juros mensais de taxa
i. Determine a renda de carmelino no mes j.

   Agora vamos la': 
   Sejam r_j a renda e T_j o total aplicado no mes j. Temos, para todo j=1,
(1)   r_j=s_j+i.T_(j-1)=a+bj+i.T_(j-1)  ;
(2)   T_j=T_(j-1)+r_j/p , 
   com T_0=0.
   De (1), temos T_(j-1)=(r_j-a-bj)/i, e T(j)=(r_(j+1)-a-b(j+1))/i.
Substituindo em (2), obtemos r_j/p=T_j-T_(j-1)=(r_(j+1)-r_j-b)/i, donde
r_(j+1)=((i+p)/p).r_j+b. A solucao geral desse tipo de recorrencia e'
r_j=A.((i+p)/p)^j+B, para alguma escolha das
constantes A e B (pois (i+p)/p nao e' 1; um jeito de ver isso e' notar que a
recorrencia implica r_(j+2)-r_(j+1)=((i+p)/p).(r_(j+1)-r_j), e logo
d_j=r_(j+1)-r_j e' uma PG de razao (i+p)/p, bastando soma-la para obter uma
formula para r_j). Resolvendo o sistema
A.((i+p)/p)+B = r_1 = a+b
A.((i+p)/p)^2+B = r_2 = a+2b+(i/p).(a+b),
obtemos A=p(ai+b(p+i))/(i(p+i)) e B=-bp/i, e portanto
r_j=p(ai+b(p+i))/(i(p+i)).((i+p)/p)^j-bp/i.

   Abracos,
Gugu


Re: [obm-l] Raiz QuadradaCumprindo o prometido.. :)
2) Algo que eh util em varios problemas de recorrencia eh ir calculando =
os 1o termos da sequencia para ver o que esta acontecendo:
No mes 0, Carmelino (C) nada fez e portanto tem M(0) =3D S(0) =3D a (M =
de montante total).
Ja no mes 1, C ganha S(1) =3D a + b de salario e ele ganha i*M(0) de =
juros (pois os juros incidem sobre=20

3) Seja p(n) a resposta pedida. p(1)=3Dp(2)=3D1, pq pra ganhar taca sao =
necessarios ao menos 3 torneios. Vamos olhar para p(3):
Para que o torneio nao termine no 3o torneio, eh razoavel dividir a =
situacao em dois casos:
a) Se o vencedor do 2o torneio for diferente do vencedor do 1o (o =
que ocorre com probabilidade 4/5), entao o vencedor do torneio3 pode ser =
qq um que a taca nao sera ganha.
b) Se por outro lado, o vencedor V(2) do 2o torneio for igual ao do =
1o - V(1) (o que ocorre com prob. 1/5), entao para a taca nao ser ganhar =
no 3o,basta que V(3) seja diferente de V(2) (o que ocorre com prob. =
4/5).=20
Somando, veja que p(3) =3D 4/5  + 1/5 * 4/5  =3D 24/25

Agora p(4):
a) Se V(2) !=3D V(1) (prob. 4/5), entao o problema agora eh: Qual a =
probabilidade de, nos torneios 2,3,4, a taca nao ser dada para ninguem. =
E isso eh justamente p(3) (apenas os torneios mudaram de nome).=20
b) Se V(2)=3DV(1) (prob. 1/5), entao o torneio 3 tem que ser ganho =
por um time diferente de V(1) (prob. 4/5), e dai basta que a taca nao =
seja ganha nos torneios 3,4 (essa probabilidade eh p(2)=3D1).
Portanto, p(4) =3D 4/5 * p(3) + 1/5 * 4/5 * p(2)

Esse raciocinio pode ser generalizado de maneira analoga:
Para calcular p(n+2), divida em dois casos:
a) Se V(2)!=3DV(1) (prob. 4/5), entao basta saber a probabilidade de =
a taca nao ser ganha nos torneios 2,3,...,n+2, que eh p(n+1).
b) Se V(2)=3DV(1) (prob. 1/5), entao deve-se ter V(3)!=3DV(2) =
(prob.4/5) e dai a taca nao deve ser ganha nos torneior 3,4,...,n+2, o q =
ocorre com prob. p(n).
Logo, p(n+2) =3D 4/5 * p(n+1) + 1/5 * 4/5 * p(n), i.e, 25p(n+2) =3D =
20p(n+1) + 4p(n).
Resolvendo a eq. caracteristica correspondente: 25t^2 - 20t - 4 =3D 0, =
donde t =3D (2 +- 2sqrt(2)) / 5.
Logo, p(n) =3D A* [(2+2sqrt(2))/5]^n + B*[(2-2sqrt(2))/5]^n=20
Analisando p(1) e p(2) (ou mais simples ainda, note que pondo n=3D0 na =
recorrencia, tem-se p(0) =3D (25-20)/4 =3D 5/4 para a recorrencia fazer =
sentido em 0):
A+B =3D 5/4
2*(A+B) + 2sqrt(2)*(A-B) 

Re: [obm-l] Re: [obm-l] Questão do cartaz

2003-06-03 Por tôpico Carlos Gustavo Tamm de Araujo Moreira
   Caro Raul,
   Nao entendi sua objecao quanto a solucao que esta' em
http://www.obm.org.br/provas/obm2002/obm20021fase.htm
Nessa solucao, comecamos a elevar de cima para baixo (em particular 7^7^7 e'
7^823543, e nao 7^49). Qual e' a solucao que voce tem em maos ?
   Abracos,
   Gugu


Obrigado Carlos pela resposta bem elaborada. Mas agora então me parece
errada a resolução da questão do cartaz que pergunta o último dígito de
7^7^7^7...(onde aparecem 2002 setes). Na resolução que acompanha o gabarito
é feita a análise que 7^7 termina em 3, ao elevar a 7 novamente termina em
7. Assim é feita a conclusão que ficará alternando 3 e7 ao continuar
elevando. Como tem 2002 números 7, conclui-se que terminará em 3. Não está
indo contra a convenção mais aceita é que ^ é associativa à direita?
Agradeço aos que quiserem realmente ajudar.
Raul

- Original Message -
From: Carlos César de Araújo [EMAIL PROTECTED]
To: [EMAIL PROTECTED]
Sent: Thursday, May 29, 2003 2:05 PM
Subject: [obm-l] Re: [obm-l] Re: [obm-l] Re: [obm-l] Questão do cartaz


 Raul,

 Você pergunta:

   Uma dúvida então: está errado ensinar que 2^3^2=2^9 por não haver
  parêntesis?

 Repetindo o que eu disse, a convenção mais aceita é que ^ é associativa à
 direita, de modo que 2^3^2=2^(3^2)=2^9=512. Portanto, ensinar que
 2^3^2=2^9 é simplesmente uma forma de aderir a essa convenção. A maioria
 dos softwares com os quais trabalho no meu site seguem essa regra. Uma
 exceção é o Excel: se você digitar

 =2^3^2

 numa célula e pressionar Enter, verá 64 como resultado. Isto mostra que
o
 Excel decodifica a expressão associando pela ESQUERDA.

 Algumas apostilas de cursinho e alguns livros que consultei
  trazem exercícios que diferenciam (2^3)^2=2^6 do exemplo anterior.

 Sim, a expressão (2^3)^2 tem que ser diferenciada de 2^3^2=2^(3^2) porque,
 como eu ressaltei, a operação ^ não é associativa. Onde está a dúvida? Era
 isso mesmo que você queria dizer?

 Infelizmente, questões SINTÁTICAS e METODOLÓGICAS como essas não são
 discutidas SISTEMATICAMENTE nos cursos tradicionais de matemática. (Isto
 não acontece em cursos de Lógica Matemática ou de Programação de
 Computadores.) Conseqüentemente, os alunos aprendem a fazer cálculos e
 resolver problemas padronizados, mas não aprendem a pensar e CRITICAR
fatos
 estabelecidos. Tão importante quanto a arte de resolver problemas é a
 capacidade de organizar o conhecimento em um corpo coeso de fatos e
 CONVENÇÕES baseadas em julgamentos inteligentes. Matemáticos não são
apenas
 resolvedores de problemas; são, também, construtores de teorias. Aqui vai
um
 exercício para você treinar a sua observação e senso crítico: por que se
 convenciona que a multiplicação tem precedência sobre a adição? Isto é,
por
 que se convenciona que a + b*c = a+(b*c) e não que a + b*c = (a+b)*c?
 PENSE sobre isto e poderá chegar à resposta por si mesmo (como eu próprio
 cheguei).

 Carlos César de Araújo
 Matemática para Gregos  Troianos
 www.gregosetroianos.mat.br
 Belo Horizonte, MG

  - Original Message -
   Raul,
  
   A operação binária (a,b)-- a^b não é associativa, de modo que, em
   princípio, a expressão
  
   a^b^c
  
   é ambígua: significaria (a^b)^c OU a^(b^c)? Contudo, repare que uma
 dessas
   alternativas leva a um resultado bem definido: (a^b)^c=a^(bc). Como
   conseqüência (e abreviando um pouco o que poderia ser uma longa
 discussão
   metodológica), convencionou-se que a operação (a,b)-- a^b é
ASSOCIATIVA
 À
   DIREITA. Ou seja, por definição,
  
   a^b^c = a^(b^c).
  
   Em particular,
  
   7^7^7 = 7^(7^7) = 7^49.
  
   PS: Questões como essas são discutidas detalhadamente num dos
capítulos
 do
   meu CD-ROM Números (cujas vendas, no momento, estão paralisadas à
 espera
   de acordos viáveis com distribuidoras em território nacional.)
  
   Carlos César de Araújo
   Matemática para Gregos  Troianos
   www.gregosetroianos.mat.br
   Belo Horizonte, MG
  
   - Original Message -
  
   Olá a todos.
   No cartaz da OBM 2003 há uma questão para ensino médio que
pergunta
  qual
   o último algarismo de sete elevado a sete elevado a sete...(com 2002
  setes).
   Acontece que não há parêntesis entre os expoentes. Na resolução da
 questão
   eu achei que tudo foi feito como se houvesse parêntesis. Em resumo
sete
   elevado a sete elevado a sete foi tratado como sete elevado a 49 e não
  como
   sete elevado a 823543. Quero saber onde eu estou errado.
   Agradeço desde já.
   Raul
  
  
  
  
 =
   Instruções para entrar na lista, sair da lista e usar a lista em
   http://www.mat.puc-rio.br/~nicolau/olimp/obm-l.html
  
 =
  
   Esta mensagem foi verificada pelo E-mail Protegido Terra.
   Scan engine: VirusScan / Atualizado em 28/05/2003 / Versão: 1.3.13
   Proteja o seu e-mail Terra: 

Re: [obm-l] Problema 5 da OBMU

2003-05-30 Por tôpico Carlos Gustavo Tamm de Araujo Moreira
   Caro Luis,
   Nao sei se o Rousseau entendeu o enunciado. Acho que ele esta' pensando
que o numero de logs e' constante, e ai o resultado e' bem classico. Nesse
problema o numero de logs em cada termo depende de n (ou de x, na integral). 
Ou seja: a funcao e' 1/x entre 1 e e, 1/(x.log(x)) entre e e e^e e assim por
diante. E' um fato interessante (que a meu ver mostra o quao delicada e'a
questao da convergencia dessa serie) que, se trocarlos log (logaritmo natural) 
por lg (logaritmo na base 2), a serie, em vez de divergir, converge.
   Abracos,
Gugu


Sauda,c~oes,

Apaguei a msg original do Gugu. O que sobrou foi:

Caros colegas,
Para uma serie cuja discussao sobre convergencia e'
mais delicada, vejam o problema 5 da ultima OBM
universitaria.
Trata-se da serie
Soma(n=1)(1/(n.log(n).log log (n). ... . log log ... log(n))),
onde os logaritmos sao naturais, e o numero de termos
no produto depende de n:
paramos no ultimo log log ... log(n) que e' maior ou igual a
1.

Antes de ver a solução proposta na Eureka, propus o
problema pro prof. Rousseau. Sua resposta demorou um
pouco por razões que não vêm ao caso mas chegou.

Recentemente falou-se do teste da integral numa outra
série . pelo Salvador???
Mais um uso do mesmo teste.

[]'s
Luis

===
The series with nth term
1/(n log n log log n \cdots log log \cdots log n) diverges
by the integral test.  Let log_k x = log \cdots \log x with
k iterates.  To see that \int_a^R dx/(x log_k x )
tends to infinity with R, argue by induction and set x = e^u.
Then the integral in question becomes
\int_{log a}^{log R} du/(u \log_{k-1} u)
which tends to infinity with R by induction.

Alternatively, one can avoid integrals by using
the Cauchy condensation test.

Cecil
===


=
Instruções para entrar na lista, sair da lista e usar a lista em
http://www.mat.puc-rio.br/~nicolau/olimp/obm-l.html
=

=
Instruções para entrar na lista, sair da lista e usar a lista em
http://www.mat.puc-rio.br/~nicolau/olimp/obm-l.html
=


Re: [obm-l] 2 Problemas

2003-05-27 Por tôpico Carlos Gustavo Tamm de Araujo Moreira
   Caro Duda,
   O problema 2 e' realmente muito interessante. Acho que para todo n o
maximo e' atingido pelo n-esimo polinomio de Chebyshev P_n(x) (que e'
definido por cos(nx)=P_n(cos(x)), e satisfaz a recorrencia
P_(n+1)(x)=2x.P_n(x)-P_(n-1)(x), P_0(x)=1, P_1(x)=x). O valor da soma dos
modulos dos coeficientes de P_n e' s_n:=((1+raiz(2))^n+(1-raiz(2))^n)/2
(note que (s_n) satisfaz s_(n+1)=2.s_n+s_(n-1)).
   Os polinomios de Chebyshev sao extremais em muitos sentidos, sendo o mais
popular e mais importante o seguinte: se P e' um polinomio real de grau n=1
tal que |P(x)|=1 para -1=x=1, entao o coeficiente lider (de x^n) de P tem
modulo no maximo 2^(n-1), que e' atingido por P_n. E' um bom exercicio
provar isso (Sugestao: P_n(x) tem modulo 1 para os seguintes n+1 valores de
x no intervalo [-1,1]: cos(k.pi/n), com 0=k=n).
   Usando esse resultado (e a prova dele), nao e' muito dificil mostrar que
a soma dos modulos dos coeficientes de um polinomio P(x) de grau n tal que
|P(x)|=1 para -1=x=1 e' no maximo s_n+2.s_(n-1)2.s_n (exercicio). Eu
acho que sei provar que o maximo de fato e' s_n, mas isso da' mais trabalho.
   Abracos,
   Gugu



Caros colegas da lista,

tenho dois problemas a propor. O primeiro é filosófico e pedagógico. O
segundo é sobre polinômios. Lá vão eles:

Problema 1.  O que vocês acham sobre o método de ensino da matemática,
deve-se começar pelo concreto e ir em direção ao abstrato mais geral, ou
seguir o caminho inverso? Por exemplo, estudar topologia geral depois
estudar topologia no R^n e em seguida topologia R, como coisas particulares.
Ou começar pelo R, depois R^n e a visão mais geral. Eu tenho a impressão que
partir de idéias muito abstratas e gerais é uma abordagem que não dá bons
resultados, pois começa-se a partir de um ponto onde a confusão é muito mais
natural de acontecer. Particularmente, eu nunca comecei desse modo, sempre
estudei as coisas na ordem usual. O Halmos, por exemplo, defende a idéia de
estudar espaços de Hilbert simultaneamente a espaços vetoriais de dimensão
finita.

A ordem de criação da matemática é do mais concreto para o mais abstrato,
pelo que compreendo. Depois que se viram muitas estruturas de
características similares (espaços vetoriais com certas propriedades, p.e.),
se retira uma noção mais geral e abstrata (espaços de Hilbert) e daí
retiram-se propriedades mais fracas mas muito gerais. E a teoria continua
num crescendum, até que, tavez, todos os campos se unifiquem numa grande
visão, essa é a minha esperança. A criação da matemática, pelo que tenho na
minha memória, segue a direção concreto -- abstrato. Mas isso não implica
que o ensino deva seguir essa mesma direção. Existem experiências de ensino
ou alguém já estudou algum assunto seguindo a ordem inversa? O que relata
dessas experiências?

Problema 2. Se um polinômio p(x) de grau n (particularmente gostaria de
saber sobre o caso n=3) é tal que |p(t)| = 1 para |t| = 1, então o que
podemos dizer sobre os coeficientes de p(x)? Qual o máximo módulo que eles
podem ter? E sobre a soma em módulo dos coeficientes, qual o máximo?

Este problema me surgiu na aula de Análise no R^n. É certo que tal máximo de
fato existe, pois todas as normas em R^n são equivalentes, mas determinar o
máximo me parece um problema interessante. Não sei se ele tem uma resposta
simples, acho que não, mas pode-se fazer algumas estimativas do máximo.

Um Abraço a todos,
Duda.

=
Instruções para entrar na lista, sair da lista e usar a lista em
http://www.mat.puc-rio.br/~nicolau/olimp/obm-l.html
=

=
Instruções para entrar na lista, sair da lista e usar a lista em
http://www.mat.puc-rio.br/~nicolau/olimp/obm-l.html
=


Re: [obm-l] Re: [obm-l] demonstração de função bijet

2003-03-30 Por tôpico Carlos Gustavo Tamm de Araujo Moreira

Primeiramente, obrigado Carlos por responder a questão. 
O problema é que ainda curso o ensino médio, e não 
conheço os conceitos de derivada. Na verdade, eu tenho a 
resolução dessa questão, mas não entendi alguns pontos 
sobre a verificação da sobrejeção. Estou mandando 
novamente a pergunta, sua respectiva resposta (relativa 
a sobrejeção) e minha dúvida. Fico grato se alguem me 
exclarecer.
 
Demonstre que f, definida no intervalo 0  x  s (s  0) 
do seguinte modo: F(x) = (2x - s)/[x(s - x)] é uma 
função bijetora desse intervalo nos reais.

Notemos que f(x) = [x + (x - s)]/[x(x - s)] = 1/(x - s) 
+ 1/x.

1. Para todo y E R, se y = (2x - s)/[x(s - x)], resulta:
y(xs - x^2) = 2x - s  -  yx^2 + (2 - ys)x - s = 0.

Fazendo g(x) = yx^2 + (2 - ys)x - s, vem:

a · g(0) = y(-s) 
a · g(s) = y(s)
- ag(0) e ag(s) têm sinais opostos  -  existe um x´ 
tal que y = (2x´ - s)/[x´(s - x´)] 
então f é sobrejetora.

(DÚVIDA) Por que g(0) e g(s) são multiplicados por a.
Não entendi a conclusão, ou seja, por que ela é 
sobrejetora?

   Eu tambem nao entendi que negocio e' esse de multiplicar por a, mas
g(0)=-s e g(s)=s tem sinais contrarios, e portanto existe x entre 0 e s com
g(x)=0 (ou seja, a equacao do segundo grau em x dada por g(x)=0, i.e., 
yx^2 + (2 - ys)x - s = 0, tem uma raiz entre 0 e s), e portanto f(x)=y, o 
que prova que f e' sobrejetiva, pois y pode assumir qualquer valor real.
   Abracos,
   Gugu 


obrigado pela atenção.
Ass: Marcelo Paiva

 
__
E-mail Premium BOL
Antivírus, anti-spam e até 100 MB de espaço. Assine já!
http://email.bol.com.br/


=
Instruções para entrar na lista, sair da lista e usar a lista em
http://www.mat.puc-rio.br/~nicolau/olimp/obm-l.html
O administrador desta lista é [EMAIL PROTECTED]
=

=
Instruções para entrar na lista, sair da lista e usar a lista em
http://www.mat.puc-rio.br/~nicolau/olimp/obm-l.html
O administrador desta lista é [EMAIL PROTECTED]
=


Re: [obm-l] fracoes parciais

2003-03-28 Por tôpico Carlos Gustavo Tamm de Araujo Moreira
  Caro Luis,
  Isso so' vale se o grau de P for menor que n, por exemplo: x/(x-1) nao e'
igual a 1/(x-1), como o seu enunciado implicaria... 
  Seja R(x)=soma(k=1 ate' n)([P(a_k) / Q'(a_k)] . [1 / x - a_k]).
R(x) e' uma funcao racional cujo denominador e' o produto para k variando
entre 1 e n de (x-a_k), ou seja,Q(x). Ao multiplicarmos a soma acima por
Q(x), obtemos um polinomio de grau menor que n. Vamos calcular o valor desse
polinomio em a_k: como Q(a_k) vale 0, todos os termos se anulam exceto o
termo [P(a_k) / Q'(a_k)] . [1 / x - a_k]. O produto de Q(x) por esse termo
e' [P(a_k) / Q'(a_k)] . [Q(x) / x - a_k]. Como, pela definicao de derivada,
lim(x-a_k)(Q(x)/(x-a_k))=Q'(a_k), que nao e' 0, pois a_k  e' raiz simples
de Q(x), segue que Q(x).R(x) tende a P(a_k) quando x tende a a_k, para todo
k. Isso mostra que Q(x).R(x)=P(x), pois a diferenca entre os dois lados e'
um polinomio de grau menor que n que se anula nos n pontos a_1,a_2,...,a_n.
   O item ii) e' um corolario imediato do item i).
   Abracos, 
   Gugu

   

This is a multi-part message in MIME format.

--=_NextPart_000_011C_01C2F540.E6DAB480
Content-Type: text/plain;
   charset=iso-8859-1
Content-Transfer-Encoding: quoted-printable

Sauda,c~oes,

Sejam P(x) e Q(x) polin=F4mios e a_k as
(todas) n ra=EDzes simples de Q(x).

Mostre que P(x) / Q(x) =3D \sum_{k=3D1}^n

[P(a_k) / Q'(a_k)] . [1 / x - a_k]  (*)

Ou em LaTeX:

\frac{P(x)}{Q(x)} =3D \sum_{k=3D1}^n
\frac{[P(a_k)}{Q'(a_k)}\frac{1}{x - a_k}

Exemplos:

i)
P(x) =3D 2x + 1
Q(x) =3D x(x - 1)(x - 2)
Q'(x) =3D 3x^2 - 6x + 2

P(0) =3D 1; P(1) =3D 3; P(2) =3D 5
Q'(0) =3D 2; Q'(1) =3D -1; Q'(2) =3D 2

P(x) / Q(x) =3D 1/2x - 3/x-1 + 5/2(x-2)=20

ii)

se P(x) =3D Q'(x), ent=E3o P(x)/Q(x) =3D \sum {1 / x-a_k}.

Como provar (*) ?? Ou refer=EAncias???

Obrigado.

[]'s
Lu=EDs


--=_NextPart_000_011C_01C2F540.E6DAB480
Content-Type: text/html;
   charset=iso-8859-1
Content-Transfer-Encoding: quoted-printable

!DOCTYPE HTML PUBLIC -//W3C//DTD HTML 4.0 Transitional//EN
HTMLHEAD
META http-equiv=3DContent-Type =
content=3Dtext/html;charset=3Diso-8859-1
META content=3DMSHTML 5.50.4807.2300 name=3DGENERATOR
STYLE@font-face {
   font-family: Tahoma;
}
@page Section1 {size: 8.5in 11.0in; margin: 1.0in 1.25in 1.0in 1.25in; }
P.MsoNormal {
   FONT-SIZE: 12pt; MARGIN: 0in 0in 0pt; FONT-FAMILY: Times New Roman
}
LI.MsoNormal {
   FONT-SIZE: 12pt; MARGIN: 0in 0in 0pt; FONT-FAMILY: Times New Roman
}
DIV.MsoNormal {
   FONT-SIZE: 12pt; MARGIN: 0in 0in 0pt; FONT-FAMILY: Times New Roman
}
A:link {
   COLOR: blue; TEXT-DECORATION: underline
}
SPAN.MsoHyperlink {
   COLOR: blue; TEXT-DECORATION: underline
}
A:visited {
   COLOR: blue; TEXT-DECORATION: underline
}
SPAN.MsoHyperlinkFollowed {
   COLOR: blue; TEXT-DECORATION: underline
}
SPAN.EmailStyle17 {
   COLOR: navy; FONT-FAMILY: Arial
}
DIV.Section1 {
   page: Section1
}
/STYLE
/HEAD
BODY lang=3DEN-US vLink=3Dblue link=3Dblue bgColor=3Dwhite
DIVFONT size=3D2Sauda,c~oes,/FONT/DIV
DIVFONT size=3D2/FONTnbsp;/DIV
DIVFONT size=3D2Sejam P(x) e Q(x) polin=F4mios e a_k as/FONT/DIV
DIVFONT size=3D2(todas) n ra=EDzes simples de Q(x)./FONT/DIV
DIVFONT size=3D2/FONTnbsp;/DIV
DIVFONT size=3D2Mostre que P(x) / Q(x) =3D =
\sum_{k=3D1}^n/FONT/DIV
DIVFONT size=3D2/FONTnbsp;/DIV
DIVFONT size=3D2[P(a_k) / Q'(a_k)]nbsp;. [1 / x -=20
a_k]nbsp;nbsp;nbsp;nbsp; nbsp;(*)/FONT/DIV
DIVFONT size=3D2/FONTnbsp;/DIV
DIVFONT size=3D2Ou em LaTeX:/FONT/DIV
DIVFONT size=3D2/FONTnbsp;/DIV
DIVFONT size=3D2\frac{P(x)}{Q(x)} =3D \sum_{k=3D1}^n
DIVFONT size=3D2\frac{/FONTFONT =
size=3D2[P(a_k)}{Q'(a_k)}\frac{1}{x -=20
a_k}/FONT/DIV
DIVnbsp;/DIV
DIVExemplos:/DIV
DIVnbsp;/DIV
DIVi)/DIV
DIVP(x) =3D 2x + 1/DIV
DIVQ(x) =3D x(x - 1)(x - 2)/DIV
DIVQ'(x) =3D 3x^2 - 6x + 2/DIV
DIVnbsp;/DIV
DIVP(0) =3D 1; P(1) =3D 3; P(2) =3D 5/DIV
DIV
DIVQ'(0) =3D 2; Q'(1) =3D -1; Q'(2) =3D 2/DIV
DIVnbsp;/DIV
DIVP(x) / Q(x) =3D 1/2xnbsp;- 3/x-1 + 5/2(x-2)nbsp;/DIV
DIVnbsp;/DIV
DIVii)/DIV
DIVnbsp;/DIV
DIVse P(x) =3D Q'(x), ent=E3o P(x)/Q(x) =3D \sum {1 / x-a_k}./DIV
DIVnbsp;/DIV
DIVComo provar (*) ?? Ou refer=EAncias???/DIV
DIVnbsp;/DIV
DIVObrigado./DIV
DIVnbsp;/DIV
DIV[]'s/DIV
DIVLu=EDs/DIV
DIVnbsp;/DIV/DIV/FONT/DIV/BODY/HTML

--=_NextPart_000_011C_01C2F540.E6DAB480--

=
Instruções para entrar na lista, sair da lista e usar a lista em
http://www.mat.puc-rio.br/~nicolau/olimp/obm-l.html
O administrador desta lista é [EMAIL PROTECTED]
=

=
Instruções para entrar na lista, sair da lista e usar a lista em
http://www.mat.puc-rio.br/~nicolau/olimp/obm-l.html
O administrador desta lista é [EMAIL PROTECTED]
=


Re: [obm-l] Numero redondo

2003-03-28 Por tôpico Carlos Gustavo Tamm de Araujo Moreira
  Bem, pela interpretacao abaixo, que parece razoavel, o problema e' achar
uma solucao de k(k+1)/2-r(r+1)/2=0 (mod n) com 1=rk e k minimo. Temos que
k(k+1)/2-r(r+1)/2=(k-r)(k+r+1)/2. Queremos entao achar dois numeros (k-r e
k+r+1) com paridades distintas, cuja diferenca e' pelo menos 3, cujo produto 
e' multiplo de 2n e cuja soma seja minima. Nesse caso seu produto sera'
igual a 2n (senao podemos cortar fatores deles para que seu produto seja 2n
fazendo a sua soma diminuir - temos so' que cuidar do caso em que ao cortar 
esses fatores obtemos dois fatores com diferenca 1, e nesse caso o produto 
poderia ser igual a 4n no caso otimo - por causa dessas coisas seria mais 
natural comecar com k=0...), e a maior potencia de 2 que divide 2n dividira' 
um deles.  
  Devemos escolher uma tal fatoracao de 2n (ou de 4n) de modo que os fatores
sejam os mais proximos possiveis. E' claro que a forma da solucao otima 
depende de n. Se n=2^u, por exemplo, teremos k=2^u e r=2^u-1 ( esse e' o
unico caso, alem de n=3, em que k=n), e se n  3 e' um primo impar, devemos 
ter k=(n+1)/2 e r=(n-3)/2.Por outro lado, se n=(u+2)(u-1)/2 entao k=u e r=1; 
esse e' o caso em que k=(sqrt(9+8n)-1)/2 e' o menor possivel comparado com n.
   Abracos,
   Gugu

Obs: Se n=36=8.9/2, o produto no caso otimo (16.9=144) e' 4n, e nao 2n
(nesse caso o melhor que podemos conseguir com produto igual a 2n com
diferenca pelo menos 3 entre os fatores, dado que os fatores devem ter
paridade diferente e' 24.3=72, 24+3=27  25=16+9). Nesse caso, portanto,
k=12 e r=3. Esses fenomenos so podem ocorrer quando n e' da forma k(k+1)/2 
(e nem sempre ocorrem nesses casos).


Claudio,

Acho ki o problema inicial nao e so saber que o processo para, mas como =
determinar em qual valor de k o processo pararia para a given N.

Estou meio ki stuck, quem sabe vc pode me ajudar... vou tentar explicar =
minhas observacoes ate agora com alguns exemplos:

para N=3D6
1 2 3 4 5 6
1   2 4   3 - para quando k =3D 5 ocupa a celula 3

para N=3D8
1 2 3 4 5 6 7 8
1 4 2 7 6 3 5   - para quando k =3D 8 ocupa a celula 4

para N=3D11
1 2 3 4 5 6 7 8 9 10 11
1   2 5   34 - para quando k =3D 6 ocupa a celula 10


ficha k sempre ocupa a celula R onde=20
R =3D (1+2+...+k) mod N ou a celula N se R=3D0 ( basta mudar o label da
celula N para 0 )


o processo acaba quando celula R ja esta ocupada, ou seja
existe um a  k para o qual (1+2+...+a) mod N =3D R

outras observacoes (talvez obvias ):

Sum(1,k) - Sum(1,a) =3D xN onde x =3D 1

Sum(1,k)  N

Eu tenho ki ralar, entao paro por aki... a minha pergunta e:
Sera possivel, escrever k em funcao de N?

-Auggy



- Original Message -=20
From: Cl=E1udio (Pr=E1tica) [EMAIL PROTECTED]
To: [EMAIL PROTECTED]
Sent: Friday, March 28, 2003 8:31 AM
Subject: Re: [obm-l] Numero redondo



Oi, JP:

De qualquer forma, com um no. finito N de c=E9lulas (contando-se mod N, =
ou,
equivalentemente, com as c=E9lulas em torno de um c=EDrculo como disse o =
Gugu),
o processo p=E1ra qualquer que seja k, pois pelo princ=EDpio das casas =
de
pombos, depois de no m=E1ximo N passos haver=E1 necessariamente uma =
c=E9lula com
duas fichas.

Um abra=E7o,
Claudio.

- Original Message -
From: Cl=E1udio (Pr=E1tica) [EMAIL PROTECTED]
To: [EMAIL PROTECTED]
Sent: Thursday, March 27, 2003 5:36 PM
Subject: Re: [obm-l] Numero redondo


 Quantas c=E9lulas ou compartimentos existem? Se for um n=FAmero =
infinito,
ent=E3o
 n=E3o p=E1ra nunca. Se for um n=FAmero finito (digamos N), ent=E3o =
qual a regra?
 Volta ao in=EDcio mod N?

 Outra d=FAvida: voc=EA coloca a ficha 1 na c=E9lula 1. A=ED, se voc=EA =
saltar 1
 c=E9lula, ir=E1 colocar a ficha 2 na c=E9lula 3. T=E1 certo isso?

 - Original Message -
 From: [EMAIL PROTECTED]
 To: [EMAIL PROTECTED]
 Sent: Thursday, March 27, 2003 4:31 PM
 Subject: [obm-l] Numero redondo


  Turma,tenho uma questao que esta me matando!!!Temos uma sequencia de
 fichas
  que devemos colocar em celulas assim:coloca a FICHA 1 NUM espa=E7o,e
 indutivamente
  ao se colocar a ficha k em seu compartimento,saltamos k =
compartimentos e
  passamos a colocar a ficha k+1 na proxima celula.O processo para =
quando
  algum compartimento contiver duas fichas.Para quais k o processo =
para?
 
  TEA WITH ME THAT I BOOK YOUR FACE
 
 
  --
  Use o melhor sistema de busca da Internet
  Radar UOL - http://www.radaruol.com.br
 
 
 
 
=3D=3D=3D=3D=3D=3D=3D=3D=3D=3D=3D=3D=3D=3D=3D=3D=3D=3D=3D=3D=3D=3D=3D=3D=3D=
=3D=3D=3D=3D=3D=3D=3D=3D=3D=3D=3D=3D=3D=3D=3D=3D=3D=3D=3D=3D=3D=3D=3D=3D=3D=
=3D=3D=3D=3D=3D=3D=3D=3D=3D=3D=3D=3D=3D=3D=3D=3D=3D=3D=3D=3D=3D=3D=3D
  Instru=E7=F5es para entrar na lista, sair da lista e usar a lista em
  http://www.mat.puc-rio.br/~nicolau/olimp/obm-l.html
  O administrador desta lista =E9 [EMAIL PROTECTED]
 
=3D=3D=3D=3D=3D=3D=3D=3D=3D=3D=3D=3D=3D=3D=3D=3D=3D=3D=3D=3D=3D=3D=3D=3D=3D=
=3D=3D=3D=3D=3D=3D=3D=3D=3D=3D=3D=3D=3D=3D=3D=3D=3D=3D=3D=3D=3D=3D=3D=3D=3D=

  1   2   >